X



トップページ数学
1002コメント362KB
分からない問題はここに書いてね443
レス数が1000を超えています。これ以上書き込みはできません。
0002132人目の素数さん
垢版 |
2018/05/04(金) 23:38:55.65ID:PXR1I5Eh
空海と老子はどっちの方が頭が良いですか?
0003132人目の素数さん
垢版 |
2018/05/05(土) 00:04:01.88ID:BUSpq5hZ
削除依頼を出しました
0004132人目の素数さん
垢版 |
2018/05/05(土) 02:28:14.85ID:cos8i+vX
>>1
スレ立て乙

[前スレ.991] [前スレ.996] を一般化

nが奇数のとき
 f(x) = {x(1-x)}^{(n-1)/2} (2x-1),
 ∫[0,1] {x(1-x)}^(n-1) (2x-1)^2 dx = 2 n! (n-1)! / (2n+1)!

nが偶数(n≧4) のとき
 f(x) = {x(1-x)}^(n/2 -1) (2x-1)^2,
 ∫[0,1] {x(1-x)}^(n-2) (2x-1)^4 dx = 12 n! (n-2)! / (2n+1)!

n=2 のとき
 f(x) = x(1-x),
 1/30

最小ぢゃねゑだろうが…
0006132人目の素数さん
垢版 |
2018/05/05(土) 06:52:13.42ID:wrHy7dhw
>>5
10000まで調べてひとつもない。問題合ってる?
0007132人目の素数さん
垢版 |
2018/05/05(土) 07:43:01.59ID:NY9sEJP5
n=0
0009132人目の素数さん
垢版 |
2018/05/05(土) 09:06:20.58ID:UvwWzyVD
ほーん
なんで答え知っとんのや
スレタイ読める?
0010132人目の素数さん
垢版 |
2018/05/05(土) 09:52:45.87ID:tEdcrB57
答えなんて書かれてない
0011132人目の素数さん
垢版 |
2018/05/05(土) 10:07:42.78ID:1elA/pW/
n,pを自然数とするとき
np^n=p^(n+1)
のpを満たす自然数を求めよ
0012132人目の素数さん
垢版 |
2018/05/05(土) 10:16:18.59ID:NY9sEJP5
無理
0013132人目の素数さん
垢版 |
2018/05/05(土) 10:25:29.02ID:y3sGc/sx
>>11
p=n
0014132人目の素数さん
垢版 |
2018/05/05(土) 12:09:43.97ID:1elA/pW/
>>13 あたり
じゃあnしかないことの証明は?
0015132人目の素数さん
垢版 |
2018/05/05(土) 12:14:15.76ID:NY9sEJP5
p=0
0016132人目の素数さん
垢版 |
2018/05/05(土) 13:10:32.85ID:K6Vuq8aH
>>15 自然数じゃないっすね
0018132人目の素数さん
垢版 |
2018/05/05(土) 13:19:10.86ID:GyiImAHu
>>14
スレタイ読めない文盲はタヒねカス
0019132人目の素数さん
垢版 |
2018/05/05(土) 14:22:48.89ID:UMWHPgmy
>>5が出来る気がしない。これ元ネタなんだろう?数オリ系?
0020132人目の素数さん
垢版 |
2018/05/05(土) 14:31:43.90ID:NwM7wsxx
周長が1の円Cと、周長が1の凸n角形Knがある。ただしn≧3である。
Knの周及び内部の領域をDnとするとき、C全体をDnに含めることは不可能であることを示せ。
0022132人目の素数さん
垢版 |
2018/05/05(土) 15:24:47.69ID:UMWHPgmy
>>21
何すかソレ?
0023132人目の素数さん
垢版 |
2018/05/05(土) 17:01:02.48ID:EMULr+4U
>>5
k:={3^n+2^(n+1)}/{2^n+1}-2=(3^n-2)/(2^n+1)が自然数となる自然数nを求めることが必要十分.
kが自然数と仮定する.
3^n-2=k(2^n+1)…@
@の両辺をmod 2で考えkは奇数2l-1.
@の両辺をmod 3で考えnは偶数2mでありl≡2(mod 3).k=6r-1とかける.
3^{2m}-2=(6r-1)(2^{2m}+1)の両辺をmod 6で考えmは偶数2s.
3^{4s}-2=(6r-1)(2^{4s}+1)の両辺をmod 5で考えr≡2(mod 5).k=30t-19とかける.
3^{4s}-2=(30t-19)(2^{4s}+1)の両辺をmod 10で考え,8≡6^s.
このような非負整数sは存在しない.よって{3^n+2^(n+1)}/{2^n+1}が整数となるnは存在しない.
0024132人目の素数さん
垢版 |
2018/05/05(土) 17:07:06.96ID:NY9sEJP5
0は自然数ですよ。
0025132人目の素数さん
垢版 |
2018/05/05(土) 17:28:15.22ID:vn0wm8Ig
基礎論以外の分野では自然数に0は含まれないと思います
0026132人目の素数さん
垢版 |
2018/05/05(土) 17:34:52.58ID:NwM7wsxx
連続関数に対しては必ず、任意の区間での定積分が定義できますか?
0027132人目の素数さん
垢版 |
2018/05/05(土) 17:41:58.45ID:vn0wm8Ig
できますね
0028132人目の素数さん
垢版 |
2018/05/05(土) 17:42:09.76ID:/CiIYXay
もしかしたらクソ簡単で叩かれるかも知れませんけど、この文の意味がわかりません。
最大数の集合ってことですか?

https://i.imgur.com/1DL0o0c.jpg
0029132人目の素数さん
垢版 |
2018/05/05(土) 17:44:26.32ID:vn0wm8Ig
左と右どっちが大きいですかってことですね
0030132人目の素数さん
垢版 |
2018/05/05(土) 17:50:43.82ID:/CiIYXay
>>29
εの値を変化させて、最大数が1 - ε/2の集合と1/2の集合を作りますよ ってことですか?
0031132人目の素数さん
垢版 |
2018/05/05(土) 17:53:15.41ID:vn0wm8Ig
1-ε/2と1/2の大きい方をaεとするということです
0032132人目の素数さん
垢版 |
2018/05/05(土) 17:55:58.18ID:/CiIYXay
>>31
てっきり集合のことかと勘違いしてました
スレの流れを邪魔してごめんなさい
0033132人目の素数さん
垢版 |
2018/05/05(土) 18:12:45.84ID:/CiIYXay
A:=[0,1)に対して sup=1 となることを証明せよ
0034132人目の素数さん
垢版 |
2018/05/05(土) 18:14:27.77ID:vn0wm8Ig
自明ですね
0035132人目の素数さん
垢版 |
2018/05/05(土) 18:22:30.39ID:/CiIYXay
>>34
この定理を利用して証明して欲しいです

(2)の方を証明するために、さっき質問したようにaεを定義して利用しろと言われているんですが、この先が分かりません
https://i.imgur.com/Ktd7e1o.jpg
0036132人目の素数さん
垢版 |
2018/05/05(土) 18:22:38.64ID:NY9sEJP5
>>25
>基礎論以外の分野では自然数に0は含まれないと思います
ウゾだーー
0037132人目の素数さん
垢版 |
2018/05/05(土) 18:23:17.62ID:y3sGc/sx
>>14
バカ?
0038132人目の素数さん
垢版 |
2018/05/05(土) 18:47:19.33ID:vn0wm8Ig
>>35
任意のεを選択したとき、1-ε<aε<≦1
となることを示しましょう
aεが必ず[0,1)に入ることも示せば、その定理より証明できたことになりますね

>>36
基礎論でも自然数が1以上をさすこともあります
0を含むという定義は小数派ですね
0039132人目の素数さん
垢版 |
2018/05/05(土) 18:55:33.83ID:NwM7wsxx
xy平面上の曲線C:y=f(x)(a≦x≦b)の長さが、ある初等関数g(x)を用いてg(b)-g(a)と表されるとする。
このとき、Cをx軸の周りに一回転させてできる曲面Dの面積も、ある初等関数h(x)を用いてh(a)-h(b)と表せることを示せ。
0040132人目の素数さん
垢版 |
2018/05/05(土) 19:17:32.69ID:UMWHPgmy
>>23
あってる?
0041132人目の素数さん
垢版 |
2018/05/05(土) 19:20:24.86ID:UMWHPgmy
>>39
f(x)は初等関数?
0042132人目の素数さん
垢版 |
2018/05/05(土) 19:30:15.42ID:UMWHPgmy
>>23
失礼しました。合ってるね。mod2、mod3、mod5で考えて必要条件出して、その後mod10で矛盾って何って思ったけど、背理法だからこういう事もあるのね。
0043132人目の素数さん
垢版 |
2018/05/05(土) 19:35:13.52ID:oI3zjkm/
sinh/hは1で y= sinuを微分するとy'= cosxの違いを教えてください
0044132人目の素数さん
垢版 |
2018/05/05(土) 19:36:43.83ID:vn0wm8Ig
0での微分では一致しますね
0045132人目の素数さん
垢版 |
2018/05/05(土) 20:15:57.27ID:oI3zjkm/
>>44
どういうことですか?詳しく教えてほしいです
0046132人目の素数さん
垢版 |
2018/05/05(土) 20:41:42.90ID:UcMeYSP2
東京大学理学部数学科は、東京大学の頂点ですか?
0047132人目の素数さん
垢版 |
2018/05/05(土) 20:48:21.41ID:2uxMooR/
>>4
整数係数だと、るじゃんどるみたいにウマい多項式はないのでしょうか
0048132人目の素数さん
垢版 |
2018/05/05(土) 20:54:35.93ID:S3jerne2
>>39
曲面は
y=f(x)cosθ
z=f(x)sinθ
0049132人目の素数さん
垢版 |
2018/05/05(土) 21:05:10.07ID:r0nMKpCd
217の一番なんですが、この一般項anはどうやって出したのですか?
等差数列と等比数列の足し算なので
an=(4n-2)+[-4(2)↑n-1]
はダメなのですか?
https://i.imgur.com/Xvj5iB6.jpg
https://i.imgur.com/Xvj5iB6.jpg
0051132人目の素数さん
垢版 |
2018/05/05(土) 22:00:51.56ID:NY9sEJP5
>>38
>0を含むという定義は小数派ですね
基礎論とか整数論はやったこたないが、おらっちはずっと0は自然数だお
0052132人目の素数さん
垢版 |
2018/05/05(土) 22:19:15.71ID:/fJjnEsy
微分方程式の入り口に来たんだけど、一般解を求めよって言われたときに、
特異解のことも答えたほうがいいもの?
0053132人目の素数さん
垢版 |
2018/05/05(土) 22:21:39.76ID:66Kv0Reu
>>51
義務教育では1以上と習ったはずですが?
0054132人目の素数さん
垢版 |
2018/05/05(土) 22:23:30.40ID:66Kv0Reu
>>52
入り口ならそもそもないと思いますよ
0055132人目の素数さん
垢版 |
2018/05/05(土) 22:44:32.10ID:k4JBC7qB
>>39は成り立たん気がする。
結局
l(x) = ∫[a,x]√(1+f’(t)^2)dt が初等関数のとき
S(x) = ∫[a,x]πf(t)^2dt も初等関数であるか?
だけどS(x)が初等関数ならf(x)^2=(S’(x)/π)も初等関数になるけど
v(x) = √(1+f’(x)^2)とおいたとき
l(x) = ∫[a,x]v(t)dtは初等関数だけど
f(x)^2 = ∫[a,x]√(v(t)^2-1)dtが楕円積分になる例なんていくらでもありそうな希ガス。
0056132人目の素数さん
垢版 |
2018/05/05(土) 23:01:30.41ID:/fJjnEsy
うーん、普通はないのかな。
微分方程式の変数分離型の最初の問題からいきなり特異解がひっついてるんだけど。
なんか理由があるんだろうな
さんきゅ
0057132人目の素数さん
垢版 |
2018/05/05(土) 23:15:14.78ID:6jnEKWaB
変数分離型で分母が0になったりすると出て来るから
問題しだいだな
0058132人目の素数さん
垢版 |
2018/05/06(日) 00:52:47.38ID:KhrVKVJy
>>39

x軸に垂直な断面の円周の長さは 2πf(x)
xy-平面内の幅は g '(x) dx = √{1 + f '(x)^2} dx
h(b) - h(a) = ∫[a,b] 2πf(x) g '(x) dx
 = 2π{f(b)g(b)-f(a)g(a)} - ∫[a,b] 2πf '(x) g(x) dx
 = 2π{f(b)g(b)-f(a)g(a)} - ∫[a,b] 2π√{g '(x)^2 - 1} g(x) dx

う〜む
0059132人目の素数さん
垢版 |
2018/05/06(日) 01:04:34.75ID:KhrVKVJy
>>49

> 等差数列と等比数列の足し算なので

ちがいます、掛け算です。

 a_n = {(-1)^(n-1)} * 2n
0060132人目の素数さん
垢版 |
2018/05/06(日) 01:06:26.13ID:dOgOsLZa
前スレ>>953で素数の間隔について質問したものです
前スレ>>954さん、>>956さん、情報ありがとうございました

ベルトラン・チェビシェフの定理で証明されているのですね

解答ありがとうございました
0061132人目の素数さん
垢版 |
2018/05/06(日) 01:09:22.86ID:WFF9FQ3N
数列{an}は初項が1の隣接k項間漸化式である。例えばk=3のとき、0でない実数s,tを用いてa(n+2)=sa(n+1)+ta(n)と表される。
この数列がlim[n→∞] a(n)=+∞となるとき、a(j)>a(j-1)なるjを少なくとも何個持つといえるか。
0062132人目の素数さん
垢版 |
2018/05/06(日) 01:44:22.54ID:NxPHNYwv
>>58

> = 2π{f(b)g(b)-f(a)g(a)} - ∫[a,b] 2π√{g '(x)^2 - 1} g(x) dx

の最後の積分のみならず、前の方のf(x)すら初等的という仮定が使えない。
使えるのは “g(x)が初等関数” のみ。
到底できる気がしないんだけど。
0063132人目の素数さん
垢版 |
2018/05/06(日) 02:28:51.16ID:hl5U1bPk
>>23
mod 5でうまくいくようにしたのにmod 5で矛盾するっておかしいと思ったら
r≡2(mod 5)じゃなくてr≡3(mod 5)じゃないか
0067132人目の素数さん
垢版 |
2018/05/06(日) 15:36:46.99ID:7UgnNpKw
じゃ、次の数列の漸化式教えて、
8,9,7,9,3,2,3,8,4,6,…
ある規則で並んでるんだけどね
0068132人目の素数さん
垢版 |
2018/05/06(日) 15:57:55.67ID:i0xgdZwj
>>67

> じゃ、次の数列の漸化式教えて、
> 8,9,7,9,3,2,3,8,4,6,…
> ある規則で並んでるんだけどね

教えて か〜ら〜の〜 ある規則で並んでるんだけどねwww
0069132人目の素数さん
垢版 |
2018/05/06(日) 16:06:01.75ID:KhrVKVJy
>>67

 a_n = [10^n・π] - 10・[10^(n-1)・π]  (小数点下n桁目)

3,1,4,1,5,9,2,6,5,3,5,8,9,7,9,3,2,3,8,4,6,2,6,4,3,3,8,3,2,7,9,5
0070132人目の素数さん
垢版 |
2018/05/06(日) 16:48:52.61ID:KhrVKVJy
>>64

√(nn+k) -n - k/(2k+1)
= k/{√(nn+k) +n} - k/(2n+1)
= k{(n+1) - √(nn+k)} / ( {√(nn+k) +n} (2n+1) )
= k(2n+1-k) / ( {(n+1)+√(nn+k)} {√(nn+k) +n} (2n+1) )

ここで 2n(2n+1)^2 < (分母) < (2n+2)(2n+1)^2

分子だけたすと Σ[k=0,2n] k(2n+1-k) = 2n(2n+1)(2n+2) /6

lim[n→∞] (与式) = 1/6,
0072132人目の素数さん
垢版 |
2018/05/06(日) 17:11:12.46ID:fwnVSR3d
A={1-1/n:n∈N}のminAって存在しますよね?
教授が存在しないって言ってたんですけどだれかご教授下さい
0073132人目の素数さん
垢版 |
2018/05/06(日) 17:15:16.81ID:FzJFoTTt
maxの間違えでしょうね
0074132人目の素数さん
垢版 |
2018/05/06(日) 17:27:46.77ID:7UgnNpKw
>>69 うぬ
0075132人目の素数さん
垢版 |
2018/05/06(日) 22:19:11.73ID:KhrVKVJy
>>64

別々に計算すると
 Σ[k=0,2n] {k/(2n+1) + n} = n + (2n+1)n = 2n(n+1),

Σ[k=0,2n] √(nn+k)
 = {n/2 + Σ[k=1,2n] √(nn+k) + (n+1)/2} - 1/2  …… 割線
 < ∫[nn,(n+1)^2] √x dx - 1/2
 = 2n(n+1) + 1/6,
から
(与式) < 1/6,

Σ[k=0,2n] √(nn+k)   …… 接線
 > ∫[nn-1/2,(n+1)^2 -1/2] √x dx
 = ∫[nn,(n+1)^2] √x dx - ∫[(n+1)^2 -1/2,(n+1)^2] √x dx + ∫[nn-1/2,nn] √x dx
 > 2n(n+1) + 2/3 - {n+1 - √(nn-1/2)}/2
 = 2n(n+1) + 1/6 - {n - √(nn-1/2)}/2
 = 2n(n+1) + 1/6 - 1/[4{n + √(nn-1/2)}]
から
(与式) > 1/6 - 1/[4{n + √(nn-1/2)}] → 1/6 (n→∞)
0076132人目の素数さん
垢版 |
2018/05/06(日) 22:32:20.00ID:KhrVKVJy
>>75

∫[nn,(n+1)^2] √x dx = [ (2/3) x^(3/2) ]_{x:nn→(n+1)^2}
 = (2/3) {(n+1)^3 - n^3}
 = (2/3) (3nn +3n +1)
 = 2n(n+1) + 2/3,
を使った。
0078132人目の素数さん
垢版 |
2018/05/06(日) 23:56:29.96ID:GoWoY/5G
もしかして:2進数展開
0079132人目の素数さん
垢版 |
2018/05/07(月) 00:04:02.48ID:h2biOA7U
お願い
(問)
0<a<1として、a1=a, an+1 = 4an*(1-an)  として漸化式で数列anを定義する。
lim[n->∞]an = 0 のとき、aN=0 となる自然数Nが存在することを示せ。
0080132人目の素数さん
垢版 |
2018/05/07(月) 00:23:31.94ID:2KHwpYyw
>>79
定義より
a[n]<1/2 ⇒ a[n+1] = 4a[n](1-a[n]) ≧ 2a[n]
である。lim a[n]=0からn≧N ⇒ a[n]<1/2となるNがとれるが、このときn≧Nに対して
1/2 > a[n] ≧ 2^(n-N)a[N]
により
0≦a[N]<2^(N-n)(1/2)
を得る。n→∞とすればa[N]=0。
0081132人目の素数さん
垢版 |
2018/05/07(月) 03:58:55.21ID:O7EK/B2R
>>79
y=4x(1-x) と y=x の グラフを 0≦x≦1 の範囲で描いて、
初期値aによって、a[n]の値がどのような“動き”をするか調べてみるとよい。
多くの場合は、大きくなったり、小さくなったりと、複雑な動きをすることが判ると思う。

しかし、特徴的な動きもみられる。

あるところでa[k]が3/4近辺の値をとると、次の値も、3/4近辺になる。当然、その次も3/4近辺だ。
3/4を含むある範囲では、常にこのループに陥り、a[k]=0となる様なことはない。
つまり、あるkで、a[k]の値が、この範囲に入るような値を取ると、lim[n->∞]a[n] = 0 となる様なことはない。

また、あるところでa[k]が、1/2 という値を取ると、a[k+1]=1、a[k+2]=0、となり、n≧k+2では常にa[n]=0となる
a[k]が1/2という値を取るためには、a[k-1]が 1/2=4x(1-x) の解 つまり、(2±√2)/4 という値を取っていた場合である。
さらに、(2±√2)/4=4x(1-x) の解を取っていると、.... というように、ある特別な値を取っていた時に限り、
あるところでa[k]=0となり、それ以後、常に0となる。

それ以外の値の場合は、大きくなったり、小さくなったり、あるいは、3/4近辺でぐるぐる回っていたりするだけで、
lim[n->∞]a[n] = 0 等という事は起こらない。このようなことを説明すればよい。
0082132人目の素数さん
垢版 |
2018/05/07(月) 04:24:39.31ID:O7EK/B2R
補足。というか、こちらの方が、本命かもしれない。
多くの場合、
lim[n->∞]a[n] = 0
というのを見ると、a[n]≠0 だけど、 nが大きくなるにつれて、|a[n]|が
どんどん小さくなるような場合を思い浮かべると思うけど、
この問題の場合の lim[n->∞]a[n] = 0 は、そのようなケースではなく、
あるところで、a[k]が0になり、その後は、定義から常に0という場合に限られる。

何故なら、あるところで、a[k]=ε、(ただし、εは非常に小さい値で、正)、を取ったとすると、

a[k+1]=4ε(1-ε)≒4ε=4a[k] となり、前項より大きくなる。

このような性質を持っていては、
“a[n]≠0 だけど、 nが大きくなるにつれて、|a[n]|がどんどん小さくなるような場合”
の、lim[n->∞]a[n] = 0 は起こらない。実際、図を描いても確かめられる。
従って、lim[n->∞]a[n] = 0 というのは、あるところで、a[k]=0 となり、その後全ての項が0
の場合に限られると結論できる。
0083132人目の素数さん
垢版 |
2018/05/07(月) 05:18:54.47ID:N+kxDSb8
xy平面上に点A(1,0)も単位円Cがある。
点Aの、C上の点Pにおける接線lに関する対称点をBとするとき、以下の問に答えよ。

(1)P(cosθ,sinθ)とするとき、Bの座標をθを用いて表せ。

以下、θは0≦θ<2πを動くものとする。

(2)Bの座標を(s,t)とおき、点KをK(s+t,st)により定める。Kが動いてできる曲線Tの式を求めよ。

(3)Tの長さを求めよ。
0084132人目の素数さん
垢版 |
2018/05/07(月) 07:50:42.17ID:P9aQi060
>>48
ds=√(1+(f')^2)dx
dS=f(x)dsdθ
s=s(x)初等関数
ds=s'(x)dx
dS=f(x)s'(x)dxdθ
S(x)初等関数とは限らない
ボクの考えた最強のアホな問題
0085132人目の素数さん
垢版 |
2018/05/07(月) 08:08:08.16ID:Q4lXAlAO
限らないことの証明は
0086132人目の素数さん
垢版 |
2018/05/07(月) 11:24:22.82ID:LPTyY7qu
>>79
 α = arcsin(√a) = arccos(1-2a)/2,
とおくと
 a_n = {sin(2^(n-1)・α)}^2 = {1 - cos(2^n・α)}/2

 α = (奇数)π/{2^(n0-1)} ⇔  a_n = 0 (n≧n0)
0088132人目の素数さん
垢版 |
2018/05/07(月) 12:47:28.38ID:7NDVoze6
2017年早稲田理工の5番ですが
f(x)=x^3+x^2+px+q g(x)=-1/x+1
条件:f(x)=0の任意の解αに対してg(α)もf(x)=0の解である。

当然一つの解はαでもう一つは-1/α+1
模範解答だともう一つはg(g(α))=-α+1/αになります
この後に解が同じか違うかで場合分けという流れですが

はじめの2つの解と解と係数の関係でもう一つの解を出すと-α^2-2α/α+1がでてしまいますが、これはなんでしょうか?任意解を入れれば初めの3つの解のどれかと同じになるということでしょうか??
0089132人目の素数さん
垢版 |
2018/05/07(月) 12:59:11.20ID:LPTyY7qu
>>83

 P-接線: (cosθ)x + (sinθ)y = 1,
 Q (cos(2θ),sin(2θ)) とおくと、P は BQ の中点。
 B (2cosθ-cos(2θ),2sinθ-sin(2θ)) = (s,t)

 s+t = (√2){2sin(θ +π/4) - sin(2θ +π/4)},
 st = (3/2)sin(2θ) - (2-cosθ)sin(3θ) = 2sin(2θ) - 2sin(3θ) + (1/2)sin(4θ),
0090132人目の素数さん
垢版 |
2018/05/07(月) 14:13:35.36ID:gONr+mmN
接線の性質として
半径の中心と異なる端で,半径に垂直な直線は,この円の接線である。

と書いてあるがこれはどういうこと?
半径の中心と異なる端ってことは、円周か円の中心ってこと?
0091132人目の素数さん
垢版 |
2018/05/07(月) 14:22:04.37ID:gONr+mmN
>>90
自己解決した

つまり
半径の中心と異なる端ってことは円周か
円周上で半径に垂直な直線は接線だぜってことか
0092132人目の素数さん
垢版 |
2018/05/07(月) 19:39:07.93ID:pC2LS//C
未だ>>5解けないんだけど。これホントに解けるんかな?
0094132人目の素数さん
垢版 |
2018/05/07(月) 21:05:04.27ID:dkZJD9G+
>>93
いや、それは後で間違い指摘されて実際間違ってる。今のところ正解出てないと思う。でもこのスレ解けない問題上がってくる事もあるからその類かもしれないけど。
0095132人目の素数さん
垢版 |
2018/05/08(火) 00:32:16.79ID:VHCLxHr+
今ふっと思い立ったんだけどもしかして>>23が出題者でその解答が間違ってたのかな?
0096132人目の素数さん
垢版 |
2018/05/08(火) 10:58:07.18ID:mTlWCjoA
f=sin(ax)/axとした時に
甜0,1]fdx / 甜0,1]f^2dx
って求められますか?

教員曰く簡単らしいんですけれど全く無理でした…
0097132人目の素数さん
垢版 |
2018/05/08(火) 11:17:44.07ID:iiV8V26X
たぶん誤解してる
0098132人目の素数さん
垢版 |
2018/05/08(火) 12:20:56.12ID:rSTdfkqz
>>96

∫f(x)dx = (1/a)∫sin(ax)/x dx = (1/a)Si(ax),

∫{f(x)}^2 dx = ∫{sin(ax)/ax}^2 dx
 = - {sin(ax)}^2 /(aax) + (1/a)∫sin(2ax)/x dx
 = - {sin(ax)}^2 /(aax) + (1/a)Si(2ax),

にて簡単
0099132人目の素数さん
垢版 |
2018/05/08(火) 13:35:30.51ID:iiV8V26X
>>98
で?
0101132人目の素数さん
垢版 |
2018/05/08(火) 14:34:06.13ID:hahWjMqy
蒸し蒸し
0102132人目の素数さん
垢版 |
2018/05/08(火) 14:35:14.14ID:ELxTBK1l
なるほど。簡単♡
0103132人目の素数さん
垢版 |
2018/05/08(火) 14:38:13.13ID:G9yXtLP2
云々
0104132人目の素数さん
垢版 |
2018/05/08(火) 15:04:15.29ID:wp3HQiZB
>>98
Si(x)とは?
0106132人目の素数さん
垢版 |
2018/05/08(火) 16:13:37.69ID:hahWjMqy
積分がわからんから姑息な手段を使ったのか
0108132人目の素数さん
垢版 |
2018/05/08(火) 16:33:42.19ID:3em1sf2J
かたつむりか。ターミネーターじゃないのか。
0109132人目の素数さん
垢版 |
2018/05/08(火) 20:12:32.29ID:p9FzWnM6
3次元の座標系を任意に回転させたいのですが、何回回転させればよいか、場合分けすることはできますか?
1回のとき、2回のとき、3回のときがあると思います。
0111132人目の素数さん
垢版 |
2018/05/08(火) 21:47:22.34ID:iiV8V26X
>>109
質問になってない
0112132人目の素数さん
垢版 |
2018/05/08(火) 21:55:54.22ID:MbAKKwrj
SO(3)は3次元
0113132人目の素数さん
垢版 |
2018/05/08(火) 22:09:00.06ID:+3UojrGT
すまん、この問題がわからないんだが・・・・
https://imgur.com/4utNyqX
お願いします(´・ω・`)
0114132人目の素数さん
垢版 |
2018/05/08(火) 22:17:14.40ID:iiV8V26X
xz座標だけθ回転させるだけだよ
結局回転で変わるのは外積だけだが
0115132人目の素数さん
垢版 |
2018/05/08(火) 22:17:58.83ID:p9FzWnM6
109です。
わかりづらくてすみません。
座標系じゃなく3次元の立体図形を回転させるもきに、
0116132人目の素数さん
垢版 |
2018/05/08(火) 22:23:00.28ID:p9FzWnM6
書き間違えました。
少し質問を変えます。
座標系じゃなく3次元の直方体のような立体図形にします。
立体図形を任意に回転させるときに、例えば上下反対にするには1回転させますが、
左右を上下にして裏表反対にするには2回転必要かと思います。
ここで思ったのですが、2次元の場合は1回転で全ての回転を表されるので、3次元の場合は2回転あればすべての回転を表されると思ったのですが、いかがでしょうか。
0117132人目の素数さん
垢版 |
2018/05/08(火) 22:30:46.76ID:MbAKKwrj
>>116
SO(3)は3次元なので3種類の回転を用意せんとダメだっての
0118132人目の素数さん
垢版 |
2018/05/08(火) 22:32:53.46ID:MbAKKwrj
>>116
補足
ただしかける回数に制限がないなら2種類で可能。
0119132人目の素数さん
垢版 |
2018/05/08(火) 22:37:40.37ID:hahWjMqy
おいらに任せろ
0120132人目の素数さん
垢版 |
2018/05/08(火) 22:49:41.27ID:p9FzWnM6
116です。
回答ありがとうございます。
3種類の回転が必要ですか。SO(3)という数式を初めて見ました。
回転の軸を3次元中に自由に取れるとしても必ず3種類の回転が必要になりますでしょうか。
0121132人目の素数さん
垢版 |
2018/05/08(火) 23:01:33.14ID:MbAKKwrj
>>120
なるよ。もしA(x)B(y) (x,yはパラメータ)でSO(3)全体をカバーできたらそれは2次元空間から3次元空間への全射を与えてしまうけどそれは無理。
0123132人目の素数さん
垢版 |
2018/05/08(火) 23:08:55.52ID:p9FzWnM6
>>121
なるほど、なんとなくわかりました。
>>116で書いたとおり、1回転とか2回転で達成できるような場合もあると思うのですが、そういった特殊な例はどういった場合とかって簡潔に言えますか?
0124132人目の素数さん
垢版 |
2018/05/08(火) 23:10:34.09ID:p9FzWnM6
>>122
ありがとうございます。オイラー角についてググってみます。
0125132人目の素数さん
垢版 |
2018/05/08(火) 23:26:31.89ID:q2L0o+ot
>>123
さあ?そこまでいくとわかんない。でも2個の回転行列A(x)、B(y)を指定すれば
A(x)B(y)の形の回転の全体は実2次元部分空間になるからシンプルな表現はあるとは思うけど。
0126132人目の素数さん
垢版 |
2018/05/08(火) 23:31:33.29ID:p9FzWnM6
>>125
ありがとうございます。教えていただいたのをもとに自分なりに勉強してみます。
0127132人目の素数さん
垢版 |
2018/05/08(火) 23:53:08.87ID:DjWjgehL
>>96
どうやらこの積分、分子分母にx^2を掛けても結果が変わらないそうです
どうやってそのことを導出したのでしょうか…?
0128132人目の素数さん
垢版 |
2018/05/09(水) 00:09:52.11ID:6yVxW1Wf
>>127
分子分母に同じ数かけたらそら同じ数になるのでは?
0129132人目の素数さん
垢版 |
2018/05/09(水) 00:35:56.85ID:V+z5IWFo
>>127
分子分母の被積分関数にx^2かけるの?全然違う値になるけど?問題間違ってない?

f(x):=sin(a*x)/(a*x);
romberg(f(x),x,0.0001,1)/romberg(f(x)^2,x,0.001,1),a:1,numer;
romberg(f(x)*x^2,x,0.0001,%pi)/romberg(f(x)^2*x^2,x,0.001,%pi),a:1,numer;
romberg(f(x),x,0.0001,1)/romberg(f(x)^2,x,0.001,1),a:2,numer;
romberg(f(x)*x^2,x,0.0001,1)/romberg(f(x)^2*x^2,x,0.001,1),a:2,numer;
romberg(f(x),x,0.0001,1)/romberg(f(x)^2,x,0.001,1),a:3,numer;
romberg(f(x)*x^2,x,0.0001,1)/romberg(f(x)^2*x^2,x,0.001,1),a:3,numer;
(%o69) 1.055384728709074
(%o70) 2.000000016713958
(%o71) 1.195429148161006
(%o72) 1.464505695121646
(%o73) 1.306210378186181
(%o74) 1.981775232275942
0130132人目の素数さん
垢版 |
2018/05/09(水) 00:49:21.65ID:hO1Ej9LV
ミスってたのでやりなおし。やっぱり同じ値にはならない??

>>127
f(x):=sin(a*x)/(a*x);
romberg(f(x) ,x,0.00001,1)/romberg(f(x)^2 ,x,0.00001,1),a:1,numer;
romberg(f(x)*x^2,x,0.00001,1)/romberg(f(x)^2*x^2,x,0.00001,1),a:1,numer;
romberg(f(x) ,x,0.00001,1)/romberg(f(x)^2 ,x,0.00001,1),a:2,numer;
romberg(f(x)*x^2,x,0.00001,1)/romberg(f(x)^2*x^2,x,0.00001,1),a:2,numer;
romberg(f(x) ,x,0.00001,1)/romberg(f(x)^2 ,x,0.00001,1),a:3,numer;
romberg(f(x)*x^2,x,0.00001,1)/romberg(f(x)^2*x^2,x,0.00001,1),a:3,numer;

(%o102) 1.054320648435071
(%o103) 1.104493895983902
(%o104) 1.193802891132201
(%o105) 1.4645056917975
(%o106) 1.303664967479121
(%o107) 1.981775220915906
0131132人目の素数さん
垢版 |
2018/05/09(水) 02:22:57.05ID:cq9H9+pa
>>127
答えとしては
4(sin(a)-acos(a))/(2a-sin(2a))
になるらしいです…
0132132人目の素数さん
垢版 |
2018/05/09(水) 03:04:15.07ID:bPbSE1pH
>>131
そんなはずはない。もとめる値をI(a)とおくと
lim[a→∞]I(a) = 1になるけど、その値振動するやん。
0133132人目の素数さん
垢版 |
2018/05/09(水) 04:02:36.55ID:6xr7P3xx
f+gが微分可能でfが微分可能でない例
fgが微分可能でfが微分可能でない例
f・gが微分可能でfまたはgが微分可能でない例
を教えて下さい
0134132人目の素数さん
垢版 |
2018/05/09(水) 06:42:48.91ID:OgcCeOxy
nCrで選ぶ個数に文字rを使う理由って何?
0135132人目の素数さん
垢版 |
2018/05/09(水) 07:40:18.48ID:ymGjZh6L
>>133
f:ディリクレ関数、g=-fでf+g=0は微分可能
f:ディリクレ関数、g=0でfg=0は微分可能
f=0、g:ディリクレ関数でf・g=0は微分可能
0136132人目の素数さん
垢版 |
2018/05/09(水) 10:05:29.42ID:8E+qLwNX
どのような楕円であっても、その周長(一周分)を求めることはできますか?
0137132人目の素数さん
垢版 |
2018/05/09(水) 10:14:51.45ID:OgcCeOxy
はい
0138132人目の素数さん
垢版 |
2018/05/09(水) 11:07:56.72ID:k89ZCTKX
鉛筆で楕円を描いて、直線を描いた場合との比較で、
鉛筆の重さの減少量と長さとの関係から計算する
0139132人目の素数さん
垢版 |
2018/05/09(水) 11:41:54.29ID:72cH2LZd
fが区間Iで微分可能関数であるとき
任意のx,y∈Iに対して|f(x)-f(y)|≦K|x-y| (Kは定数)が成立するならば
任意のx∈Iに対し、|f'(x)|≦Kが成立することを証明して下さい
極限取ることは分かってるのですが、x=yの場合や
|lim[x→y](f(x)-f(y))/(x-y)|=lim[x→y]|(f(x)-f(y))/(x-y)|が示せなく困っています
0140132人目の素数さん
垢版 |
2018/05/09(水) 11:58:46.91ID:XOhlVUhY
>>127 >>131
なんじゃそりゃ?

∫ p(x)dx ∫ x^2p(x)dx
―――― = ――――――
∫q(x)dx ∫x^2q(x)dx

ってやっちゃったのか? ’∫’ 無視して?恐ろしいな。
0141132人目の素数さん
垢版 |
2018/05/09(水) 12:42:29.07ID:GEUtkttz
>>136 >>137

(x/a)^2 + (y/b)^2 = 1, (0<b≦a)

とする。

L = 4∫[0,π/2] √{(a・cosθ)^2 + (b・sinθ)^2} dθ

= 4a ∫[0,π/2] √{ 1 - (k・sinθ)^2} dθ     k = √{1 - (b/a)^2} 離心率

= 4a E(k)

第二種の完全楕円積分、θはパラメータ
0143132人目の素数さん
垢版 |
2018/05/09(水) 13:53:41.98ID:k89ZCTKX
>>141
楕円なんたらとかかってな名前をつけて誤魔化さないで、
ちゃんと積分してください。
0145132人目の素数さん
垢版 |
2018/05/09(水) 14:34:58.63ID:8E+qLwNX
>>141
私は高校生です
√1-t^2 はt=sinθと置けと言われました
ksinθ=aと置いたらどうですか
0146132人目の素数さん
垢版 |
2018/05/09(水) 14:38:18.51ID:vfY0Zsg+
>>145
楕円の周を求めるのは難しくて、簡単な式で表すことができないということが知られています
つまり、解けません
0150132人目の素数さん
垢版 |
2018/05/09(水) 15:03:33.73ID:k89ZCTKX
>>146
難しのと解けないのとは違うよね。
難しいからごまかしてるんでしょ?
0152132人目の素数さん
垢版 |
2018/05/09(水) 15:32:05.35ID:vfY0Zsg+
>>150
>>151

>>147みてみてくださいね
初等関数で表せないとあるはずです
初等関数というのは、簡単な式ということです
0154132人目の素数さん
垢版 |
2018/05/09(水) 16:26:06.35ID:8E+qLwNX
楕円積分の解法を思いつきました
級数展開してから積分すれば良いですね
多項式なら簡単に積分できますので
0156132人目の素数さん
垢版 |
2018/05/09(水) 16:41:53.59ID:KCh8zfY4
何で性別女って設定なの?
0157132人目の素数さん
垢版 |
2018/05/09(水) 16:46:14.40ID:PWrBl1fd
f(xy)kg(xy)=0ってなんなんなんですか?
fとgの交点の座標を求めてkをだすのも=0ってのもなんなんなんですか?
0158132人目の素数さん
垢版 |
2018/05/09(水) 16:47:58.92ID:wgZFPRFj
昔、関数の連続性を高校の範囲で証明できると主張した馬鹿がいたが
0159132人目の素数さん
垢版 |
2018/05/09(水) 17:28:42.17ID:JB8c0kqr
>>157
f(x,y)=0の曲線とg(x,y)=0の曲線があった時、f(x,y)+kg(x,y)=0の曲線は、f(x,y)=0とg(x,y)=0の全ての交点を通るということです
0160132人目の素数さん
垢版 |
2018/05/09(水) 17:52:26.77ID:FkjzUCdK
>>96=>>136
0161132人目の素数さん
垢版 |
2018/05/09(水) 18:42:01.73ID:PWrBl1fd
>>159
A+KB=0でなぜB以外の全ての直線を表せるのだろ
0162132人目の素数さん
垢版 |
2018/05/09(水) 18:47:40.43ID:JB8c0kqr
kが変われば傾きも変わりますね
0164132人目の素数さん
垢版 |
2018/05/09(水) 18:57:56.95ID:PWrBl1fd
k(ax+bx+c)すると傾き以外も変わるのじゃ?
0165132人目の素数さん
垢版 |
2018/05/09(水) 19:03:41.56ID:PWrBl1fd
K倍しても移項してyの係数割る時元に戻る気がする
0166132人目の素数さん
垢版 |
2018/05/09(水) 20:54:50.50ID:gnIjlof6
>>161
KA+LB=0のがいい
0168132人目の素数さん
垢版 |
2018/05/09(水) 23:46:01.38ID:gnIjlof6
>>167
>・円、放物線の長さは初等函数(√とlog)で表わせる。
放物線が?
0169132人目の素数さん
垢版 |
2018/05/09(水) 23:58:44.81ID:GEUtkttz
>>168

y = ax^2 とすると、

L(0,x) = ∫[0,x] √{1+(2ax ')^2} dx '

 = (1/2)x√{1+(2ax)^2} + (1/4a)log{2ax + √{1+(2ax)^2}]
0170132人目の素数さん
垢版 |
2018/05/10(木) 07:10:25.01ID:lMbFJOYg
【問題】
10から110までの数字の組み合わせの和で
10から110までのすべての整数を表現したいとき
用意しなければならない数字の最小の個数とその数字を述べよ。




お願いします。
0171170
垢版 |
2018/05/10(木) 07:11:57.01ID:lMbFJOYg
>>170
なお、組み合わせは2個でも3個でも、何個でも組み合わせて良い。
0173132人目の素数さん
垢版 |
2018/05/10(木) 10:31:25.24ID:SsvtRexk
>>170
10+10=20
はあり?
0174132人目の素数さん
垢版 |
2018/05/10(木) 10:59:04.99ID:lprgN9Zl
>>170

10+10 = 20がありなら
{10,11,12,13,14,15,16,17,18,19} の10個
無しなら
{10,11,12,13,14,15,16,17,18,19,20} の11個
0175IQの低い人
垢版 |
2018/05/10(木) 13:29:02.37ID:sToklkep
1,0でいいんじゃないの
0177132人目の素数さん
垢版 |
2018/05/10(木) 13:41:51.82ID:/57cBKqk
175,176は、10が組み合わせ可能な数字の最小値であるという仮定を無視してしまったようだ
0178132人目の素数さん
垢版 |
2018/05/10(木) 14:47:07.34ID:NEWFwW7D
>>177
> 175,176は、10が組み合わせ可能な数字の最小値であるという仮定を無視してしまったようだ
数字って0123456789のことよ
0179132人目の素数さん
垢版 |
2018/05/10(木) 16:25:13.36ID:9tRstjmn
素数pを1つとり、p^nを3で割った余りをanとする。{an}の一般項を求めよ。
0180132人目の素数さん
垢版 |
2018/05/10(木) 16:47:11.20ID:swO+9t/h
>>166
へー
0181132人目の素数さん
垢版 |
2018/05/10(木) 16:48:52.87ID:swO+9t/h
>>161
高校数学では説明できないんでしょうか
0183132人目の素数さん
垢版 |
2018/05/10(木) 16:57:21.09ID:swO+9t/h
>>182
微分積分勉強し直して出直してきます。
とりま頭に入れて問題とばそうかな
0184132人目の素数さん
垢版 |
2018/05/10(木) 17:09:16.92ID:IRATbn/s
>>181
f(x,y)=0 と g(x,y)=0の交点は、f(x,y)=0 かつ g(x,y)=0を満たす点(x0,y0), (x1,y1), ... であり
f(x0,y0)=0、g(x0,y0)=0 などを満たす。

よって、f(x0,y0) + kg(x0,y0) = 0 などとなるので、f(x,y)+kg(x,y)=0 は、必ず f(x,y)=0 , g(x,y)=0の
全ての交点を通る。
0185132人目の素数さん
垢版 |
2018/05/10(木) 19:21:06.15ID:9tRstjmn
a,bは0でない実数とする。
双曲線(x^2/a^2)-(y^2/b^2)=1の、t≦x≦t+kの区間の長さをL(t,k)とする。

(1)L(t,k)が定義できるようなtの範囲を定めよ。

(2)tは(1)の範囲にあり、また正とする。lim[k→∞] {L(t,k)/(αk)} = 1となるαが存在することを示せ。
0186132人目の素数さん
垢版 |
2018/05/10(木) 20:08:10.67ID:9tRstjmn
2^n+1(nは2以上の自然数)の形で表される自然数は、いくつかの相異なる素数の和で表せることを示せ。
0187132人目の素数さん
垢版 |
2018/05/10(木) 20:55:54.68ID:R9xe/nJK
>>180
KA+(1-K)B=0
0188132人目の素数さん
垢版 |
2018/05/11(金) 00:38:27.38ID:SdgBfY6R
>>179

 p≡1 (mod 3) のとき、a_n = 1
 p≡2 (mod 3) のとき、a_n = 2 (n:奇数)、a_n = 1(n:偶数)
 p=3 のとき、a_n = 0

>>185

(1) 任意の実数

(2)
 y = ±(b/a)√(aa-xx) (両分枝の合併)
 漸近線 y = ±(b/a)x
 α = (2/a)√(aa+bb)
0189132人目の素数さん
垢版 |
2018/05/11(金) 01:45:36.95ID:hWM/gHo5
>>186
7以上の整数nは相異なる2個以上の素数の和で表せる。
∵n≦23では以下のように正しい。
7 = 2+5, 8=3+5, 9=2+7, 10=3+7,11=2+9,12=5+7,13=2+11,
14=3+11,15=2+13,16=5+11, 17=2+3+5+7, 18=7+11, 19=2+17,
20=3+17, 21=2+19, 22=3+19, 23=3+7+13
24以上の整数Nについてn<Nで成立するとしてn=Nとする。
x以下の素数の数をπ(x)とおくと
x≧17に対しx/logx<π(x)
x≧1に対し<1.25506x/log(x)
https://ja.wikipedia.org/wiki/%E7%B4%A0%E6%95%B0%E8%A8%88%E6%95%B0%E9%96%A2%E6%95%B0
また容易にx≧11に対し
(x-7)/log(x-7)-1.25506(x/2)/log(x/2)>0
よってx≧24に対しπ(x-7)-π(x/2)>0。よってn/2<p<n-7を満たす素数が存在する。
帰納法の仮定より素数の集合PでΣ[q∈P]q = n-p<n/2となる。
またPの要素はすべてp未満である。よってn = p + Σ[q∈P]qとなりn=Nのときも正しいとわかった。
特にn≧3にたいし2^n+1は相異なる2個以上の素数の和で表せる。
また2^2+1=2+3。
0190132人目の素数さん
垢版 |
2018/05/11(金) 01:48:38.09ID:BJQnbXOk
>>189
訂正。14行目。
×:n/2<p<n-7
○:n/2<p≦n-7
0191132人目の素数さん
垢版 |
2018/05/11(金) 02:01:18.43ID:dn9lA8yy
計算ミスったorz。やり直し。

>>186
7以上の整数nは相異なる2個以上の素数の和で表せる。
∵n≦24では以下のように正しい。
7 = 2+5, 8=3+5, 9=2+7, 10=3+7,11=2+9,12=5+7,13=2+11,
14=3+11,15=2+13,16=5+11, 17=2+3+5+7, 18=7+11, 19=2+17,
20=3+17, 21=2+19, 22=3+19, 23=3+7+13, 24=5+19
25以上の整数Nについてn<Nで成立するとしてn=Nとする。
x以下の素数の数をπ(x)とおくと
x≧17に対しx/logx<π(x)
x≧1に対し<1.25506x/log(x)
https://ja.wikipedia.org/wiki/%E7%B4%A0%E6%95%B0%E8%A8%88%E6%95%B0%E9%96%A2%E6%95%B0
また容易にx≧25に対し
(x-7)/log(x-7)-1.25506(x/2)/log(x/2)>0
よってx≧25に対しπ(x-7)-π(x/2)>0。よってn/2<p<n-7を満たす素数が存在する。
帰納法の仮定より素数の集合PでΣ[q∈P]q = n-p<n/2となる。
またPの要素はすべてp未満である。よってn = p + Σ[q∈P]qとなりn=Nのときも正しいとわかった。
特にn≧3にたいし2^n+1は相異なる2個以上の素数の和で表せる。
また2^2+1=2+3。
0192132人目の素数さん
垢版 |
2018/05/11(金) 03:14:45.17ID:GCL6tPR6
スレ汚しすまんorz。再挑戦

12以上の整数nは相異なる2個以上の素数の和で表せる。
∵n≦39では以下のように正しい。

12=5+7,13=2+11,14=3+11,15=2+13,16=5+11, 17=2+3+5+7, 18=7+11, 19=2+17,
20=3+17, 21=2+19, 22=3+19, 23=3+7+13, 24=5+19, 25=2+23, 26=3+23, 27=3+5+19,
28=5+23, 29=3+7+19, 30=7+23, 31=2+29, 32=3+29, 33=2+31, 34=3+31, 35=5+7+23,
36=5+31, 37=3+5+29, 38=7+31, 39=2+37。

40以上の整数Nについてn<Nで成立するとしてn=Nとする。
x以下の素数の数をπ(x)とおくと
x≧17に対しx/logx<π(x)
x≧1に対し<1.25506x/log(x)
https://ja.wikipedia.org/wiki/%E7%B4%A0%E6%95%B0%E8%A8%88%E6%95%B0%E9%96%A2%E6%95%B0
また容易にx≧40に対し
(x-12)/log(x-12)-1.25506(x/2)/log(x/2)>0
よってx≧40に対しπ(x-12)-π(x/2)>0。よってn/2<p≦n-12を満たす素数が存在する。
帰納法の仮定より素数の集合PでΣ[q∈P]q = n-p<n/2となる。
またPの要素はすべてp未満である。よってn = p + Σ[q∈P]qとなりn=Nのときも正しいとわかった。
0194132人目の素数さん
垢版 |
2018/05/11(金) 03:50:10.98ID:AP8YngWJ
>>193
そのなかのどれかつかってもっとうまく示せる?
“異なる”と”2個以上”という縛りがあるからサラッと処理する方法思いつかんかったんだけど?
0195132人目の素数さん
垢版 |
2018/05/11(金) 03:59:37.57ID:0HlT8DZW
>>161
>A+KB=0でなぜB以外の全ての直線を表せるのだろ
A、Bが直線の式(2x+3y-5とか)を示しているんだったら、全ての直線を表すことはできないぞ

A+KB=0が表すのは、A=0とB=0の交点を通る直線のうちで、B以外のすべての直線だ。
0196132人目の素数さん
垢版 |
2018/05/11(金) 07:45:25.94ID:j9aOBqih
>>191
>またPの要素はすべてp未満である。
0197132人目の素数さん
垢版 |
2018/05/11(金) 07:46:45.87ID:j9aOBqih
>>195
>A+KB=0が表すのは、A=0とB=0の交点を通る直線のうちで、B以外のすべての直線だ。
KA+(1-K)B=0が表すのは、A=0とB=0の交点を通るすべての直線だ。
0198132人目の素数さん
垢版 |
2018/05/11(金) 08:30:10.47ID:lqFhgDwc
極方程式r=e^(-θ)で表される曲線の、y≧0の部分をCとする。またC上のy座標が0である点のうち、(1,0)でないものをBとする。
xy平面上を速さ1で動く点Pは、点A(2,0)から出発してCの外部かつy>0の領域を通り、C上の点Sに到達して、そこからC上を通って点Bに至る。
点Pが最も早く点Bに至るように、点Sの位置を定めよ。
0199132人目の素数さん
垢版 |
2018/05/11(金) 09:22:02.18ID:mYebBGTa
>>196
n/2<p≦n-12
0201132人目の素数さん
垢版 |
2018/05/11(金) 13:48:25.25ID:FtejLL1m
>>179
mod 3で
p≡1のときa_n=1
p≡2のときa_n=(3-(-1)^n))/2
p=3のときa_n=0
0202132人目の素数さん
垢版 |
2018/05/11(金) 14:22:49.94ID:FtejLL1m
>>157
f(a,b)=0, g(a,b)=0を満たす点(a,b)が存在するとする。
f(a,b)=0を満たす(a,b)はもちろんグラフf(x,y)=0上の点。
g(a,b)=0を満たす(a,b)はもちろんグラフg(x,y)=0上の点。
つまり(a,b)はf(x,y)=0, g(x,y)=0の交点である。

さて、実数α,β(いずれかは0でないとする)について、h(x,y)=αf(x,y)+βg(x,y)=0で表されるグラフを考える。
αf(a,b)+βg(a,b)=α*0+β*0=0より、(a,b)は(α,βによらず)グラフαf(x,y)+βg(x,y)=0上の点である。
もちろん、h(x,y)=0が(a,b)を通るあらゆる線を表すわけではない。

α≠0のとき、h(x,y)=0⇔f(x,y)+(β/α)g(x,y)=0⇔f(x,y)+kg(x,y)=0
α=0のときβ≠0で、h(x,y)=0⇔βg(x,y)=0⇔g(x,y)=0
よって、h(x,y)=0は次の2式として表せる。
f(x,y)+kg(x,y)=0
g(x,y)=0

(a,b)が複数存在する(つまりf(x,y)=0, g(x,y)=0の交点が2つ以上ある)場合も、全ての交点について上記の議論が成り立つ。
つまり、全ての交点はf(x,y)+kg(x,y)=0やg(x,y)=0上の点である。
0205132人目の素数さん
垢版 |
2018/05/11(金) 16:11:28.11ID:MgeDrhnB
接戦引けばいいだけじゃないの?
0206132人目の素数さん
垢版 |
2018/05/11(金) 16:16:37.23ID:sUzPr1Ik
>>205
接線引いてC上通るのが最短経路?
多分そうかも知れないけど微分使わず説明できる?
0207132人目の素数さん
垢版 |
2018/05/11(金) 16:17:21.46ID:MgeDrhnB
変分原理でも使うか?
0208132人目の素数さん
垢版 |
2018/05/11(金) 16:23:25.85ID:MgeDrhnB
x軸とのなす角θとして道のりをθで表して微分すればいいだけだよ。
0209132人目の素数さん
垢版 |
2018/05/11(金) 16:36:59.32ID:sUzPr1Ik
互いに素な自然数a,bと3以上の素数pに対して、(a+bi)^pは実数でないことを示せ。
ただしiは虚数単位である。
0210132人目の素数さん
垢版 |
2018/05/11(金) 16:45:53.09ID:0yS46dEA
>>206
微分つかわんのはしんどいやろ。
長さの合計をLとして
dl = 接線方向への単位ベクトル + A方向への単位ベクトル
これが法線ベクトルと平行になる点が答え。つまりA方向へのベクトルが接線と平行のとき。
0212132人目の素数さん
垢版 |
2018/05/11(金) 16:52:18.28ID:MgeDrhnB
にこう定理で解けるやろ
0213132人目の素数さん
垢版 |
2018/05/11(金) 17:04:40.86ID:0yS46dEA
>>209
α=a+bi, θ=arctan(b/a)とおいてmθ∈πNと仮定する。
とくにmとしてこれを満たす最小の自然数をとる。
exp(iθ) ∈ Q(α,√(a^2+b^2))により
φ(m)=[Q(exp(iθ))]≦[Q(α,√(a^2+b^2)):Q]≦4。
∴φ(m)≦4
∴m = 1,2,3,4,5,6,8,12
このなかでtan(θ)=b/aが有理数であるのはm=4のみ。
とくに(a+bi)^pが実数となるのはpが4の倍数のときのみ。
0215132人目の素数さん
垢版 |
2018/05/11(金) 17:12:13.21ID:yjsU5oy7
複素数平面上で、右の図のように、異なる3点O(0)、A(α)、B(β)を頂点とする△OABの外側に、辺OA、辺OBをそれぞれ斜辺とする直角二等辺三角形OAC、OBDを作る。このとき、辺ABの中点をMとするとCM=DM、CM⊥DMであることを複素数を用いず証明せよ。
https://imgur.com/a/oKyaclT
写真貼れてるかどうか心配ですがよろしくお願いします。
0217132人目の素数さん
垢版 |
2018/05/11(金) 17:33:32.98ID:MgeDrhnB
>>209
0<k<p では  pCk = 0 mod p より
 (a+bi)^p = a^p +b^p i^p mod p
p>2 より、pは奇数。よって、(a+bi)^pは実数ではない。
0218132人目の素数さん
垢版 |
2018/05/11(金) 17:45:10.62ID:c/H/a+yG
井山裕太氏が囲碁の世界には進まず、普通に勉強してたら、東大理Vに首席で合格できたと思いますか?
0219132人目の素数さん
垢版 |
2018/05/11(金) 17:52:12.43ID:MgeDrhnB
>>217 これはダメだね。
0220132人目の素数さん
垢版 |
2018/05/11(金) 18:35:42.29ID:/24RCwpQ
>>174
さんきゅー
仕事がはかどります!

あなたの回答は我が国のGDPに
これから貢献することになるでしょう。
0222132人目の素数さん
垢版 |
2018/05/11(金) 19:11:01.74ID:fMewDKvN
>>215できたけど全部文字に起こす気にならん。図って偉大だなぁ
0223132人目の素数さん
垢版 |
2018/05/11(金) 19:13:51.64ID:zUp9NIGk
問題
数直線全体で定義される何回でも微分できる関数f(x)で次の条件を満たすものを見つけよ:
ある1でない正定数αと多項式Pが存在して、数直線全体でf(αx)=P(f(x))を満たす。
例. cos(3x)=4cos^3(x)-3cos(x).
0225132人目の素数さん
垢版 |
2018/05/11(金) 20:11:52.89ID:sSFRGylu
>>222
全部文字に起こすのが面倒くさいなら方針だけでも教えていただけますか。
0226132人目の素数さん
垢版 |
2018/05/11(金) 20:57:57.47ID:2MTeB4Yq
中3の中間試験の因数分解お願いします

8a^2b-2a+4a^2c-c


…問題間違えてないですか?これ
0228132人目の素数さん
垢版 |
2018/05/11(金) 22:09:16.40ID:fMewDKvN
>>225
O中心にCDMを2倍に拡大した点をC’D’M’、ベクトルOX=ベクトルD’M’となるようにXをとると△AM’C’が△AXOをA中心に90度回したものになる
0233DJgensei artchive gemmar
垢版 |
2018/05/12(土) 11:57:24.23ID:pKtCKnP+
因数分解って、稼働区域のパターンのことだから、動きを重点にしてね。
0234132人目の素数さん
垢版 |
2018/05/12(土) 20:05:54.07ID:SCW0csPu
f(x)=x(1/x)の定義域を述べ、xを限りなく0 に近づけたときのf(x)の挙動を調べよ。
0235132人目の素数さん
垢版 |
2018/05/12(土) 20:08:56.71ID:oLpBza7h
>>234
ばかすか?
0236132人目の素数さん
垢版 |
2018/05/12(土) 20:15:42.80ID:PmaOoNNr
高校数学で f(x)=√(1-x^2) の定義域を求めよとかいうふざけた問題あるよね
0237132人目の素数さん
垢版 |
2018/05/12(土) 21:04:16.27ID:oLpBza7h
>>236
普通の問題
0238132人目の素数さん
垢版 |
2018/05/12(土) 21:09:06.91ID:LdhTEb90
複素平面全体かな?
0240132人目の素数さん
垢版 |
2018/05/12(土) 21:16:42.87ID:1opnZLPA
>>239
ペアノ算術を含む任意の無矛盾な公理系に対し、あるモデルM,Nおよび論理式φが存在して、M|=φかつN|≠φとできることを示せ、という問題がわかりません

よろしくお願いします
0241132人目の素数さん
垢版 |
2018/05/12(土) 21:21:44.36ID:1opnZLPA
>>239
本当にわからないので、なるべく早く回答いただけるとありがたいですね
0242132人目の素数さん
垢版 |
2018/05/12(土) 21:36:57.99ID:oLpBza7h
>>241
分からないんですねw
0243132人目の素数さん
垢版 |
2018/05/12(土) 21:38:35.79ID:1opnZLPA
回答がありませんね


まさかとは思いますが、わからないんですか?
0246132人目の素数さん
垢版 |
2018/05/12(土) 21:58:04.16ID:b2J32MDf
閉リーマン面、C上1変数代数関数体、C上非特異射影代数曲線にはそれぞれ対応がある(反変圏同値?)らしいですが、
コンパクトn次元複素多様体、C上n変数代数関数体、C上非特異n次射影代数多様体も同様の対応が成り立つのでしょうか?
0247132人目の素数さん
垢版 |
2018/05/12(土) 22:32:03.07ID:eiK+3dSt
>>246
関数体からC上非特異n次射影代数多様体へは広中の定理でいけそうだけどコンパクトn次元複素多様体へいけるかなぁ?
畑違いなので自身ないけど。
たしか代数的に限ってもプロジェクティブでないのが作れるって聞いたことあるからダメな気がする。
ハーツホーンの練習問題にあった記憶が……
0248132人目の素数さん
垢版 |
2018/05/12(土) 22:34:18.13ID:SCW0csPu
空間の円C:x^2+y^2=1,z=0を底面とし、点(0,0,1)を頂点とする円錐面のうち、y≧0の部分をKとする。
また、Cのy≦0の部分をC'とする。
C'上を点Pが動くとき、A(0,1,tan75°)とPを結ぶ直線がKと交わる点Qの軌跡を図示せよ。
ただしKの側面の展開図上に図示すること。
0249132人目の素数さん
垢版 |
2018/05/12(土) 22:36:39.66ID:ERQyPxVg
>>212 >>217

 (a + bi)^p = {a^p - pC2・a^(p-2)・b^2 + ……} + {pC1 a^(p-1)・b - pC3・a^(p-3)・b^3 + ……}i

 bがpの倍数ならば、aはpの倍数ではない。
b = B・p^m となる最大の自然数をmとする。
 
虚数部は
 p・a^(p-1) B・p^m - p(p-1)(p-2)/6・a^(p-3)・B^3・p^(3m) + …… ≡ a^(p-1)・B・p^(m+1)
 ≠ 0  (mod p^(3m+1))
よって、(a+bi)^p は実数ではない。
0251132人目の素数さん
垢版 |
2018/05/12(土) 22:49:33.56ID:1opnZLPA
>>248
ある無矛盾な公理系τの任意のモデルに対してある論理式φが常に真となるならば、τからφがLKにおいて証明可能となることを示せ、という問題がわかりません
0252132人目の素数さん
垢版 |
2018/05/12(土) 22:59:22.56ID:rulsrvzO
Σ[n=-∞,∞]1/((ni-a)(ni-b))

を求めてください
iは虚数単位で、a,bは実数で、a≠bです
aもbも0出ない時と、どちらかが0の時の場合の両方を求めてくださるとありがたいです
0254132人目の素数さん
垢版 |
2018/05/12(土) 23:38:12.32ID:xLUYWUsP
>>232
因数分解されるとしたら
(aについて0次)(aについて3次) または (aについて1次)(aについて2次)
となるしかない。

前者の場合
 f(b,c)*(g(b,c)a^3+(aについて2次以下))
という形だが、a^3 の係数を考えると f(b,c) が定数でなければならず不適。

後者の場合
上と同様に a^3 の係数について考えれば
 (a+f(b,c))(a^2+g(b,c)a+h(b,c))
の形に分解できるはず。
定数項を比較して
 f(b,c)h(b,c)=b^3+c^3-4nbc

ここで b^3+c^3-4nbc の因数分解を考える。
もし因数分解されるとしたら、b^3 の係数が 1 なので、上と同様に
(b+F(c))(b^2+G(c)b+H(c))
の形に分解できるはず。
定数項を比較して
 F(c)H(c)=c^3
よって、F(c) は実数 k を用いて k, kc, kc^2, kc^3 のいずれかの形。
このとき、b^3+c^3-4nbc に b=-F(c) を代入すれば (c によらず) 0 になるはずだが、
得られた F(c) の候補のどれを代入してもそうならないので矛盾。
したがって b^3+c^3-4nbc は既約。

f(b,c)h(b,c)=b^3+c^3-4nbc より、f(b,c) は定数または k(b^3+c^3-4nbc) の形。
このとき、a^3+b^3+c^3-4n(ab+bc+ca)+abc に a=-f(b,c) を代入すれば (b,c によらず) 0 になるはずだが、
得られた f(b,c) の候補のどれを代入してもそうならないので矛盾。
したがって a^3+b^3+c^3-4n(ab+bc+ca)+abc は既約。

本当に高校生なのかどうかは知らないけど、あまり大人を試すような真似をするんじゃありませんよ
0256132人目の素数さん
垢版 |
2018/05/12(土) 23:44:02.93ID:ERQyPxVg
>>252
π{coth(πb) - coth(πa)}/(a-b)

(解)
1/{(a-n・i)(b-n・i)} = {1/(b-n・i) - 1/(a-n・i)}/(a-b),
1/{(a+n・i)(b+n・i)} = {1/(b+n・i) - 1/(a+n・i)}/(a-b),
辺々足したのち、次を使う。
Σ[n=1,∞] {1/(a-n・i) + 1/(a+n・i)} + 1/a
= Σ[n=1,∞] 2a/{(a-n・i)(a+n・i)} + 1/a
= π coth(πa),

a→0 のときは、n=0の項が発散しそうな希ガス…
0257132人目の素数さん
垢版 |
2018/05/12(土) 23:59:07.49ID:AcZ8M7Y5
>>252
どっちも0でないときは
Σ[n=-∞,∞]1/((ni-a)(ni-b))
=1/(ai-bi)Σ[n=-∞,∞](1/(n+ai)-1/(n+bi))
=1/(ai-bi)Σ[n=1,∞](1/(n+ai)+1/(-n+ai))
-1/(ai-bi)Σ[n=1,∞](1/(n+bi)+1/(-n+bi)) + 第0項
=-π/(ai-bi)cotπai + π/(ai-bi)cotπbi - 1/(ai-bi)(1/ai-1/bi) + 第0項
https://ja.wikipedia.org/wiki/%E4%B8%89%E8%A7%92%E9%96%A2%E6%95%B0%E3%81%AE%E9%83%A8%E5%88%86%E5%88%86%E6%95%B0%E5%B1%95%E9%96%8B
どっちか0のときは第0項が計算不能。
0258132人目の素数さん
垢版 |
2018/05/13(日) 00:00:21.16ID:iNO2x29a
かぶったorz
0259132人目の素数さん
垢版 |
2018/05/13(日) 01:58:35.15ID:mSJNHrCr
>>250
(001)からの距離を与えたら?
0260132人目の素数さん
垢版 |
2018/05/13(日) 01:58:52.74ID:mSJNHrCr
>>251
恥を知らないのですねw
0261132人目の素数さん
垢版 |
2018/05/13(日) 02:09:45.41ID:I2s0crdy
>>256
それもそうでした!
ありがとうございます!
0262132人目の素数さん
垢版 |
2018/05/13(日) 02:52:46.02ID:cuL3jHY3
>>259
試しにやってみたらこれ問題間違ってないか?
出せなくはないけど(0,1,tan 75°)が全然行きてない。
頂点が(0,0,0)でCとC’の境目がz=0ならちょっといい感じだけど。
z座標の設定変えてAの座標とか境目の座標そのままにしちゃったんじゃない?
2円錐の共通域出すだけのしょうもない作業のわりには数値うるさすぎて下らない。
0263132人目の素数さん
垢版 |
2018/05/13(日) 02:56:42.81ID:dQIRZ6zE
>>4

nが奇数のとき
 f(x) = {x(1-x)}^{(n-1)/2} (2x-1),
 Max{ |f(x)| ; 0≦x≦1 } = (1/2)^(n-1) √{(n-1)^(n-1) / (n^n)}, x = 1/2 ± 1/(2√n),

nが偶数(n≧4)のとき
 f(x) = {x(1-x)}^(n/2 -1) (2x-1)^2,
 Max{ f(x) ; 0≦x≦1 } = (1/2)^(n-3) √{(n-2)^(n-2) / (n^n)}, x = 1/2 ± 1/√(2n),

nが偶数(n≦4)のとき
 f(x) = {x(1-x)}^(n/2),
 Max{ f(x) ; 0≦x≦1 } = (1/2)^n, x = 1/2,

エレガントな解答スレ2-813
0264132人目の素数さん
垢版 |
2018/05/13(日) 05:27:32.19ID:9cj8IIRg
1,2,3 の各数字が1つだけ書かれたカードが袋の中に大量にある。
袋からカードを1枚引き、そのカードに書かれた数字を記録する操作を繰り返す。どの数字のカードを引くかは同様に確からしいとする。
kを自然数とする。

(1)操作を繰り返し、記録された数字の和がちょうどkになるか、k+1またはk+2になった時点で操作を終了する。和がkになる確率P(k)をkで表せ。

(2)lim[k→∞] P(k) を求めよ。
0265132人目の素数さん
垢版 |
2018/05/13(日) 05:46:15.31ID:e0GLK8FT
Gを位数nの有限群、dをnの約数とするとき、x^d=1をみたすxがd個より多くなる例を教えて下さい
0266132人目の素数さん
垢版 |
2018/05/13(日) 07:57:55.69ID:mBXeE/BU
>>264
(1)
p(-2) = p(-1) = 0, p(0) = 1と定めておいて
p(k+3) = (1/3((p(k+2)+p(k+1)+p(k))
(2)
1/3
0267132人目の素数さん
垢版 |
2018/05/13(日) 08:00:35.82ID:GygA6/J7
>>265
Gを位数2の巡回群の2個の直積として
位数が2の約数の元の数=4>2
0268132人目の素数さん
垢版 |
2018/05/13(日) 08:38:00.31ID:h0T9Njyo
>>266
1/6 orz
0269132人目の素数さん
垢版 |
2018/05/13(日) 08:47:41.18ID:ZQnefJyn
>>266
1/2 orz
0270132人目の素数さん
垢版 |
2018/05/13(日) 10:03:17.57ID:brauS78B
最小二乗法について教えてください。

(x1, y1), (x2, y2), …, (xn, yn)とする、n個のxとyの値が分かっているペアがあります。これらが以下の方程式

y = a * {sinh(bx)}^c

を満たす場合、最小二乗法を使って係数a, b, cを求めたいです。

どうにもうまく求められなかったため、分かる方求め方を教えてください。

また、最小二乗法ではない方法でなら求められるのであれば、その方法でもokです。

よろしくおねがいします。
0271132人目の素数さん
垢版 |
2018/05/13(日) 10:56:45.38ID:9cj8IIRg
正方形ABCDがあり、4点A,B,C,Dはこの順に反時計回りに並んでいる。
辺CDを1:3に内分する点をE、線分EAを1:4に内分する点をF、BCの中点をMとする。
このとき、∠MAE=∠FOEを示せ。
0274132人目の素数さん
垢版 |
2018/05/13(日) 12:13:20.86ID:dQIRZ6zE
>>271 >>273

AB:BM = MC:CE
∠B = ∠C = 90°
∴ 儁AB ∽ 僞MC
MA:ME = AB:MC = BM:CE = 2:1
∠AME = 90°
Mから対辺AEに垂線MHを下す。
儁EH ∽ 僊MH ∽ 僊EM
∴ AH:HM = HM:HE = AM:ME = 2:1
∴ AH:HE = 4:1
∴ H = F
∴ ∠MAE = ∠HME = ∠FME
0275132人目の素数さん
垢版 |
2018/05/13(日) 12:15:34.32ID:mSJNHrCr
>>266
>p(k+3) = (1/3((p(k+2)+p(k+1)+p(k))
q(k+2)=p(k+3)-p(k+2)=-2/3(p(k+2)-p(k+1))-1/3(p(k+1)-p(k))=-2/3q(k+1)-1/3q(k)
3q(k+2)+2q(k+1)+q(k)=0
3t^2+2t+1=0
t=(-1±i√2)/3=t±
q(k)=A(t+)^k+B(t-)^k
p(k)-p(0)=A(1-(t+)^(k+1))/(1-t+)+B(1-(t-)^(k+1))/(1-t-)
p(∞)-1=A/(1-t+)+B/(1-t-)=A/(5/3+t-)+B/(5/3+t+)=(A(5/3+t+)+B(5/3+t-))/(5/35/3+(t++t-)+t+t-)=(5/3q(0)+q(1))/(25/9-2/3+1/3)=(5/3(p(1)-p(0))+(p(2)-p(1)))/(22/9)=(p(2)+2/3p(1)-5/3p(0))/(22/9)=(4/9+2/31/3-5/3)/(22/9)=(4+2-15)/22=-9/22
p(∞)=1-9/22=11/22=1/2
0276132人目の素数さん
垢版 |
2018/05/13(日) 12:16:38.80ID:mSJNHrCr
>>275
>p(∞)=1-9/22=11/22=1/2
p(∞)=13/22
0278132人目の素数さん
垢版 |
2018/05/13(日) 12:37:29.56ID:mSJNHrCr
>>275
>(5/35/3+(t++t-)+t+t-)=
(5/35/3+5/3(t++t-)+t+t-)=2
p(∞)=1-9/2222/91/2=1/2
0279132人目の素数さん
垢版 |
2018/05/13(日) 12:39:56.60ID:mSJNHrCr
>>269
むしろなぜこの値なのか
簡単な理由がありそうな
0280132人目の素数さん
垢版 |
2018/05/13(日) 12:53:34.46ID:fpkwHaKc
>>271
△ABMと△MCDが相似で相似比は2:1。
MからAEに垂線の足Gを下ろしたとき△AME、△AGM、△EGMは相似で辺の比は1:2:√5。特にAG:GE=4:1でF=G。
0281132人目の素数さん
垢版 |
2018/05/13(日) 13:20:40.90ID:9cj8IIRg
xy平面の点A(1,1)を通る直線と円C:x^2+y^2=1が交点を持つとき、その交点のAに近い方をPとする(ただ一つの交点を持つ場合はそれをPとする)
また、この直線上のAから見てPの側に点Qをとり、AP・AQ=kとなるようにする。ここでkは正の定数である。

(1)点Qが動いてできる曲線Kにより、円Cの内部が面積が等しいように二分される場合のkの値を求めよ。

(2)(1)のとき、KとCとの2交点をそれぞれS,Tとする。sin(∠SAT)≧(i/10)となる最大の整数を求めよ。
0282132人目の素数さん
垢版 |
2018/05/13(日) 13:56:16.96ID:XAMB7xRz
0.999…=1 の説明ってさ、「実数は連続であるから」でいいよね?
0283132人目の素数さん
垢版 |
2018/05/13(日) 14:18:16.41ID:NMjJwVYY
連続をどのような意味で使っているか、疑問ではあるが、一言で言うなら、小数の表現の性質。
同じ値に対し、複数の表現方法があるのに、この事実を知らず、不思議がっている人がいるだけ。
0284132人目の素数さん
垢版 |
2018/05/13(日) 14:40:09.37ID:ig+KKpxF
まずは無限小数の定義を考えるところから
それさえわかれば上に有界な単調増加だから収束することは実数の連続性
収束値が1になることは明らか
0286132人目の素数さん
垢版 |
2018/05/13(日) 16:55:37.57ID:mSJNHrCr
>>285
ない
0288132人目の素数さん
垢版 |
2018/05/13(日) 17:03:43.23ID:bjhAtIi0
0.111111111111111111111111111111111111111111111111・・・・・・・・・・・・・・X9
0289132人目の素数さん
垢版 |
2018/05/13(日) 17:17:53.34ID:mSJNHrCr
>>287
それ許すなら無限にあるわけだが
0290132人目の素数さん
垢版 |
2018/05/13(日) 17:40:44.73ID:1Hs4Nvmc
>>289
形式的計算で 2-0.99999…=1.00000… だから
0.99999…と1.00000…はそれぞれ下からと上から近づく2パターンの表現
それ以外の無数の表現の例はどんなのがあるの?
0291132人目の素数さん
垢版 |
2018/05/13(日) 17:52:41.25ID:mSJNHrCr
1.0
0292132人目の素数さん
垢版 |
2018/05/13(日) 17:53:24.41ID:mSJNHrCr
>>290
>上から近づく
0294132人目の素数さん
垢版 |
2018/05/13(日) 18:20:06.25ID:1Hs4Nvmc
>>291
例ありがとう。

でも有限で0が終わったら厳密に1に等しいけど
無限に0が続く場合それが1に等しいことは自明ではない気がする
0295132人目の素数さん
垢版 |
2018/05/13(日) 18:34:16.99ID:9cj8IIRg
次の性質(A)を持つ立体は存在しないことを示せ。

(A)どのような平面で切っても、切断面の面積は常に同じ値をとる。
0296132人目の素数さん
垢版 |
2018/05/13(日) 18:36:47.36ID:mSJNHrCr
>>293
何それ?
0297132人目の素数さん
垢版 |
2018/05/13(日) 18:37:30.46ID:mSJNHrCr
>>294
>無限に0が続く場合
とはどういう定義か考えてないの?
0298132人目の素数さん
垢版 |
2018/05/13(日) 18:50:56.10ID:NsIUdaFs
z=cosθ+i sinθのとき
⑴2cosθ=z+1/zを示せ
⑵2 cosnθ=z^n+1/z^nを示せ(分母にだけn)
⑶3z^4-z^3+2z^2-z+3=0のとき
 a, 6cos2θ-2cosθ+2=0
b. 方程式の4つの実数ではない解を示せ

この問題たちがわからない、だれか部分部分でもいいので教えてください
0299132人目の素数さん
垢版 |
2018/05/13(日) 19:12:07.48ID:gywqs905
>>298
3.(a)意味ぷー
(b)3z^4-z^3+2z^2-z+3=(3z^2-4z+3)(z^2+z+1)と2次の判別式。
0300132人目の素数さん
垢版 |
2018/05/13(日) 19:25:43.07ID:NsIUdaFs
>>299 thx ごめん、3のaは3z^4-z^3+2z^2-z+3=0を6cos2θ-2cosθ+2=0の形に変形できることを示せ、でしたm(_ _)m
0301132人目の素数さん
垢版 |
2018/05/13(日) 19:53:28.92ID:zoXfCvTV
>>300
そんなこと成り立たんでしょ?3z^2-4z+3=0の解は
z=(1/3)(2±√5i)なので絶対値は1とは限らん。
0302132人目の素数さん
垢版 |
2018/05/13(日) 19:54:31.64ID:ig+KKpxF
(1)オイラーの公式
(2)オイラーの公式
0303132人目の素数さん
垢版 |
2018/05/13(日) 21:13:45.97ID:w1azPazB
>>300>>301
寝ぼけてた。>>301
>z=(1/3)(2±√5i)
これ絶対値1やね。でも絶対値が全部1であること示すよりz+1/z=tとおいて
与式⇔3(t^2-2)-t+2=0⇔t=4/3,-1
を出してz+1/z=-1とz+1/z=4/3とく方が早い。誘導どうりやった方がしんどいクソ誘導は無視すべし。
0304132人目の素数さん
垢版 |
2018/05/13(日) 21:36:55.44ID:dQIRZ6zE
>>298 >>300

(b) 3zz -z +2 -1/z +3/zz = 3(z+1/z)^2 - (z+1/z) - 4 = {3(z+1/z) -4} {(z+1/z) +1},

 z+1/z = -1 から z = (-1±i√3)/2 = e^(±i(2π/3)),

 z+1/z = 4/3 から z = (2±i√5)/3,
0306132人目の素数さん
垢版 |
2018/05/14(月) 00:13:38.70ID:N5/oSqy7
>>267
>>277
ありがとうございます
0307132人目の素数さん
垢版 |
2018/05/14(月) 00:14:59.96ID:N5/oSqy7
もう一つ質問なのですが整域上の次数nの多項式で根をn個以上持つものはありますか?
0308132人目の素数さん
垢版 |
2018/05/14(月) 00:25:51.14ID:s90IxSDn
>>256
ちなみにですけど、この無限級数がπcoth(πa)の形で表される事ってどのように導出しましたか?
0309132人目の素数さん
垢版 |
2018/05/14(月) 00:31:49.43ID:cV/gIJVZ
>>307
四元数でx^2=-1とか
0312132人目の素数さん
垢版 |
2018/05/14(月) 00:44:19.98ID:BPMfd3hq
>>281

交点のAから遠いほうの交点を P~ とする(ただ一つの交点を持つ場合はそれを P=P~ とする。)
方べきの定理より、AP・AP~ = 1,
題意より、AQ = k・AP~
∴ Kは、円C(x>0 かつ y>0 の部分を除く)を、Aを中心としてk倍したもの。
∴ Kは、中心が (1-k,1-k) 半径がkの円(x>1-k かつ y>1-k の部分を除く)
 (x-1+k)^2 + (y-1+k)^2 = k^2,

(1) k = 0.728967367687286

(2) ∠SOT /2 = α とおく。
 cosα = (3-k)/√8 = 0.802931287302128
 sinα = 0.596071596262855
 tan(∠SAT /2) = sinα・(√8)/(1+k)
 ∠SAT = 1.54560093958281 < π/2
 sin(∠SAT) = 0.99968261302211979

相変わらずセンスのない作問者…
0313132人目の素数さん
垢版 |
2018/05/14(月) 00:47:19.25ID:N5/oSqy7
ありがとうございます
では整域を可換環に置き換えると存在するでしょうか?
0314132人目の素数さん
垢版 |
2018/05/14(月) 00:57:39.14ID:rt0PhzAS
線形代数ってなんですか?
何が線形なんですか?
a11 a12
a21 a22
☝これの何が線形なんですか
0316132人目の素数さん
垢版 |
2018/05/14(月) 01:05:05.51ID:cHwVvG6S
>>314
難しいこと考える前に計算できるようにしときましょう
0318132人目の素数さん
垢版 |
2018/05/14(月) 01:10:49.92ID:N5/oSqy7
>>315
ありがとうございます
0319132人目の素数さん
垢版 |
2018/05/14(月) 01:38:22.71ID:V8xenapF
零因子のある可換環で多項式関数値が零因子になるような理論はまだ未整備か?
>>315では x^2+x の x に対象としている環のどの元を代入しても零因子、もしくは0になる例
0324132人目の素数さん
垢版 |
2018/05/14(月) 02:38:15.96ID:F8vZ30dB
1つの群には複数の環構造が入りますか?
つまりA,Bを環としてAとBが加法群として同型でも環としては同型でないことはありますか?
0325132人目の素数さん
垢版 |
2018/05/14(月) 02:44:07.84ID:V8xenapF
そんな程度なら、片一方を0環にすればいいだけの話だろ。
も少し建設的な問題を設定せよ。
0327132人目の素数さん
垢版 |
2018/05/14(月) 04:45:06.25ID:N5/oSqy7
>>319
ありがとうございます
最近勉強を始めたばかりで殆ど何も知りません
それについて詳しく書かれている本はありますか?(もしくは通常数行で済ませてしまうようなことでしょうか?)
0328132人目の素数さん
垢版 |
2018/05/14(月) 04:54:51.17ID:vWTDt18w
>>325
環は単位的なものを想定していました
0329132人目の素数さん
垢版 |
2018/05/14(月) 10:04:11.06ID:IhdOpBbv
自分で作った問題で自分で解けないときならスレ違いではないと思うけどそれならそれでその旨は書いといてほしい。
そういうのは解けない、解けるにしてもドエライ解になってしまうかもしれないから。
0332132人目の素数さん
垢版 |
2018/05/14(月) 10:43:53.74ID:6uJ2QcNR
さすがのwolframも未定の関数が入ってる式は処理できないんだなぁ。
0334132人目の素数さん
垢版 |
2018/05/14(月) 15:06:51.16ID:SGjgBc66
日本民法の父、穂積陳重の『法窓夜話』を現代語に完全改訳

法律エッセイの古典的名著が短編×100話で気軽に読めます
リライト本です。「なか見検索」で立ち読み頂けます。原版は
国立国会図書館デジタルコレクションで無料で読めます

法窓夜話私家版 (原版初版1916.1.25)
https://www.amazon.co.jp/dp/B07BT473FB
(続)法窓夜話私家版 (原版初版1936.3.10)
https://www.amazon.co.jp/dp/B07BP9CP5V
0335132人目の素数さん
垢版 |
2018/05/14(月) 16:04:25.17ID:F+1vy2oQ
あるところを境に全てが0になる規則性をもった数列って存在しますか?
あるとしたらどんな一般項になるんでしょう
一応自分が考えた案としては、前の項+前の項×なんらかの数列(例えばマイナスから始まる奇数項)なんですが、これの一般項の求め方がわかりません
教えて下さい
0337132人目の素数さん
垢版 |
2018/05/14(月) 18:12:16.53ID:SJeCUFf8
すみません、わからない問題というか、質問なんだけど

「あなたは自分にとって∫f(x)dxにおけるdxだ」

と言われた場合のdxって何ですか?
たぶん告白だろうとは思うんですが、きちんと意味をつかんでから返事がしたいので
文系の自分にはさっぱりです
0338132人目の素数さん
垢版 |
2018/05/14(月) 19:12:06.43ID:VRvJuuxP
>>337

まらん
0339132人目の素数さん
垢版 |
2018/05/14(月) 19:28:25.94ID:gmAMWTBM
>>317
ああ、それはそうですね
寝ぼけたこと書いてました
0341132人目の素数さん
垢版 |
2018/05/14(月) 20:39:16.96ID:XUjOMsY1
質問です。
任意の自然数nに対して、次の不等式が成り立つことを示せ。
2^(n+2)>n^2
という問題なのですがこれって任意の自然数じゃなくてn≧3のときですよね?
0342270
垢版 |
2018/05/14(月) 20:39:23.72ID:7Rk7XC5X
>>270もお願いします。
0344132人目の素数さん
垢版 |
2018/05/14(月) 21:06:09.18ID:XUjOMsY1
>>343
すいません。解けましたありがとうございます
0345132人目の素数さん
垢版 |
2018/05/14(月) 22:26:03.02ID:xg2RAmH0
⑴zとwが複素数(x+yi)で、w=1/1-z, (絶対値z)^2=1のとき、wの実数部分xを求めよ
⑵zがcisθのときz^2-1/z^2+1=i tanθを証明せよ
誰かこの二つお願いm(_ _)m
cisはr (cosθ+i sinθ)
0348132人目の素数さん
垢版 |
2018/05/14(月) 23:12:49.35ID:s90IxSDn
>>308
お願いします
0349132人目の素数さん
垢版 |
2018/05/14(月) 23:18:34.60ID:VYiPAVkp
タイラー天気じゃないのかな?
0351132人目の素数さん
垢版 |
2018/05/14(月) 23:35:38.99ID:GuJhfaMX
間違ってません
0352132人目の素数さん
垢版 |
2018/05/14(月) 23:35:47.06ID:48jxLhgY
>>348
一般にはHadamardの因数分解定理だけど三角関数とかだと初等的な証明もある。Wikipediaにも載ってるはず。
0354132人目の素数さん
垢版 |
2018/05/15(火) 00:49:59.50ID:KmuIpfTz
>>345 >>347
(z^2-1)/(z^2+1)だな
0355132人目の素数さん
垢版 |
2018/05/15(火) 01:27:17.00ID:eFrawDDn
>>308 >>348

無限乗積表示(オイラー):
 sinh(πa) = πa・Π[n=1,∞] {1 + (a/n)^2} = πa・Π[n=1,∞] {1 + (a/n)i}{1 - (a/n)i},
対数をとってaで微分する。
 π coth(πa) = 1/a + Σ[n=1,∞] 2a/(aa+nn) = 1/a + Σ[n=1,∞] {1/(a-in) + 1/(a+in)},
0356132人目の素数さん
垢版 |
2018/05/15(火) 02:23:17.12ID:Omn+setj
zは複素数で、複素数平面上の単位円上を動く。
複素数wをw=z+(z")^2+2z"とするとき、wが動いてできる曲線で囲まれる領域の面積をSとする。
(zの共役複素数をz"と表した)

(1)S≧nをみたす最大の非負整数nを求めよ。

(2)nは(1)で求めた値とする。
S≧n+(i/4)をみたす最大の非負整数iを求めよ。
0357132人目の素数さん
垢版 |
2018/05/15(火) 02:29:40.74ID:11UbkqUX
天上神と東大史上最高の天才はどっちの方が賢いですか?
0358132人目の素数さん
垢版 |
2018/05/15(火) 02:49:14.33ID:yTfs4dgd
>>356
複素数の問題で虚数単位以外の意味で使う i が同時に出てきたらあかんだろ
そして出題したいだけならよそにスレを立ててやってくれ
その方があとで参照するときにも都合がよい
0359132人目の素数さん
垢版 |
2018/05/15(火) 03:31:19.89ID:RWV1I2yh
>>348 >>308

おそらく最も単純な方法:f(z)=πcot(πz)/(z^2+a^2)と置いて留数定理を用いると目的の級数が得られる。

おそらく最も初等的な方法:三角関数の2N倍角の公式と根と係数の関係より
cot(x) = (1/(2N))Σ[n=0,2N-1]cot((x+πn)/(2N))
= (1/(2N))[cot(x/(2N)) + Σ[n=1,N-1]{cot((x+πn)/(2N)) - cot((-x+πn)/(2N))}]
が成り立ち、N→∞とすると和のペア部はO(1/n^2)で絶対収束するので極限の交換ができて
cot(x) = 1/x + Σ[n=1,∞]{1/(πn+x) - 1/(πn-x)}
そしてx=aπiと置くと目的の級数が得られる。
0360132人目の素数さん
垢版 |
2018/05/15(火) 04:02:58.65ID:eFrawDDn
>>356

z = e^(i・2π/3),z = -1,z = e^(i・4π/3) で w = -2 となる。(3重点)

∴3つの単純閉曲線が w = -2 で交わったもの…
0362132人目の素数さん
垢版 |
2018/05/15(火) 16:20:33.30ID:/SFsFp0F
今日Wolfram君に教えてもらいました。

integral_0^∞ x^(-s) sin(x) dx = cos((π s)/2) Γ(1 - s) for 0<Re(s)<2…(1)
integral_0^∞ x^(-s) cos(x) dx = sin((π s)/2) Γ(1 - s) for 0<Re(s)<1…(2)

(1)でs=1のとき Dirichlet積分、s=1/2のときFresnel積分となかなかかっっちょええ公式。
Wolfram君は不定積分も教えてくれて確かに微分して元の積分核が出ることもx=0のとき-右辺になることもチェックはできます
…が、こんなん思いつくかボケ!んなもん不定積分なんかせんでもHankelの公式で一撃じゃ
…でもなかったorz。
なんか(1)の左辺をHankelの公式で計算すると(1)と(2)の左辺が混ざった形がでてきて切り離せない???
どなたか初等的な証明(=留数定理とかCauchyの積分公式とか級数展開とかまで)知ってます?orできます?
もしかしてこの積分なんか名前ついてます?ちなみに数学辞典には(1)の方はのってます。
0365132人目の素数さん
垢版 |
2018/05/15(火) 17:16:23.18ID:eFrawDDn
>>356

 z = e^(it),  -π≦t≦π.
とおける。
 w = e^(it) + e^(-2it) +2e^(-it)
 u = Re{w} = 3cos(t) + cos(2t),
 v = Im{w} = -sin(t) -sin(2t),

s(t1,t2) = ∫[t1,t2] u '(t) v(t) dt = -∫[t1,t2] u(t) v '(t) dt

>>360 により3つに分ければ
s(-π,-2π/3) = 5π/6 -(3√3)/2 = 0.0199176666
s(-2π/3,2π/3) = 10π/3 + 3√3 = 15.6681279347
s( 2π/3, π) = 5π/6 -(3√3)/2 = 0.0199176666

S = s(-π,π) = 5π = 15.70796327

(1) n=15

相変わらず趣旨が分からない問題
0366132人目の素数さん
垢版 |
2018/05/15(火) 19:02:23.40ID:Omn+setj
一辺の長さが1の正四面体を平面で切ったときに出来る多角形全体からなる集合をSとする。
Sの要素のうち面積が最大である多角形からなる集合をTとする。

(1)Tに属する多角形はすべて合同であることを示せ。

(2)Tの要素の1つをkとする。以下の命題の真偽を判定せよ。
「kの外接円の半径は、Sに属する多角形の外接円の半径のうちで最大である」
0367132人目の素数さん
垢版 |
2018/05/15(火) 19:10:27.52ID:j0q7+E4u
以下の六つの中で、最も天才と呼ぶのに相応しいのはどれですか?

超絶天才数学者
超絶天才プログラマー
超絶天才画家
超絶天才ピアニスト
超絶天才建築家
超絶天才彫刻家
0369132人目の素数さん
垢版 |
2018/05/15(火) 19:39:16.43ID:hVzl3U0R
環の問題2つです.
次の証明が合っているか,教えてください.
よろしくお願いします.

[1]R:環とする.
   任意のRの元xについて,x^2=xが成り立つ
   ならば,
   任意のRの元x,yについて,xy=yxが成り立つ.
[証明]
  条件から
   (x+y)^2=x+y
   x^2+xy+yx+y^2=x+y
   xy=−yx.
  また,
   1=1^2=(−1)^2=−1.
  よって,
   xy=yx. □□

[2]R:環とする.
   任意のRの元xについて,x^3=xが成り立つ
   ならば,
   任意のRの元x,yについて,xy=yxが成り立つ.
[証明]
  条件から
   x^2−x=(x^2−x)^3=x^6−3x^5+3x^4−x^3=4(x^2−x)
   3(x^2−x)=0.
   (x^2−x)^2=x^4−2x^3+x^2=2(x^2−x)=−(x^2−x)
   {−(x^2−x)}^2=−(x^2−x)
  Rの元yについて,y^2=yをみたす元の集合をZ(R)とすると,
  Rの部分環になる.
   (x^2)^2=x^2,{−(x^2−x)}^2=−(x^2−x)
   x^2,−(x^2−x)は,Z(R)の元.
   x=x^2−(x^2−x)は,Z(R)の元.
   x^2=x 
   xy=yx. □□
0371132人目の素数さん
垢版 |
2018/05/15(火) 20:04:13.05ID:j0q7+E4u
>>370
理由を教えてください。
0372132人目の素数さん
垢版 |
2018/05/15(火) 20:46:22.89ID:uUBv6rUz
>>369
ええでえ
0374132人目の素数さん
垢版 |
2018/05/15(火) 20:50:01.16ID:hVzl3U0R
>>372 さま
ありがとうございます.
助かりました.
0376132人目の素数さん
垢版 |
2018/05/15(火) 21:37:59.72ID:uUBv6rUz
>>366
三角形か四角形
三角形の最大は表面の正三角形
四角形は三角形より狭い
0377132人目の素数さん
垢版 |
2018/05/15(火) 21:59:58.14ID:klxreP45
>>375
ペアノ算術を含む任意の無矛盾な公理系に対し、あるモデルM,Nおよび論理式φが存在して、M|=φかつN|≠φとできることを示せ、という問題がわかりません
0379132人目の素数さん
垢版 |
2018/05/15(火) 22:26:26.17ID:In+Nnt+U
>>375の宇宙がポリゴンで出来てるって事だろ
0381132人目の素数さん
垢版 |
2018/05/16(水) 00:39:15.17ID:Yf3BPH11
しっかし誰も解けない難しい質問ばっかでつまんねえなぁ。
本当に「実際は解いている連中ばっか」状態になったこと一度もねえじゃんw
もっと簡単な質問してこい、脳みそウンコまみれの底辺層ども。
0384132人目の素数さん
垢版 |
2018/05/16(水) 01:10:27.38ID:hoGpnmIF
しょうがないなあ簡単な質問してやる。
これ解けません

一辺の長さが1の正方形の形をした折り紙がある。1つの角を平面の原点Oに重ね、Oから出る2辺をx軸とy軸の正の部分に重ねる。(1,0)にある角をAとする。
a>0とし、直線y=axに沿って、この折り紙のAを含む側を他方の側に重なるよう折り曲げる。
以下の問に答えよ。

(1)a=1/2,a=2のとき、それぞれ2枚の紙が重なる部分の面積を求めよ。

(2)一般のaに対し、2枚の紙が重なる部分の面積を求めよ。
0385132人目の素数さん
垢版 |
2018/05/16(水) 01:11:16.97ID:Je9uJcpl
>>369
>Rの元yについて,y^2=yをみたす元の集合をZ(R)とすると,
>  Rの部分環になる.
ここ証明できてない希ガス。和について閉じてる事は要証明じゃね?
0387132人目の素数さん
垢版 |
2018/05/16(水) 01:14:44.80ID:hoGpnmIF
こちらはより簡単

a,bを整数とする。
x^4+ax^+bが整数を係数とする2つの既約な多項式の積に因数分解できるとき、a,bが満たす必要十分条件を求めよ。
0388132人目の素数さん
垢版 |
2018/05/16(水) 01:14:46.03ID:Je9uJcpl
>>383
すまん。正確には
与式⇔f(x+1)^2 = f(x)^2、f(x)≠f(x+1)⇔f(x) = - f(x+1)
相変わらず死ぬほど解ある。
0390132人目の素数さん
垢版 |
2018/05/16(水) 01:17:40.09ID:hoGpnmIF
そろそろこのスレとお別れしたいが、大学受験面白い問題載ってる本を教えてくれ
難問がいい
あと、大数とチャートの難問集は読んで実際もう解いた
理科も理一の合格点取れるしあまり勉強することないんだわ
0393132人目の素数さん
垢版 |
2018/05/16(水) 01:38:40.60ID:/MdEF5MU
>>380
f(x)^2=g(x) とおけば
g(x+1)-g(x)=1
もし定義域が実数全体であるなら、十分小さい x で g(x)<0 となる。
さらにもし関数 f(x) が実数値であると仮定するなら g(x)=f(x)^2≧0 より矛盾。
定義域をいじったり複素数値を許せば死ぬほどある
0394132人目の素数さん
垢版 |
2018/05/16(水) 01:44:22.11ID:iriy4b81
>>369

〔参考書〕
数セミ増刊「数学の問題」第(1)集、日本評論社 (1977) No.72
数セミ増刊「数学の問題」第(2)集、日本評論社 (1978) No.60 & 増補
N.Jacobson: "Structure of rings" Amer. Math. Soc. (1964) の 10章、§1

〔一般化〕
任意の x∈R に対して自然数 n(x) >1 があって x^n(x) = x ならば Rは可換。(ベキがxにより異なってもおk)
0以外のベキ零元をもたない有限環は可換。

〔参考文献〕
N. Jacobson: Annals of math., 2nd series, 46(4), p.695-707 (1945/Oct)
 "Structure theory for algebraic algebras of bounded degree"
  http://www.jstor.org/stable/1969205

A. Forsythe & N. H. McCoy: Bull. Amer. Math. Soc., 52(6), p.523-526 (1946)
 "On the commutativity of certain rings"
 http://pdfs.semanticscholar.org/4eb0/7131867b9883b7aebd93a5ed74db74824a14.pdf
0400132人目の素数さん
垢版 |
2018/05/16(水) 03:39:08.52ID:Je9uJcpl
>>366
>Sに属する多角形の外接円の半径のうちで最大である

って外接円持たないSの要素はどうするん?
0401132人目の素数さん
垢版 |
2018/05/16(水) 04:33:38.95ID:Bv6MQJ5v
東大の入試問題作りました
良問だと思います

第一問
一辺の長さが10の正四面体ABCDがある。
辺AB上にAP=kとなる点Pを、辺AC上にAQ=mとなる点Qをとる。ただしk,mは10より小さい正整数である。
このとき、△PQDの面積が整数となる(k,m)の組が存在するか、結論と理由を述べよ。
0402132人目の素数さん
垢版 |
2018/05/16(水) 04:41:27.32ID:Bv6MQJ5v
東大の入試問題作りました
良問だと思います

第ニ問
pを素数、m,nを1≦m≦p-1、1≦n≦p-1を満たす正整数とする。
このとき、二項係数の比
(pCm)/(pCn)……(A)
を既約分数の形で表わせ。二項係数を用いて表してよい。
0403132人目の素数さん
垢版 |
2018/05/16(水) 04:53:42.08ID:Bv6MQJ5v
東大の入試問題作りました
良問だと思います

第三問
x軸およびy軸の正の部分にそれぞれ点P,Qがあり、PQ=1を満たすように動く。
座標平面の原点をOとし、△OPQの内接円をCpqとする。また、Cpqの周および内部の領域をDpqとする。

(1)点Pが(1,0)に限りなく近づくとき、Cpqの中心はどのような点に限りなく近づくか。同様に、Pが(0,0)に限りなく近づく場合はどうか。

(2)PQが動くとき、座標平面上でDpqに含まれうる領域の面積を求めよ。
ただしPが(0,0)および(1,0)に一致する場合は、(1)で求めた点をDpqとせよ。
0404132人目の素数さん
垢版 |
2018/05/16(水) 05:01:23.77ID:iriy4b81
>>369 [2]

以下、第(2)集 No.60 からのコピペ  >>394

 xx・xx = x・x^3 = x・x つまり xx はベキ等。

〔補題〕 xxyy = yyxx,  ……(6)

xx=X,yy=Y はベキ等だから、
 X -Y = (X-Y)^3 = X^3 -Y^3 -XYX +YXY = X -Y -XYX +YXY,
 XYX = YXY,
 XY = (XY)^3 = (XYX)(YXY) = (YXY)(XYX) = (YX)^3 = YX,

 (xy)^2 = xx(xy)^2 = (xy)^2・xx,
 xy = (xy)^3 = (xy)^3・xx = (xy)xx … (7)

 (xy)^2 = (xy)^2・yy = yy(xy)^2 
 xy = (xy)^3 = yy(xy)^3 = yy(xy) …… (8)
xとyを入れ替えて
 yx = xx(yx) …… (9)
(8)*(7)
 (xy)^2 = yy(xy)・(xy)xx = y(yx)^3・x = y(yx)x = yy・xx = xx・yy,
xとyを入れ替えて
 (xy)^2 = (yx)^2,
 xy = (xy)^3 = (xy)(yx)^2 = x(yyxy)x = x(xy)x = yx …… (10)
0405132人目の素数さん
垢版 |
2018/05/16(水) 05:45:28.63ID:qlzfKH7q
>>362 >>364

Γ(s)の定義式を積分路変更することで容易に導けます
(Hankel の公式からでも導出可能ですが、リーマン面で考えないといけないので少し面倒になります)

定義式:Γ(1-s)=∫[0,∞] z^(-s) e^(-z) dz において積分路を実軸から虚軸に変更する
(厳密には半径rとRの1/4円弧と実軸と虚軸上の線分を結ぶ閉曲線を考えr→0,R→∞とする)と
Γ(1-s)=∫[0,∞] (it)^(-s) e^(-it) d(it)
=∫[0,∞] i^(1-s) t^(-s) (cos t -isin t) dt
となって、この式の両辺に i^(-1+s)=e^(πi(-1+s)/2)を乗じれば直ちに(1),(2)式が得られます
また(1)式が0<s<2でも成り立つことはs平面での解析接続より明らか
0408132人目の素数さん
垢版 |
2018/05/16(水) 06:54:49.93ID:iriy4b81
>>384
しょうがないから軽く解くか…

 y=ax (これは単位円の直径)に関してAと対称な点をA'とする。
 A' = ((1-aa)/(1+aa),2a/(1+aa))も単位円周上にある。
 0<a<1 では Aを含む側 ⊂ 反対側
 a>1 では Aを含む側 ⊃ 反対側
よって
 (1/2)min{a,1/a}
0409132人目の素数さん
垢版 |
2018/05/16(水) 07:45:09.56ID:hMthHlx8
じゃ>>401
面積は1/4√(4(k^2-10k+100)(m^2-10m+100)-(km-10k-10m+200)^2)。
√のなかはmod 5で3k^2m^2に合同。よって平方数になるにはどっちか5。
k=5としてよい。
面積は5/4√(15m^2-20m+700)。
さらにmは5でないとだめ。
でも√のなかは24375。
奇数なのでアウト。
0410132人目の素数さん
垢版 |
2018/05/16(水) 07:53:54.90ID:NcyWMFku
>>404
x^4=xだとかに一般化は出来るの?
0411132人目の素数さん
垢版 |
2018/05/16(水) 07:57:56.87ID:lbcFBVcw
>>403
(1)略
(2)内接円を固定して斜辺をうごかすときlog(cot(x))が下に凸よりOP・OQが最小となるのはOP=OQのとき。
よって条件内で内接円の半径が最大となるのはOP=OQ=1のとき。以下ry
0414132人目の素数さん
垢版 |
2018/05/16(水) 08:23:39.86ID:ej8w+lSP
わかりません。
0415132人目の素数さん
垢版 |
2018/05/16(水) 08:34:10.43ID:Yf3BPH11
>>413
•dotupはリンク先からさらにリンクに飛ばなければならないのでなるべく使わないようにしましょう
•パソコンで数式書く暇があるなら自分で調べましょう
0416132人目の素数さん
垢版 |
2018/05/16(水) 08:41:37.04ID:IdiwIaZW
1日 a リットルずつ供給される水道で1日 b リットルまで到達させている
b リットルまで x 倍の時間で到達させたい場合、時間あたりの供給量を y 倍すれば良い

xとyの関係を式で表すとどうなりますか?
0417132人目の素数さん
垢版 |
2018/05/16(水) 08:49:02.25ID:mSOZV66q
>>415
タイトル100万回見直せば如何?
0418132人目の素数さん
垢版 |
2018/05/16(水) 08:54:33.64ID:Yf3BPH11
>>417
へー、てことは、わからないんですね(笑)
0419132人目の素数さん
垢版 |
2018/05/16(水) 09:16:07.30ID:c+HnAvYR
xについての不定積分にθが現れているという点で解法2の答え方は不十分
x=tanθとおいたのだからcosθ=(1+x^2)^(-1/2)を用いて変形すれば解法1の答えと一致する
0421イナ ◆/7jUdUKiSM
垢版 |
2018/05/16(水) 10:55:23.89ID:njbpuZLY
/_/_/人人_/_/_/_
/_/_(_^_)/_/_/_
/_/_(_)_)/_/_/_
/_/_( (`_)/_/_/_
/_/_(_っ-┓_/_/_
/_/_◎゙┻υ◎゙/_/_/_/_/_/_/_/_/_/_/_/_/キコキコ……。>>350分母を有利化すると見た目が違ってきます。髪の毛が風でうしろになびくと頭頂部の見た目が変わるでしょ。
0422132人目の素数さん
垢版 |
2018/05/16(水) 11:21:08.44ID:hpTFF7qj
画像中央の行にある式の波線部分がどのような指数計算をして出したのか途中式を交えて教えてくだされば幸いです
二項定理そのものはわかりますがこの指数計算がわからず質問しました
よろしくお願いします
https://i.imgur.com/y5krm7F.jpg
0423132人目の素数さん
垢版 |
2018/05/16(水) 12:19:58.64ID:ej8w+lSP
眠い
0424132人目の素数さん
垢版 |
2018/05/16(水) 13:11:47.77ID:phdn5f0F
422
(x^2)^k (2/x)^(10-k)=x^(2k) 2^(10-k) x^(-(10-k))
=x^(2k) 2^(10-k) x^(-10+k)=2^(10-k) x^(2k) x^(-10+k)
=2^(10-k) x^(2k-10+k)=2^(10-k) x^(3k-10)
0426413
垢版 |
2018/05/16(水) 15:11:17.93ID:mSOZV66q
>>419
なるほど
そういう事か
3Q
0428132人目の素数さん
垢版 |
2018/05/16(水) 16:49:16.19ID:iriy4b81
>>410

指数nが偶数のとき
 (-x)(-x) = xx,
 の両辺を n/2 乗して
 -x = (-x)^n = x^n = x,
 また
 x + xx + … + x^(n-2) ∈ Z(R)

n=2 のとき(ベキ等環)
 xy + yx = (x+y)^2 -x^2 -y^2 = (x+y) -x -y = 0
 ゆえ反可換、可換。

n=4 のとき
 x + xx ∈ Z(R) より
 x(xy+yx) = (xy+yx)x,
 xxy = yxx,
 ここで x→xx とすれば
 xy = x^4・y = y・x^4 = yx,

n=6 のとき
 n=2 に帰着する。

n=4〜8は証明できたらしい。(淡中忠郎教授)
0429132人目の素数さん
垢版 |
2018/05/16(水) 17:08:33.77ID:GzTDB8cM
まぁだからどやねんというツッコミが絶えないんだよなぁ、>非可換環ろん
0430132人目の素数さん
垢版 |
2018/05/16(水) 18:54:03.69ID:X5WFdbcF
東大の入試問題作りました
良問だと思います

第四問
iは虚数単位、p,qは実数とする。
数列{a_n}を以下のように定める。
a_1=1、a_2=i、
a_(n+2)=pa_(n+1)+qa_n

(1)すべてのnに対し|a_n|=1となるために、p,qが満たすべき必要十分条件を求めよ。

(2)すべての点Pn(a_n)が同一円周上にあるために、p,qが満たすべき必要十分条件を求めよ。
0432132人目の素数さん
垢版 |
2018/05/16(水) 19:39:58.76ID:X5WFdbcF
東大の入試問題作りました
良問だと思います

第五問
空間の2点(0,0,0)と(1,0,0)を直径の両端とする円をC1、2点(0,0,1)と(1,0,1)を直径の両端であとする円をC2とする。
C1とC2を底面とする円柱を曲面z=x^2によって2つの領域に切り分けるとき、領域の体積比V1:V2を求めよ。
0433132人目の素数さん
垢版 |
2018/05/16(水) 19:40:36.33ID:i0JFdzw4
東大の入試問題を作りました
良問だと思います

ある無矛盾な公理系τの任意のモデルに対してある論理式φが常に真となるならば、τからφがLKにおいて証明可能となることを示せ
0434132人目の素数さん
垢版 |
2018/05/16(水) 19:49:29.84ID:X5WFdbcF
東大の入試問題作りました
良問だと思います

第六問
a,bは正の実数とし、xの関数f(x)をf(x)=ax-b+e^(-x)と定める。相異なる実数p,qに対して、f(p)とf(q)の大小を比較せよ。
0436132人目の素数さん
垢版 |
2018/05/16(水) 20:27:54.26ID:+HHSIr+p
東大の入試問題を作りました
良問だと思います

なぜ劣等感婆は数学板、物理板を荒らすのか?
400字以内で回答せよ。キイワードは必須とする。

キイワード
素人、劣等感、自演
0437132人目の素数さん
垢版 |
2018/05/16(水) 21:08:19.70ID:MPlRBdj8
>>413
結論はどちらも正しい。

2つめはθをxに直せば、θ→arctan xであるが
cos (arctan x)を調べたら、(1+x^2)の(-1/2)乗と等しいので
上と一致する
0438132人目の素数さん
垢版 |
2018/05/16(水) 21:17:32.79ID:zoVvkzUT
ここの回答者って、簡単な問題だとすでに回答がついていても同じ回答つけるんですな
0440132人目の素数さん
垢版 |
2018/05/16(水) 21:25:12.26ID:+HHSIr+p
東大の入試問題を作りました
良問だと思います

なぜ劣等感婆は基礎論を勉強したのに予備校をくびになったのか?
400字以内で回答せよ。
0442270
垢版 |
2018/05/16(水) 23:00:10.70ID:1o6QYqoa
>>270は無理でしょうか……
0443132人目の素数さん
垢版 |
2018/05/17(木) 02:04:16.25ID:3t2VcDet
工夫すればエレガントに解けるそうですが
教えてください。

立方体ABCD-EFGHと1~10までのカードが1枚ずつある。
このカードの中から8枚選び、各頂点に1枚ずつ割り当てる。このとき、
”4つの頂点のカードの数字の和が偶数となる”ような面がちょうど3つ存在する確率を求めよ。
0444132人目の素数さん
垢版 |
2018/05/17(木) 02:27:49.32ID:1BF/qMvl
>>439
第1問は面積を表すまでは簡単ですがその先で余りに着目できないと詰みです
C***

第2問はpCiが全てpで割り切れることを知らないと手がつけられません。その後のpCkとpCmの処理も難しく、6問中の最難問です
D#

第3問は円板の通過領域の把握がやや難しいです。面積計算は平易です
C***

第4問は6問中最も易しいです
B**

第5問も易しいですが、円柱をタテに切らないと計算が面倒になります
B**

第6問は意外に難しいです。場合分けや論証に手こずると思います
C****
0445132人目の素数さん
垢版 |
2018/05/17(木) 02:34:09.45ID:1BF/qMvl
>>439
第4問と第5問は完答必須。ここで20*2の40点は確保したい

第1問と第3問はあわせて1題分以上の点数を取りたい。20点程度を目標

第6問は手数がかかる上に緻密さも必要で、後回しにしたい
第2問は出来るところまでで残りは捨てたほうがいい
合計で1題ブン取れれば上出来、20点弱が目標

合格者平均
理一65、理ニ55、理三80
0447132人目の素数さん
垢版 |
2018/05/17(木) 02:39:01.75ID:rInW77ym
>>443
エレガントかどうか知らんけど
奇数のカード数≡奇数の面数 (mod 2)
より奇数のカード数は奇数。
このとき常に奇数の面数は3。
∵奇数カード数が1なら自明(無視していいケースだが)
奇数カード数が3、隣接頂点に奇数カードが配置されるときは残り1枚の奇数カードをどこにおいても(実質2ケースしかない)奇数の面数は3。
奇数カード数が3、隣接頂点に奇数カードが配置されないときはどの2頂点も対角に配置できないから正三角形の頂点をなすように配置するしかなく、奇数の面数は3。
奇数カード数が5のときは奇数カードと偶数カードの配置を総入れ替えすれば奇数カード3のケースに帰着される。
以上により奇数カード数が奇数となることが条件。
確率は2×C[5,3]×C[5,5]/C[10,8]。
0448132人目の素数さん
垢版 |
2018/05/17(木) 02:43:36.84ID:rInW77ym
>>446
いつまで受験数学レベルやってんの?もう卒業したら?
もっと素晴らしい世界がひらけてるのに。
そのくらいの問題作れるなら次のステップに進む素地は整ってるんだから。
0449132人目の素数さん
垢版 |
2018/05/17(木) 02:50:56.14ID:1BF/qMvl
>>448
大学受験で大学数学使えないんで
それに実数とか極限とか当たり前のことを精密に議論するのって何が楽しいんですか
線形代数やっとけって先輩から言われましたが使いみちが分かりません。空間の点を回転したり対称移動できるとかつまらない

大学行ってから勉強します
今は遊びます
0450132人目の素数さん
垢版 |
2018/05/17(木) 03:04:52.90ID:FDCSht5h
>>270

b=b0 を固定する。
 
log(y) = log(a) + c・log|sinh(bx)|
より
 X = log(sin(bx)),Y = log(y)
とし、(X,Y)データを最小二乗法で直線回帰する。
 Y = log(a) + c・X
ただし、(a,c) は b0 に依存する。

次に、
Z = sinh^(-1){(y/a)^(1/c)}
とし、(x,Z)データを最小二乗法で直線回帰する。
Z = b・x + d,
ただし、(b,d)は(a,c)に依存する。

これを (a,b,c,d) が収束するまで繰り返す。

SCF (Self-consistent Field)
0451132人目の素数さん
垢版 |
2018/05/17(木) 03:05:20.32ID:FDCSht5h
>>270

b=b0 を固定する。
 
log(y) = log(a) + c・log|sinh(bx)|
より
 X = log(sin(bx)),Y = log(y)
とし、(X,Y)データを最小二乗法で直線回帰する。
 Y = log(a) + c・X
ただし、(a,c) は b0 に依存する。

次に、
Z = sinh^(-1){(y/a)^(1/c)}
とし、(x,Z)データを最小二乗法で直線回帰する。
Z = b・x + d,
ただし、(b,d)は(a,c)に依存する。

これを (a,b,c,d) が収束するまで繰り返す。

SCF (Self-consistent Field)
0452132人目の素数さん
垢版 |
2018/05/17(木) 03:23:01.81ID:FDCSht5h
>>403

 P(p,0) Q(0,q) p>0,q>0 とする。
 題意より pp+qq=1
 Cpq の中心 (r,r) ここに r = (p+q-√(pp+qq))/2 = (p+q-1)/2,
(1) p→1 のとき q→0,r→0
  p→0 のとき q→1,r→0

(2) は >>411
0453132人目の素数さん
垢版 |
2018/05/17(木) 03:56:27.34ID:FDCSht5h
>>430

a_{n+2} = p・a_{n+1} + q・a_n,
より
|a_{n+2}|^2 = a_{n+2}・a_{n+2}~
= (p・a_{n+1} + q・a_n) (p・a_{n+1}~ + q・a_n~)
= pp|a_{n+1}|^2 + pq・Re(a_{n+1}・a_n~) + qq|a_n|^2,

(1) pp + qq = 1,pq = 0,
  (p,q) = (0,±1) (±1,0)
0456132人目の素数さん
垢版 |
2018/05/17(木) 07:28:57.67ID:1xjVKUsZ
広島大学理学部化学科とかですかね
0458132人目の素数さん
垢版 |
2018/05/17(木) 10:39:15.71ID:qdvcO8MT
>>449

精密に議論するのは、雑な議論で答えだけ出るのと比べて結局同じ答えしか出てないからつまらなく感じるかもしれないけど、それはより深い世界へと進むための準備で大切な事だよ。
今の日本だと “受験” という人参をぶらさげて他の人が出せない答えがどれだけだせるか?という競争の中で勉強させられるから間違いやすいけど数学は決して “ほら、俺こんな問題もとけるぜ” って得意がるための道具じゃないよ。
高々受験数学レベルの問題のあたりを一生ウロチョロして終わるか、数学が真に “人生の友” と呼べる素敵な “学問” になるかの分かれ目だねぇ。
0459132人目の素数さん
垢版 |
2018/05/17(木) 12:37:07.69ID:0BPyzBl6
>>443 >>447
ちょい改善。
abcdefghをそれぞれABCD-EFGHに奇数カードのせたとき1,偶数カードのとき0をとる変数とする。
奇数面1、5個がないことを示せば良い。
ABCDのみが奇数面なら
a+b+c+d≡1 (mod2)。
一方他の面は偶数面だから
a+b+e+f≡0 (mod2) c+d+g+h≡0 (mod2) e+f+g+h≡0 (mod2)
の3つ足して
a+b+c+d≡0 (mod2)。
∴矛盾。
奇数面5個がないのは今の議論で右辺の0と1を入れ替えれば良い。
0460132人目の素数さん
垢版 |
2018/05/17(木) 12:49:27.37ID:wzusziGa
>>458
ある無矛盾な公理系τの任意のモデルに対してある論理式φが常に真となるならば、τからφがLKにおいて証明可能となることを示せ

精密な議論でよろしくお願いしますね
これがわからないということは、論理や証明とは何かが分かっていないことと同義です
0462132人目の素数さん
垢版 |
2018/05/17(木) 13:23:27.09ID:kGwb7h50
まぁ何も高いレベルに挑戦し続けることだけが数学ではないからね。
受験数学レベルを楽しむのもよし。数学的なレベルは問題じゃない。
それで “俺様ってすごいだろ” って事にしか数学を使えてないのが問題だねぇ。
基礎論の勉強も結局 “お前らはそこまでやってないだろ” って粋がりたいためにしか使えてない。完全性定理レベルでねぇ。
0463132人目の素数さん
垢版 |
2018/05/17(木) 14:04:36.68ID:Iir1USr+
>>460
回答がありませんね
何故でしょうか
0465132人目の素数さん
垢版 |
2018/05/17(木) 14:15:29.34ID:DyYiDvcL
それエーデルワイズの定理だろ、
0467132人目の素数さん
垢版 |
2018/05/17(木) 14:16:03.22ID:Iir1USr+
>>464
わかるなら答えられるはずですね
答えないということは、わからないということですね
0469132人目の素数さん
垢版 |
2018/05/17(木) 14:36:52.12ID:bK8jl4eF
>>465
そうなん?完全性定理じゃないの?基礎論の教科書なんてもう何年も開いてないから忘れてしまったorz
0470132人目の素数さん
垢版 |
2018/05/17(木) 14:37:17.34ID:Iir1USr+
わからないんですね(笑)
0471132人目の素数さん
垢版 |
2018/05/17(木) 14:45:00.32ID:8bFcDFwY
>>2

>>460
いつまでみっともないこと晒すかねこの人
0472132人目の素数さん
垢版 |
2018/05/17(木) 14:59:15.60ID:zdWSgKfO
今度の劣等感婆は無差別に噛み付く芸風に変えたのか
0473132人目の素数さん
垢版 |
2018/05/17(木) 15:05:24.94ID:bK8jl4eF
諭されて改める=負け
なんだろうなぁ。一生懸命に数学勉強して獲得したものがこれだけなのはちょっとかわいそうではある。
今の “他人を打ち負かしてナンボ” という受験至上主義のかわいそうな犠牲者ではある。
0474132人目の素数さん
垢版 |
2018/05/17(木) 15:28:55.68ID:Iir1USr+
わからない人が何か言ってますね
0476132人目の素数さん
垢版 |
2018/05/17(木) 15:38:11.25ID:RMm2CIYv
「ある無矛盾な公理系τの任意のモデルに対してある論理式φが常に真となるならば、τからφがLKにおいて証明可能である」という命題が証明可能であることを示せ

という問題が分かりません。教えてください
0477132人目の素数さん
垢版 |
2018/05/17(木) 15:49:13.11ID:0y+f26PG
ここでこの答えを書く人は居ません
分からない人は書けませんし、分かる人は書きません
なぜならあなたの文章は人を試しているように見えるからです(あなたの本心は関係ありません あなたの文章を他人がどうとらえるかが大事です)
あなたに認められるより問題が解けた事実の方がはるかに価値があるのです
0478132人目の素数さん
垢版 |
2018/05/17(木) 15:55:17.88ID:Iir1USr+
私はリーマン予想が解くことができましたが、ここには書きません
書けたという事実が大事だからです
フィールズ賞は論文提出しなくてももらえるんですかね
0479132人目の素数さん
垢版 |
2018/05/17(木) 16:20:03.46ID:PIxyPgar
余白が狭すぎる
0481132人目の素数さん
垢版 |
2018/05/17(木) 16:29:39.43ID:1BF/qMvl
一辺の長さが1の正五角形ABCDEと、正方形CDFGがある。ただし2つの図形は直線CDについて同じ側にある。
点Fはこの正五角形の内部、周上、外部のいずれに位置するか。
0483イナ ◆/7jUdUKiSM
垢版 |
2018/05/17(木) 18:20:49.21ID:l1vlYSML
>>481同じ側なんだから題意から五角形の中じゃん? てか、かっこいいね、
_
/\龍雄って名前。
\_/ ̄ ̄ ̄ ̄ ̄ ̄/\
 ̄| ̄∩∩ ∩∩ ̄\/|
_|((-_-)-_-)) / |
 ̄|`(っu~)U⌒U、/| |
]| ‖υυ~UU~‖ |/ \
_| ‖ □ □ ‖ /  /
 ̄\‖____‖/  /|
_________/||
 ̄ ̄ ̄ ̄ ̄ ̄ ̄ ̄‖| |/
] □  □`;,□ ‖| /
______;__‖|/_
________‖//_/_/_/_/_/_/_/_/_/_/_/_/_/_/_/_/_/_
0484132人目の素数さん
垢版 |
2018/05/17(木) 18:53:54.51ID:Kc1v/rUO
尋常じゃないくらい頭が悪いけど、東京大学理学部数学科に入りたい。
0485132人目の素数さん
垢版 |
2018/05/17(木) 20:30:19.59ID:374sM+up
>>477
>あなたに認められるより問題が解けた事実の方がはるかに価値があるのです
うむ
0487132人目の素数さん
垢版 |
2018/05/17(木) 21:08:32.20ID:0QGsx+P0
√9= √(-3)^2 = -3
この解き方がダメな理由を妹に説明したいです
√9はプラスの数だよと言っても納得してもらえず困っています…力を貸してください…
0488132人目の素数さん
垢版 |
2018/05/17(木) 21:24:45.18ID:DyYiDvcL
間違いではない。
0491132人目の素数さん
垢版 |
2018/05/17(木) 23:13:58.68ID:374sM+up
>>487
正しいだろって言ってやれよ
3=√3^2=√9=√(-3)^2=-3
これでいいだろって
0492132人目の素数さん
垢版 |
2018/05/17(木) 23:19:30.80ID:8TelVl60
↓この証明なんかおかしくない?
a,a'∈A
b,b'∈B
fは全射ゆえV(f)=B
よって∃[a∈A](b∈f(a))⇔b∈B
b∈f(a)⇔a∈f^-1(b)より
∃[a∈A](a∈f^-1(b))⇔b∈B
よってD(f ^-1)=B-@
fは写像ゆえ(b∈f(a)∧b'∈f(a))⇒b= b'
集合の相当の定義より(b∈f(a)⇔b∈f(a')) ⇒f(a)=f(a')
よって(b∈f(a)∧b∈f(a'))⇒f(a)=f(a')
fは単射ゆえf(a)=f(a')⇒a=a'
よって(b∈f(a)∧b∈f(a'))⇒a=a'
b∈f(a)⇔a∈f^-1(b)より
(a∈f^-1(b) ∧a'∈f^-1(b))⇒a=a'-A
@、Aより全単射の写像fの逆対応f ^-1は写像


特にこの部分とかめっちゃ飛躍してるよね
>fは写像ゆえ(b∈f(a)∧b'∈f(a))⇒b= b'
>集合の相当の定義より(b∈f(a)⇔b∈f(a')) ⇒f(a)=f(a')
>よって(b∈f(a)∧b∈f(a'))⇒f(a)=f(a')
>fは単射ゆえf(a)=f(a')⇒a=a'
>よって(b∈f(a)∧b∈f(a'))⇒a=a'
0494132人目の素数さん
垢版 |
2018/05/17(木) 23:25:27.27ID:5tSjyrNL
>>492
まず、4行目の  b∈f(a)  なんて記述が写像を理解していない証。
どう書かなければいけないかが分かったところで、もう一度質問を書き込んでみてね。

4行目で読む気が失せたので。
0495132人目の素数さん
垢版 |
2018/05/17(木) 23:28:56.03ID:girYMyxF
>>494
え?f(a)=bという表記じゃないと認めないってこと?本来はf(a)={b}なんだから別にいいと思うけど
0497132人目の素数さん
垢版 |
2018/05/17(木) 23:33:00.23ID:girYMyxF
>>496
何が違うの?対応って知ってる?
0499132人目の素数さん
垢版 |
2018/05/17(木) 23:42:52.53ID:girYMyxF
>>498
対応による像が部分集合である以上定義域が始集合と等しく各像が1つの元から成る対応を写像としているわけだから写像による像も部分集合と認めざるを得ないよね
f(a)=bは慣習的表記に過ぎない
0500132人目の素数さん
垢版 |
2018/05/17(木) 23:47:32.78ID:5tSjyrNL
>>499
ナンセンス


きみがおかしな証明といっているその証明の対象である命題は、多分こういうことなのだろうと思う。

集合Aから集合Bへの全単射fが存在するとき、逆像対応 f^(-1);B→2^A は f の逆写像を定義していることを示せ。
ここに fの逆像対応f^(-1)とはb∈Bに対してf^(-1)(b)={a∈A|f(a)=b}で定まる対応である。
0501132人目の素数さん
垢版 |
2018/05/17(木) 23:50:26.44ID:girYMyxF
>>500
いや、俺は>>492じゃないよ
0503132人目の素数さん
垢版 |
2018/05/17(木) 23:55:50.59ID:Mrc1Smh5
一般的な(教科書等に書いてある)記述はf(a)=bまたはf({a})={b}なので、それに合わせるのは普通だと思うが
0505132人目の素数さん
垢版 |
2018/05/18(金) 00:00:14.72ID:AZta2T+Z
>>502
妄言ってアンタ…写像は対応の一種でしょうが
0506132人目の素数さん
垢版 |
2018/05/18(金) 00:02:29.20ID:phLq/mSI
>>503
写像 f(a)=b に対して f({a})={b} と書いてある教科書をご教示下さい。

写像f が集合間写像である汎写像を定義する、という類の説明は別の問題、ということでよろしく。。
0507132人目の素数さん
垢版 |
2018/05/18(金) 00:02:53.55ID:AZta2T+Z
まぁ別にこだわるところでもないからいいけどさ、もう俺は寝る
0511132人目の素数さん
垢版 |
2018/05/18(金) 00:14:12.78ID:m1kJBbKU
>>495
>え?f(a)=bという表記じゃないと認めないってこと?
認めないんじゃないの?
0512132人目の素数さん
垢版 |
2018/05/18(金) 00:15:59.45ID:m1kJBbKU
>>499
f({a})とf(a)とを近藤
0513132人目の素数さん
垢版 |
2018/05/18(金) 00:16:30.09ID:IChGANrB
√ は二課関数だから、どっちでもいい。
0514132人目の素数さん
垢版 |
2018/05/18(金) 00:18:11.29ID:6J8mIcZT
>>495ってこういうこと?
https://ja.m.wikipedia.org/wiki/対応_(数学)
定義
対応 f = (A, B, Gf) は、
「各元 a ∈ A に対して (a, b) ∈ Gf となるような b ∈ B が一つしかない(すなわち、A のどの元 a についても f(a) がただ一つの元からなる)」
という条件をみたすとき、部分写像(一意対応)という。特に D(f) = A(全域的)なとき写像と呼ばれる。
対応 f が(部分)写像であるとき、f(a) = {b} となることを f(a) = b と略記して、この元 b を a の像と呼ぶ。
0515132人目の素数さん
垢版 |
2018/05/18(金) 00:23:03.44ID:+W5C6ZEg
>>506
誤解を避けるために言っておくと、f({a})={b}という書き方をしても構わないという話な
このような表記が許されるということが分かる例という話なら、例えば集合位相入門(松坂)p.30だが
0516132人目の素数さん
垢版 |
2018/05/18(金) 00:27:22.85ID:K2Dl9lJX
おお、ほんと。wikipediaはそういう書き方なのか。ビックリ。
でもこれは一般的な記法ではないに一票。
この書き方したらいかんとは言わないけど使うなら一言但し書きいれないとだめじゃね。
てかwikipediaこんな特殊な書き方するなら一言入れないとダメな気がする。
もしかして俺のほうが少数派なのかもしれんけど。
0517132人目の素数さん
垢版 |
2018/05/18(金) 00:29:48.54ID:K2Dl9lJX
>>515
wikipediaのリファレンスも松阪先生の教科書だね。
この書き方のほうがメジャーなん?
初めて見る……
0518132人目の素数さん
垢版 |
2018/05/18(金) 00:57:09.46ID:7GRL3rM4
f(集合)=集合

こんなの像の定義そのものですよね
どれだけレベルが低いんですか、このスレッドは
0519132人目の素数さん
垢版 |
2018/05/18(金) 04:05:37.85ID:JUWy+p9g
天国と数学はどっちの方が凄いのでしょうか?
0520132人目の素数さん
垢版 |
2018/05/18(金) 04:19:42.13ID:539vwTx6
>>481

△DEFは2等辺△だから
∠DEF < 90゚ < 108゚ = ∠AED
∴ 点Fは∠AED の内部にある。 … (1)

CDの中点をMとすると、対称性より、
∴ 点F は∠AMD の内部にある。 … (2)

(1)(2)より、点F ⊂ ◇AMDE ⊂ ABCDE
0521132人目の素数さん
垢版 |
2018/05/18(金) 05:20:32.37ID:fyKum6I9
(1)nは2以上の正整数とする。n!は平方数にならないことを示せ。

(2)kは2以上の正整数とする。k個の連続した正整数の積は平方数にならないことを示せ。
0522132人目の素数さん
垢版 |
2018/05/18(金) 05:49:12.03ID:/qlvVszp
>>519
真面目に数学の勉強するのは地獄だって自覚表明みたいなこといつまで続けるつもりなの?
0523132人目の素数さん
垢版 |
2018/05/18(金) 05:51:31.10ID:fyKum6I9
p,qは整数とする。
a_1=1、a_2=2、a_(n+2)=pa_(n+1)+qa_nである数列{a_n}について、以下の問に答えよ。

(1)任意のiに対しa_(i+1)>a_iとなるために、p,qが満たすべき必要十分条件を求めよ。

(2)p,qは(1)の条件を満たすとする。さらに{a_n}が以下の条件[C]を満たすために、p,qが満たすべき必要十分条件を求めよ。
[C]どのような3以上の整数jに対しても、1≦mj<kj≦3である正整数mj,kjが存在して、{a_(j+kj)-a_(j+mj)}/(kj-mj)がa_j,a_(j+1),a_(j+2),a_(j+3)のいずれかに等しくなるようにできる。
0524132人目の素数さん
垢版 |
2018/05/18(金) 06:20:27.53ID:CvPbHZEO
>対応 f が(部分)写像であるとき、f(a) = {b} となることを f(a) = b と略記して、この元 b を a の像と呼ぶ。

この定義の仕方は初めて見たな。普通は f(a)=∪{b} (右辺は和集合の公理から定まる集合)
と定義しないか?これなら文字通り f(a)=b だよ。
0525132人目の素数さん
垢版 |
2018/05/18(金) 07:13:56.88ID:7GRL3rM4
↑これが数学板の実力です
専門板なのに異常にレベルが低い
せいぜい数学の少しできる高校生レベル


本当にレベル低すぎませんか?
この程度なんですか、数学板って
0527132人目の素数さん
垢版 |
2018/05/18(金) 09:57:55.27ID:tBeUSVjx
1/z
0528132人目の素数さん
垢版 |
2018/05/18(金) 11:47:13.48ID:dH1Oea0s
あざます
≠0の時点で察するべきだったか…
0529132人目の素数さん
垢版 |
2018/05/18(金) 15:25:53.82ID:fyKum6I9
2次方程式の実数解は座標平面上だとx軸と放物線の交点に対応します
そこで、虚数解を座標平面の何かに対応させることはできるんでしょうか?
虚数とはいえ、その値は放物線のパラメーターに依存するので、放物線や座標平面との図形的対応ができないかと考えています
0532132人目の素数さん
垢版 |
2018/05/18(金) 17:47:10.12ID:syEIqgPS
>>530の質問内容の書き忘れです
なぜ0<k<b, 0<l<bという風にkとlの範囲が絞られているのかもお教え頂けますか
連投すみません
0535132人目の素数さん
垢版 |
2018/05/18(金) 18:27:13.33ID:i54mOA7u
自作ボードゲーム市場に詳しい「ペンとサイコロ」というブログの
「ゲムマ2017秋・アンケート結果 第二弾:2016→2017年比較」の記事によると

ゲームマーケットに出品した人の半分が赤字で半分が黒字でちょうど半々だそうだ
50万以上の儲けが5%いるが逆に50万以上赤字なのも5%いる
そして初参加の人の7割が赤字なのに対して、ノウハウありや知名度や固定ファン層が居る
中堅サークル7割が黒字になってる
継続性とブランド力構築とノウハウが大事だという事だと思う
初参加の人は作る個数と需要を見極めツイッターやユーチューブでの宣伝がカギになる
最初は50〜100個ぐらいをいかに金かけないで作って売るかの勝負になる

これがゲムマ2016・2017年(初の二日開催)の販売数
https://cdn-ak.f.st-hatena.com/images/fotolife/r/roy/20171220/20171220211924.png
これが販売金額
https://cdn-ak.f.st-hatena.com/images/fotolife/r/roy/20171220/20171220212902.png
これがイベントでの利益
https://cdn-ak.f.st-hatena.com/images/fotolife/r/roy/20171220/20171220213109.png
0536132人目の素数さん
垢版 |
2018/05/18(金) 20:46:20.13ID:Q0zCCt8k
教えてください

xy平面上で、D={(x,y)| 3≦x≦5, 3≦y≦5}とする。
Dをx軸を中心に回転させた立体をD、Dをさらにy軸を中心に回転させた立体をE
とするとき、Eの体積を求めよ。
0538132人目の素数さん
垢版 |
2018/05/18(金) 21:50:21.55ID:drDf2Sp5
1+1/2+1/2+1/3+1/3+1/3+1/4+1/4+1/4+1/4+1/5+1/5+1/5+1/5+1/5+.......の部分和を表す一般的な式はありますか?
発散することを示したいです
0539132人目の素数さん
垢版 |
2018/05/18(金) 23:19:58.38ID:uUybNQSy
a[n]=(2n+k^2+k)/(2k+2) ただし、k=Floor[(Sqrt[8n+1]-1)/2]
0540132人目の素数さん
垢版 |
2018/05/18(金) 23:41:23.02ID:m1kJBbKU
>>516
>おお、ほんと。wikipediaはそういう書き方なのか。ビックリ。
ウィキペディアが嘘
0541132人目の素数さん
垢版 |
2018/05/18(金) 23:42:30.96ID:m1kJBbKU
>>518
だから
f(集合)=集合
f(元)=元
0544132人目の素数さん
垢版 |
2018/05/19(土) 00:15:09.51ID:oHNHehfx
515 名前:132人目の素数さん :2018/05/18(金) 00:23:03.44 ID:+W5C6ZEg
>>506
誤解を避けるために言っておくと、f({a})={b}という書き方をしても構わないという話な
このような表記が許されるということが分かる例という話なら、例えば集合位相入門(松坂)p.30だが


これのどこが、f(集合)=元なんですか?
0545132人目の素数さん
垢版 |
2018/05/19(土) 00:19:02.80ID:9nKwXEtV
>>540
基礎論とかでそういう導入をするのかもしれない。
一般的な定義と同値なら問題は無いし、嘘とまで言う必要はないだろう。
0546132人目の素数さん
垢版 |
2018/05/19(土) 00:24:19.21ID:+EZsmfjZ
一般的な定義はどんな感じなんですか?
0547132人目の素数さん
垢版 |
2018/05/19(土) 00:27:47.79ID:nPyNoqtI
アスペ婆さん
0548132人目の素数さん
垢版 |
2018/05/19(土) 00:29:49.28ID:D3NocoQb
この問題が分かりません。点QがL1とL2の間に入り込む場合や、L2の向こう側にある場合など、場合をどう分けたらいいでしょうか。

a,bを実数とする。
xy平面上の2点A(-5,-5),B(-5,5)を結ぶ線分から両端を除いたものをL1、2点C(5,-5),D(5,5)を結ぶ線分から両端を除いたものをL2とする。
点P(-9,3)からL1もL2も通らずに点Q(a,b)に至る最短経路の長さをa,bで表し、ab平面に図示せよ。
0549132人目の素数さん
垢版 |
2018/05/19(土) 00:35:09.63ID:NIi6yyUc
1+2/2+3/3+4/4 + ….. + n/n = n かな。
0550132人目の素数さん
垢版 |
2018/05/19(土) 00:36:15.50ID:+EZsmfjZ
>>547
わからないんですね
0551132人目の素数さん
垢版 |
2018/05/19(土) 00:41:23.92ID:9nKwXEtV
>>546
普通に数学する分には、それこそwikipediaの「写像」のページにあるように
>集合 A の各元に対してそれぞれ集合 B の元をただひとつずつ指定するような規則 f が与えられているとき、f を「始域または定義域A から終域 B への写像」といい
ぐらいで十分でしょ
ZFC公理とか気にする文脈なら>>514みたいにすればいい
0552132人目の素数さん
垢版 |
2018/05/19(土) 00:47:37.72ID:IVV+JcaX
一般的ではない自分好みの記述をしたければいくらでもやればいいとはおもうけど、それならそれでその旨明記せんとダメやろ。一応辞書なんだから。
0553132人目の素数さん
垢版 |
2018/05/19(土) 00:52:23.24ID:S5Pbjzeo
>>548
問題文おかしくね?
「最短経路の長さを a,b で表す」 は意味があるが
その後にいきなり 「ab平面に図示せよ」 は不自然だ
0554270
垢版 |
2018/05/19(土) 00:53:13.24ID:cYWcA9YX
>>450

ありがとうございます。やはり反復計算になってしまうのですね。
0555132人目の素数さん
垢版 |
2018/05/19(土) 00:54:38.70ID:+EZsmfjZ
>>551
なるほど
数学板でもそこまでレベルが低いとは思いませんでした
流石ですね
0556132人目の素数さん
垢版 |
2018/05/19(土) 01:00:48.90ID:kHKNlegy
君が数学板のレベルをあげるんだ!
人を試すのではなく自分を試せ!
0557132人目の素数さん
垢版 |
2018/05/19(土) 01:08:16.43ID:+EZsmfjZ
>>506
よく見たらこういうのもひどいですね
なんですか汎写像って
0558132人目の素数さん
垢版 |
2018/05/19(土) 01:14:42.12ID:xGV7D22I
よく気付いたね。ほめてあげる。
この際だから、今回の文脈に合うように「汎写像」の定義を作ってみたらどう。
0559132人目の素数さん
垢版 |
2018/05/19(土) 01:15:05.08ID:+EZsmfjZ
あと、ちなみにですけどウィキペディアの集合とか写像云々って基本的には素朴集合論ですからね

ここの回答者は、基礎論どころか普通の集合論すら分かっていないということが判明してしまいましたね
0560132人目の素数さん
垢版 |
2018/05/19(土) 01:20:44.53ID:xGV7D22I
「素朴集合論」ってのは慣用語の類なので、その中身は人に依りけり。
0561132人目の素数さん
垢版 |
2018/05/19(土) 01:21:25.54ID:7y7NJIN+
対応(多価写像)の特別な場合として写像を定義するならf({a})={b}をf(a)=bと書く
「対応」を定義せずに単に集合の元を対応させる規則ならそのままf(a)=b

これでいいだろ
前者も後者も同値だろうが
0562132人目の素数さん
垢版 |
2018/05/19(土) 01:24:11.97ID:+EZsmfjZ
それにケチをつけた人がいたんですよねー
0563132人目の素数さん
垢版 |
2018/05/19(土) 01:25:00.31ID:IVV+JcaX
あのwikipediaの記事はやっぱりアウトやろ。
あんな一般的でない “abuse of notation” なんのことわりもなく使ってんだから。
しかもあの使い方はあかんやろ。
f(a)と書いた場合それが一般的な意味におけるf(a)なのかf({a})なのか前後の文脈をみないといけなくなる。
自分の論文とかに書く分にはまぁすきにすればいいけどネットで人に公開するとこにかくのはあかんやろ。最低でもその旨但し書きがないと。
wikiは玉石混交やなぁ。
0564132人目の素数さん
垢版 |
2018/05/19(土) 01:27:41.45ID:BMhUCtvT
>>548
1.点ABCDのいずれも通らない場合、
2.点Aのみを通る場合
3.点Bのみを通る場合
4.点Aと点Cを通る場合
5.点Bと点Dを通る場合
の五通りかと思ったけど、Cに行くには、B経由の方が短いみたいなので4.は
4'.点Bと点Cを通る場合
に置き換えた五通りでいいんじゃない?

あと問題の意図は、図示した領域毎に、aとbの関数で表せっていう意味だよね。
0565132人目の素数さん
垢版 |
2018/05/19(土) 01:29:06.63ID:+EZsmfjZ
>>563
対応がわかりません、ってはっきり言ったらどうなんですか?
見苦しいですよ
0567132人目の素数さん
垢版 |
2018/05/19(土) 01:30:44.14ID:nPyNoqtI
Wikipediaの記事が信用に値しないのは数学に限らないがな
善意で直してやっても差し戻しされるし
0569132人目の素数さん
垢版 |
2018/05/19(土) 01:33:57.93ID:IVV+JcaX
でもwikipediaまじで勉強になるときあるのよ。
へぇぇ、こんな公式あるんやぁぁ!ってびっくらこくときあるもんね〜。
その一方でもうアホかというのもまじってるのがなんとも残念。
0570132人目の素数さん
垢版 |
2018/05/19(土) 01:35:45.92ID:xhdfIuy0
>>538

k = Floor((√(8n-7) -1)/2) とおく。

a[n] = a[n-1] + 1/(k+1)
 = …
 = a[k(k+1)/2] + (n - k(k+1)/2)/(k+1)
 = k + (n - k(k+1)/2)/(k+1)
 = ……   >>539
0575132人目の素数さん
垢版 |
2018/05/19(土) 02:40:47.86ID:p1D17qL6
>>560
> 「素朴集合論」ってのは慣用語の類なので、その中身は人に依りけり。

まあそれはその通りだが、基本的にはラッセルの逆理の原因になる集合と真のクラスの区別をしないで
集合に関する記法を「細けぇこたぁいいんだよ」とばかりに気楽に使うのが素朴集合論だと個人的には思ってる
(これを「論」と呼ぶのは変なんだけどね、むしろ集合記法を使った計算を素朴にやってるんだから
「素朴集合算(naive set calculus)」とでも呼ぶべき代物だ)
君が賛同してくれるかどうかは知らんが

だから正にcomprehension schemeで集合を指定せずに特定の性質を満たす「もの」を平気で集めたり
基底の公理や選択公理の存在を知らない・気にしないということを素朴集合算を使う時は平気でやってるわけでね
0578イナ ◆/7jUdUKiSM
垢版 |
2018/05/19(土) 11:07:37.51ID:G63RZAIS
>>548a>5,0<b<5のとき、

PB+BD+DQ=2√5+10+√{(a-5)^2+(5-b)^2}

P→B→D→QとP→B→C→QでPB+BD+DQとPB+BC+CQが等しくなる点Q(a,b)(5<a<b)の式は、
10+√{(a-5)^2+(5-b)^2}=10√2+√{(a-5)^2+(b+5)^2}
0580132人目の素数さん
垢版 |
2018/05/19(土) 15:39:39.43ID:zVeK+Elc
めんどくさくなってるから上のスレお前ら来てくれ
0581132人目の素数さん
垢版 |
2018/05/19(土) 15:51:56.86ID:NIi6yyUc
>0.999.... = 1
定義だろ。
0583132人目の素数さん
垢版 |
2018/05/19(土) 16:52:26.90ID:B2etM5eT
お願いします。

一辺がaの正方形(a≧2)の各頂点を中心とした半径1の円Ciがある。(i=1〜4)
C1,C2,C3,C4の周上にそれぞれ点P,Q,R,Sを取る。
四角形PQRSの面積が最小となる時、PQRSは正方形か長方形となることを示し、
その最小値を求めよ。
0584132人目の素数さん
垢版 |
2018/05/19(土) 17:11:38.34ID:KgS6VdgG
>>582
無限級数の和
0586132人目の素数さん
垢版 |
2018/05/19(土) 17:29:07.97ID:8zByjxnC
wとzは複素数でw+1/w-i=z+1/z-1のとき、w=ziである。
このとき、zの虚部が0のときwの実部が0であることを証明せよ
お願いします。
0588132人目の素数さん
垢版 |
2018/05/19(土) 18:00:31.96ID:GTRz6jNj
>>492みたいな証明してる本を見たことない
大抵はこんな感じ

f^-1:B→Aが写像であることは、定義より、Bの任意の元の像f^-1(b)がただ1つから成る、すなわちBの任意の元に対してf(a)=bとなるようなAの元aがただ1つだけあることを意味する。これは明らかにf:A→Bが全単射であることを示す性質である。
よってf^-1が写像であるための必要十分条件はfが全単射であることである。

大体の本だとこういう説明的で素朴な証明で済ませてる
0589132人目の素数さん
垢版 |
2018/05/19(土) 18:04:51.85ID:KgS6VdgG
>>566
これ何証明しようとしてるの?
0593132人目の素数さん
垢版 |
2018/05/19(土) 20:26:55.03ID:D3NocoQb
>>583
2点P,Rを固定する。
QをC2上で、SをC4上で動かす。
C2の接線でPQRSの対角線PRに平行なものをl、lとC2の接点をWとする。
同様に、C4の接線でPRに平行なものをm、mとC4の接点をXとする。
△PRQの面積はQ=Wのときに最大になる。
△PRSの面積はS=Xのときに最大になる。
0594132人目の素数さん
垢版 |
2018/05/19(土) 20:29:46.67ID:M4pEwFRY
>>592

今は、コピー用紙を使っています。

>>591

便利そうですが、コストが高いですね。

そのうち、誰もが持つようになるでしょうね。
0595132人目の素数さん
垢版 |
2018/05/19(土) 21:18:42.41ID:aFnYyJyL
自殺をしたら地獄に落ちるというのは本当ですか?
無にはなれないのでしょうか?
自分としては無になってもう二度と有になりたくないのですが、無理ですか?
0597132人目の素数さん
垢版 |
2018/05/19(土) 21:25:12.81ID:xhdfIuy0
>>523 (1)

>>576 の補足

b_n = a_{n+1} / a_n とおくと題意より
 b_{n+1} = p + q / b_n,
を満たす。
b_n が収束する ⇔
 y = p + q/x,y = x
が交点α,βをもつ ⇔
(判別式) = pp + 4q ≧ 0  …… (イ)

α = {p - √(pp+4q)}/2,  は反発解
β = {p + √(pp+4q)}/2,  は吸引解

b_n → β (n→∞)となる ⇔
 α < b_1 = 2,
 p < 4 または q > 2(2-p)  …… (ロ)

題意から b_n > 1,
 β ≧ 1,
 p≧2 または q ≧ 1-p  …… (ハ)

q>0 の場合は b_n が振動するから、
 b_2 = a_3/a_2 > 1
 q > 2(1-p)  …… (ニ)
を追加する。

求める (p,q) は、(イ)(ロ)(ハ)(ニ) の共通部分。
0598132人目の素数さん
垢版 |
2018/05/19(土) 21:28:50.37ID:aFnYyJyL
>>596
根拠を教えてください。
0602132人目の素数さん
垢版 |
2018/05/20(日) 17:30:33.18ID:1IiDnvUy
>>583

・2 < a ≦ √8 の場合
 辺長 {a±√(8-aa)}/2 の長方形のとき最小
 S = (aa-4)/2,

・a > √8 の場合
 一辺が a-√2 の正方形のとき最小
 S = (a-√2)^2,
0604132人目の素数さん
垢版 |
2018/05/20(日) 22:41:10.42ID:/rtrEB4Z
3次元で直交する2つのベクトルA(ax,ay,az)とB(bx,by,bz)を、Aをz軸の負の方向(0,0,-1)に、
Bをy軸の負の方向(0,-1,0)になるように回転させたいのですが、
そのときの回転軸と回転角を計算で求める方法があれば教えてください。
ベクトルの大きさはとくに問いません。方向だけ合えばよいです。
よく起こる例としてはA(0,0,1)とB(1,0,0)、A(0,0,1)とB(1/√2,1/√2,0)です。
よろしくお願いします。
0605132人目の素数さん
垢版 |
2018/05/21(月) 01:45:11.71ID:9YF4F+CN
>>604

↑(-Z)A = (ax,ay,az+1)
 ↑(-Y)B = (bx,by+1,bz)
はいずれも回転軸と直交する。
∴回転軸ωはこれらの外積。
 ω = (ay・bz-(az+1)(by+1),(az+1)bx-ax・bz,ax(by+1)-ay・bx),


A(0,0,1) B(sinβ,cosβ,0) のとき
 回転軸ω = (-cos(β/2),sin(β/2),0)
 回転角θ= 180°
0606132人目の素数さん
垢版 |
2018/05/21(月) 03:14:15.73ID:Jy2U15S+
a,b,cは実数で、a≠0かつc≠0とする。
実数xについての関数f(x)=ax^2+bx+cで、-1≦x≦1において-1≦|f(x)|≦1を満たすものを考える。
以下の問いに答えよ。

(1)f(α)=1かつf(β)=-1となる実数α,βが存在するために、a,b,cが満たすべき必要十分条件を求めよ。
(2)(1)の条件をみたすどのようなa,b,cに対しても、g(x)=|cx^2+bx+a|が-1≦g(x)≦1を満たすことを示せ。
0612132人目の素数さん
垢版 |
2018/05/21(月) 13:51:14.72ID:ONy1xo51
カブリ数物連携宇宙研究機構と東京大学大学院理学系研究科附属ビッグバン宇宙国際研究センターはどっちの方がレベルが高いですか?
0614132人目の素数さん
垢版 |
2018/05/21(月) 15:02:50.05ID:nW2j6zgb
カブキにすれば、世界的になれたのに。
0615132人目の素数さん
垢版 |
2018/05/21(月) 15:11:00.01ID:4By2DDva
>>613
多元数理の方が良い
0616132人目の素数さん
垢版 |
2018/05/21(月) 15:37:04.92ID:nW2j6zgb
多元数となんも関係ないし・・
0617132人目の素数さん
垢版 |
2018/05/21(月) 16:31:50.57ID:ONy1xo51
>>613
そういうことじゃなくて、宇宙の研究に関してどっちの方がレベルが高いですか?
0621132人目の素数さん
垢版 |
2018/05/21(月) 20:03:38.92ID:hotZ9RFh
誰か>>583
証明の部分よろしく
0622132人目の素数さん
垢版 |
2018/05/21(月) 21:24:24.67ID:bKyOh+u+
問題ではないのだけど、

無限大をあらわす∞この記号
infinityの記号は、

sin xの波をループさせたものですよね?

sin x と infinityには
どんな関連性がありますか?
0624132人目の素数さん
垢版 |
2018/05/21(月) 21:53:08.15ID:SUavv2Mw
>>622
タロットに決まってんじゃん
0625132人目の素数さん
垢版 |
2018/05/21(月) 22:53:17.70ID:5rl5YZ/c
>>604ですが、>>605の回答がよくわかっておりません。
回転角はどのように出せばよいのでしょうか。
また回転軸と回転角は最低2ついるのではと思っています。
0626132人目の素数さん
垢版 |
2018/05/21(月) 23:15:56.55ID:YccZYzuR
>>625
^は転置とする。
(100)^をa、(010)^をbにうつす回転行列はc=a×bとして
A=(abc)で与えられる。
Aの固有多項式の2次の係数=-2cosθ、0≦θ≦πとなるをcosθから選ぶ。
このときAの固有値λは1,cosθ+i sinθ,cosθ-i sinθでそれぞれの固有ベクトルu,v,wを計算する。具体的にはA-λIの余因子行列の列ベクトルをとれば良い。
このときAはuの方向を右ねじの方向としてθ回転の行列になる。
…たぶん。
0629132人目の素数さん
垢版 |
2018/05/21(月) 23:49:36.91ID:5rl5YZ/c
>>625です。
>>626-628さんありがとうございます。
詳しく調べようと思うのですが、調べる際のキーワードはありますでしょうか。本やホームページを教えていただけると助かります。
0630132人目の素数さん
垢版 |
2018/05/22(火) 00:01:35.66ID:/dd11ki1
うーん?あるかなぁ?とりあえず>>626->>628に書いた余因子行列とか固有多項式、固有値、固有ベクトルとかは線形代数の基本なので山程ヒットするとは思う。
ネットで調べてあるかどうか知らないのは
――
任意の直交行列(回転変換を表す行列)Aは別の直交行列Pを用いて
A=PR(θ)P^
と表される。ただし
R(θ)は(100)^,(010)^を(cosθ,sinθ,0)^, (-sinθ,cosθ,0)^に移す回転行列である。
――
あるかなぁ?そんなに証明難しくないのでやってみて下さい。
これと回転行列が外積を保存することを使えば>>626->>628は証明できると思う。
0631132人目の素数さん
垢版 |
2018/05/22(火) 00:14:10.96ID:/dd11ki1
>>625
ついでなので書いとくと回転を表す流儀は
・回転軸の右ねじの方向と回転量
・オイラー角
・四元数(quotanion)
がメジャーだと思う。余力があれば調べてみて下さい。
0632132人目の素数さん
垢版 |
2018/05/22(火) 01:39:06.48ID:RuE2vaj6
>>625 >>629

 A' = A - (A・ω)ω,
 (-Z)' = (-Z) + (Z・ω)ω,
はωに直交します。
 θ = arccos{(A'・(-Z)')/|A'||Z'|}
とします。・は内積です。

"剛体回転におけるオイラーの定理" によれば、1度の回転で可能です。
回転軸ωの方向が2、回転角θが1で、自由度3です。

3次元ユークリッド空間の回転は、行列式が1の実直交行列で表わされます。( SO(3) という。)
行ベクトル、列ベクトルは正規直交性をもつため、自由度3です。

オイラー角の場合は、回転軸は z-y-z と決まっており、回転角(α,β,γ)のみを指定します。
自由度3です。
0635132人目の素数さん
垢版 |
2018/05/22(火) 04:03:52.82ID:9eWKZkbD
NASAのゴダード宇宙飛行センターの中で圧倒的最高の頭脳を誇る理論物理学者と、
オックスフォード大学の中で圧倒的最高の頭脳を誇る数学者が、
理系学問のみに関する学力バトルで勝負をしたらどっちが勝ちますか?
0638132人目の素数さん
垢版 |
2018/05/22(火) 07:57:36.57ID:h7q96Yc3
神道とイギリス王室はどっちの方が崇高ですか?
0639132人目の素数さん
垢版 |
2018/05/22(火) 07:59:03.33ID:MvGiLf+3
幸福の科学を勉強しろ!
0640132人目の素数さん
垢版 |
2018/05/22(火) 12:11:12.21ID:yXdy01CV
問題:
x の2次方程式 25*x^2 - 35*x + 4*k = 0 が異なる2つの解をもち、
それぞれ sin(θ), cos(θ) で表わされるとき、定数 k の値を求めよ。

↑これについてですが、他スレで、


347 返信:132人目の素数さん[] 投稿日:2018/05/21(月) 15:40:20.96 ID:bPLA4deP [1/2]
>>339
x の2次方程式 25*x^2 - 35*x + 4*k = 0 が異なる2つの解をもち、
それぞれ sin(θ), cos(θ) で表わされる

k=3

という解答を述べているまでだから、問題文で与えられている前提の真偽は関係ない


と言われたのですが、


この問題は、↓の意味ですよね。明らかに。

問題:
x の2次方程式 25*x^2 - 35*x + 4*k = 0 が異なる2つの解をもち、
それぞれ sin(θ), cos(θ) で表わされる。そのとき、定数 k の値を求めよ。  👀
Rock54: Caution(BBR-MD5:1341adc37120578f18dba9451e6c8c3b)
0641132人目の素数さん
垢版 |
2018/05/22(火) 12:22:24.03ID:yXdy01CV
>>604
>>625
>>636

A → -E3 = (0, 0, -1) の回転軸は、 法線が (A + E3) / 2 で 点 (A - E3) / 2 を通るような平面 P1 に含まれる。
B → -E2 = (0, -1, 0) の回転軸は、 法線が (B + E2) / 2 で 点 (B - E2) / 2 を通るような平面 P2 に含まれる。

明らかに、

L1 := P1 ∩ P2 を軸としてある角度だけ回転すれば、

A → -E3 = (0, 0, -1)
B → -E2 = (0, -1, 0)

とできる。
0642132人目の素数さん
垢版 |
2018/05/22(火) 12:26:52.38ID:yXdy01CV
A の L1 への射影を pr(A) とする。

明らかに、

arccos([(A - pr(A)) ・ (pr(A) + E3)] / [|A - pr(A)| * {pr(A) + E3}])

が求める回転角である。

向きも容易に求められる。
0643132人目の素数さん
垢版 |
2018/05/22(火) 12:29:12.40ID:yXdy01CV
訂正します:

A の L1 への射影を pr(A) とする。

明らかに、

arccos( [(A - pr(A)) ・ (-E3 - pr(A))] / [|A - pr(A)| * |-E3 - pr(A)|] )

が求める回転角である。

向きも容易に求められる。
0644132人目の素数さん
垢版 |
2018/05/22(火) 12:31:11.39ID:yXdy01CV
訂正します:

>>604
>>625
>>636

A → -E3 = (0, 0, -1) の回転軸は、 法線が A + E3 で 点 (A - E3) / 2 を通るような平面 P1 に含まれる。
B → -E2 = (0, -1, 0) の回転軸は、 法線が B + E2 で 点 (B - E2) / 2 を通るような平面 P2 に含まれる。

明らかに、

L1 := P1 ∩ P2 を軸としてある角度だけ回転すれば、

A → -E3 = (0, 0, -1)
B → -E2 = (0, -1, 0)

とできる。

A の L1 への射影を pr(A) とする。

明らかに、

arccos( [(A - pr(A)) ・ (-E3 - pr(A))] / [|A - pr(A)| * |-E3 - pr(A)|] )

が求める回転角である。

向きも容易に求められる。
0645132人目の素数さん
垢版 |
2018/05/22(火) 12:34:47.08ID:yXdy01CV
訂正します:

>>604
>>625
>>636

A → -E3 = (0, 0, -1) の回転軸は、 法線ベクトルが A + E3 で 点 (A - E3) / 2 を通るような平面 P1 に含まれる。
B → -E2 = (0, -1, 0) の回転軸は、 法線ベクトルが B + E2 で 点 (B - E2) / 2 を通るような平面 P2 に含まれる。

明らかに、

L1 := P1 ∩ P2 を軸としてある角度だけ回転すれば、

A → -E3 = (0, 0, -1)
B → -E2 = (0, -1, 0)

とできる。

A の L1 への射影を pr(A) とする。

明らかに、

arccos( [(A - pr(A)) ・ (-E3 - pr(A))] / [|A - pr(A)| * |-E3 - pr(A)|] )

が求める回転角である。

向きも容易に求められる。
0646132人目の素数さん
垢版 |
2018/05/22(火) 12:37:47.76ID:yXdy01CV
>>629

平面の方程式
内積
法線ベクトル

などを調べればいいのではないでしょうか?

CGの本を見れば、外積や四元数など色々載っているのではないかと推測します。
日本語の本でまともな本があるかどうかは知りませんが。
0647132人目の素数さん
垢版 |
2018/05/22(火) 12:41:19.78ID:yXdy01CV
>>645

|A| = |B| = 1 も仮定しています。
0648132人目の素数さん
垢版 |
2018/05/22(火) 14:46:03.62ID:Vg28Op4O
xy平面の単位円の周および内部を動く点(x,y)に対して
s=ax+by
t=cxy
を考える。
実数a,bが|a|≦1かつ|b|≦1かつ|c|≦1の範囲を変わるとき、(s,t)が動きうる領域がどのように変化するかを述べよ。
0649132人目の素数さん
垢版 |
2018/05/22(火) 18:53:56.41ID:Vg28Op4O
pを素数、kをp-1以下の正整数とする。
k個の二項係数
pC1,pC2,...,pCk-1,pCk
をすべて割り切る整数のうち、最大のものを求めよ。
0650132人目の素数さん
垢版 |
2018/05/22(火) 18:55:20.37ID:Vg28Op4O
xを正の実数とする。
1/xの小数部分がx/2に等しくなるようなxを求めよ。
0651132人目の素数さん
垢版 |
2018/05/22(火) 19:55:44.39ID:RuE2vaj6
>>650

[1/x] = m (整数) とおくと
1/x - m = x/2,

x = √(mm+2) - m のとき
 x > 0,
 m+1 > 1/x > 0,
 ∴ m ≧ 0

x = -√(mm+2) -m のときは
 x < 0,
 1/x = m + x/2 < m,
 ∴ 不適。
0653132人目の素数さん
垢版 |
2018/05/22(火) 22:14:57.70ID:jnbOMwuE
f(x,y)=0のとき(dy/dx)(dx/dy)=1
はよく知られていますが、
熱力学ではよく
f(x,y,z)=0のとき(∂y/∂x)(∂z/∂y)(∂x/∂z)=-1
という「オイラーの連鎖律」を使います。
例えばf(P,V,T)=(PV)/(nRT)-1=0で試してみると、確かに成り立っています。
この連鎖律が一般になりたつことは数学で証明出来るのでしょうか?
ネット上には非厳密な証明しかありません。
0654132人目の素数さん
垢版 |
2018/05/22(火) 22:26:24.93ID:50gg+9Qr
陰関数定理から
∂y/∂x=-fx/fy
∂z/∂y=-fy/fz
∂x/∂z=-fz/fx
掛け合わせればそうなりますね
0655132人目の素数さん
垢版 |
2018/05/22(火) 22:26:29.58ID:bBTBMYkh
>>645-647
>>629です。
ありがとうございます。自分でも計算出来そうです。
追加で伺って恐縮なのですが、
L1 := P1 ∩ P2 は常に原点を通る直線になりますでしょうか?原点は移動させずに回転させたいです。
また向きはどのように判断するのでしょうか?
無知で申し訳ありませんがご教示をお願いします。
0656132人目の素数さん
垢版 |
2018/05/22(火) 23:33:03.91ID:Vg28Op4O
二項係数についての式
{(n,i)・(n,j)}/(n,k)
が整数となるとき、i,j,kが満たす関係式を述べよ。

(注)(a,b)はaCbとも書く。
0658132人目の素数さん
垢版 |
2018/05/23(水) 09:29:50.69ID:5Vv/9fhG
>>655

P1 および P2 が原点を含むのでその共通部分である L1 も原点を含みます。
すなわち、 L1 は原点を通ります。

向きについては、

外積
3次の行列式
右ねじ

などをキーワードにして調べてください。
0659132人目の素数さん
垢版 |
2018/05/23(水) 09:44:22.76ID:5Vv/9fhG
>>645
なんか変なところがあるので訂正します:

>>604
>>625
>>636

A → -E3 = (0, 0, -1) の回転軸は、 法線ベクトルが A + E3 で 原点を通るような平面 P1 に含まれる。
B → -E2 = (0, -1, 0) の回転軸は、 法線ベクトルが B + E2 で 原点を通るような平面 P2 に含まれる。

明らかに、

L1 := P1 ∩ P2 を軸としてある角度だけ回転すれば、

A → -E3 = (0, 0, -1)
B → -E2 = (0, -1, 0)

とできる。

A の L1 への射影を pr(A) とする。

明らかに、

arccos( [(A - pr(A)) ・ (-E3 - pr(A))] / [|A - pr(A)| * |-E3 - pr(A)|] )

が求める回転角である。

向きも容易に求められる。
0660132人目の素数さん
垢版 |
2018/05/23(水) 09:47:41.89ID:5Vv/9fhG
訂正します:

>>655

P1 および P2 が原点を含むのでその共通部分である L1 も原点を含みます。
すなわち、 L1 は原点を通ります。

向きについては、

外積
平衡六面体の体積と3次の行列式
右ねじの進む向き
右手系、左手系

などをキーワードにして調べてください。
0661132人目の素数さん
垢版 |
2018/05/23(水) 09:48:13.64ID:5Vv/9fhG
訂正します:

>>655

P1 および P2 が原点を含むのでその共通部分である L1 も原点を含みます。
すなわち、 L1 は原点を通ります。

向きについては、

外積
平行六面体の体積と3次の行列式
右ねじの進む向き
右手系、左手系

などをキーワードにして調べてください。
0662132人目の素数さん
垢版 |
2018/05/23(水) 09:50:45.06ID:KyFiVz12
問A,B,Cの3問からなるテストがあり、配点は問Aが2点、問Bが3点、問Cが5点で10点満点である。
30人の生徒がこのテストを受けたところ、
問A,B,Cの正解者数は順に22人、18人、14人であった。
このとき、得点が5点であった者(AB2問のみの正解者またはC1問のみの正解者)の人数の最大値は
いくらか。

いろいろ当てはめながら調べると、例えば
 「AB2問のみ正解・・・16人、Cのみ正解・・・8人、AC2問のみ正解・・・4人、全問正解・・・2人」の場合
がその最大値を与える場合(つまり24人が答え)になりそうかな、と思ったのですが
ちゃんと解くにはどのように考えればよいでしょうか。
たぶん不等式に持ち込むのではないかと思うのですが難しいです。

よろしきお願いします。
0665132人目の素数さん
垢版 |
2018/05/23(水) 11:10:42.44ID:SaS67Pru
AD=BCよりBF=EA
AD//BCより∠BFG=∠EAH
定義より∠FBG=∠AEH
2角夾辺相等
0667132人目の素数さん
垢版 |
2018/05/23(水) 12:21:35.63ID:Waw1BERb
整数x,yが互いに素なときに整数a,bがあって
ax+by=1となるようにとれるというのがありますけど
1変数多項式f(x),g(x)がお互いを割り切れないときに
ある多項式a(x),b(x)があって
f(x)*a(x)+g(x)*b(x)=1となるようにとれるっていう命題は真ですか?

真ならどうやって証明できるかおしえていただけませんでしょうか
0668132人目の素数さん
垢版 |
2018/05/23(水) 12:26:16.26ID:Waw1BERb
>1変数多項式f(x),g(x)がお互いを割り切れないときに

ここ違いました。定数でない共通の多項式を約数に持たないとき、に変更してください
0669132人目の素数さん
垢版 |
2018/05/23(水) 12:35:31.02ID:pD12Z7zk
単項イデアル整域上で、f(x)とg(x)の最大公約元が1であれば成立する
0670132人目の素数さん
垢版 |
2018/05/23(水) 12:38:38.91ID:pD12Z7zk
R[x]が単項イデアル整域になることと、Rが体になることは同値だから体上か
0672132人目の素数さん
垢版 |
2018/05/23(水) 12:58:41.40ID:pD12Z7zk
整数環だとf(x)=x, g(x)=x+2とかが反例
0673132人目の素数さん
垢版 |
2018/05/23(水) 13:27:13.53ID:7JXq7gOx
すまん非常に簡単なのだろうが教えてもらえないだろうか
くだらない質問DAT落ちてたしここしかない

打率4割のバッターが5打席で2安打以上になる確率を求めよ
的な話を振られたんだが仕事に関係ないんだよこれ、4割あったら3本くらいうつやろ!

あとその確率分布に80%以上で収まるには5打席を1セットとして平均何回の試行が必要か?的な話だった
0674132人目の素数さん
垢版 |
2018/05/23(水) 13:45:32.78ID:sC+sxD3L
我輩は、炭鉱医である、真っ黒け。
0675132人目の素数さん
垢版 |
2018/05/23(水) 14:16:23.34ID:I5HdfiGT
>>673
> 打率4割のバッターが5打席で2安打以上になる確率を求めよ
> 的な話を振られたんだが仕事に関係ないんだよこれ、4割あったら3本くらいうつやろ!

確率を求めよなのに3本くらい打つやろ!っておかしくね?w
0677132人目の素数さん
垢版 |
2018/05/23(水) 14:25:16.46ID:SaS67Pru
1-(5C1)((2/5)^1)((3/5)^4)-(5C0)((2/5)^0)((3/5)^5)
=1-162/625-243/3125
=2072/3125
=0.66304
0679132人目の素数さん
垢版 |
2018/05/23(水) 17:56:49.12ID:bfNiVB5g
初歩的でもうしわけないが303の1を教えていただけませんか?
0683132人目の素数さん
垢版 |
2018/05/23(水) 19:47:07.81ID:TKWX+2XB
>>661
>655です。
大変助かりました。
後は自分で知識を補おうと思います。
ありがとうございましたm(_ _)m
0684132人目の素数さん
垢版 |
2018/05/23(水) 22:00:04.09ID:KJNnMDAS
>>662
その問題についてだけならCが不正解が16人でAが不正解が8人であることからわかる。


[ABC]+[AB]+[AC]+[A]+[BC]+[B]+[C]+[]=30。
[ABC]+[AB]+[AC]+[A]=22。
[ABC]+[AB]+[BC]+[B]=18。
[ABC]+[AC]+[BC]+[C]=14。
0≦[ABC],0≦[AB],0≦[AC],0≦[A],0≦[BC],0≦[B],0≦[C],0≦[]。
から一つずつ消去していくと
0≦[C]≦8。
0≦[AB]≦16。
2[C]≦[AB]+4。
2[AB]≦[C]+26。
0685132人目の素数さん
垢版 |
2018/05/23(水) 22:57:33.73ID:9h9uzlps
一辺の長さが1の立方体ABCD-PQRSにおいて、ABの中点をMとする。

(1)この立方体をRMの周りに一回転させてできる立体K1の体積を求めよ。

(2)この立方体をSMの周りに一回転させてできる立体をK2とする。K1とK2の共通部分の体積を求めよ。
0686132人目の素数さん
垢版 |
2018/05/24(木) 00:12:04.70ID:XIv78poq
>>681-682
ありがとうございます!
0687132人目の素数さん
垢版 |
2018/05/24(木) 01:05:44.90ID:zoTaEZJy
誰か>>583の面積が最小になる時は長方形になることの証明の部分教えてくれよ
0688132人目の素数さん
垢版 |
2018/05/24(木) 04:39:07.53ID:tZqrH1MY
大英博物館とNASAはどっちの方が価値がありますか?
0689132人目の素数さん
垢版 |
2018/05/24(木) 05:48:43.38ID:y9HXJqcS
無限群で任意の元の位数が有限となるものはありますか?
0691132人目の素数さん
垢版 |
2018/05/24(木) 06:52:59.54ID:iTcg9zRk
>>690
直積じゃ一般にはダメじゃん直和
0692132人目の素数さん
垢版 |
2018/05/24(木) 06:56:33.33ID:iTcg9zRk
F2[x]とかね
位数同じだから
F2[[x]]でもいいが
直積がダメで直和なら良い例は
ΠZn⊃⊕Zn
0694132人目の素数さん
垢版 |
2018/05/24(木) 07:10:14.98ID:T145GsN7
わからないんですね
0695132人目の素数さん
垢版 |
2018/05/24(木) 09:20:39.18ID:ycuQ9tnZ
無限巡回群とヤハウェはどっちの方が偉大ですか?
0697132人目の素数さん
垢版 |
2018/05/24(木) 13:16:31.84ID:o3kLnmYW
自分は尋常じゃないくらい頭が悪いのですが、東京大学理学部数学科に入るという夢があります。
猛烈に勉強をすれば可能性はありますか?
0698132人目の素数さん
垢版 |
2018/05/24(木) 13:40:27.29ID:f1FJL5tW
>>697
そんなことここで聞いてる暇があったら勉強しろ
お前の可能性はお前しか知らないんだから
0700132人目の素数さん
垢版 |
2018/05/24(木) 14:30:01.77ID:81MNfsiO
>>697
6年前から何か進展はありましたか?
きゃはははとか書き込んでこのスレを流した頃ですよ。
0701132人目の素数さん
垢版 |
2018/05/24(木) 14:35:42.96ID:c5tI+u1e
東大入れそうなガキとか実際芽が出て理学系に進学した連中に突っかかって時間を浪費させたいのが本音だろ
この人類の足手まとい嫉妬婆は。
0702132人目の素数さん
垢版 |
2018/05/24(木) 14:37:55.43ID:81MNfsiO
ヒマラヤは40代の長野在住のニートおっさん、劣等感婆とは区別しろよ
0703132人目の素数さん
垢版 |
2018/05/24(木) 14:39:37.90ID:EQ5K0CF7
なんと6年前から荒しを続けているとは!
0704132人目の素数さん
垢版 |
2018/05/24(木) 14:40:44.80ID:Is4CTbmF
言っとくけど俺40代でも長野在住でもねーよ。
0706132人目の素数さん
垢版 |
2018/05/24(木) 14:44:33.11ID:Is4CTbmF
50代でもねーよ。
0707132人目の素数さん
垢版 |
2018/05/24(木) 14:45:33.59ID:jsSCcTfs
「ニートおっさん」の部分は否定しないのか…
0708132人目の素数さん
垢版 |
2018/05/24(木) 14:48:14.93ID:Is4CTbmF
まぁ、でも俺の最大の夢は、東大理学部数学科に入ることではなくて、
無になってもう二度と有にならないことなんだ。
自殺をしても無にはなれないのかな?
それどころか、地獄に落ちるのかな?
0709132人目の素数さん
垢版 |
2018/05/24(木) 14:48:32.29ID:EQ5K0CF7
>>697 ぐぐったら2年前のが出てきた

https://tamae.5ch.net/test/read.cgi/shihou/1455701014/65
65 :氏名黙秘:2016/02/24(水) 14:31:46.85 ID:R4SfjPiz
東京大学理学部数学科に入りたいのですが、東大の理学部数学科は天才以外はやっていけないところなのでしょうか?
ちなみに自分は、尋常じゃないくらい頭が悪いです。
しかし、数学や物理学などに興味があります。
だから、東大の理学部数学科に入りたいのですが、やはり無理なんでしょうか?
0710132人目の素数さん
垢版 |
2018/05/24(木) 14:51:47.33ID:Is4CTbmF
どうすれば無になってもう二度と有にならなくて済むのか?
有は嫌だ。
0712132人目の素数さん
垢版 |
2018/05/24(木) 14:59:24.54ID:Is4CTbmF
もっともっと思索を続けて究極を見つけたいという気持ちも少しはあるが、もう難しい。
究極を見つけられなくても良いからとにかく無になってもう二度と有になりたくないという気持ちの方が強くなりつつある。
0713132人目の素数さん
垢版 |
2018/05/24(木) 15:01:17.97ID:81MNfsiO
7年前の物理板の書き込み

335 名前:ご冗談でしょう?名無しさん [sage]: 2012/01/12(木) 12:04:04.36 ID:???
日本の山でお願いします。
もの凄く雪深い山でお願いします。
0715132人目の素数さん
垢版 |
2018/05/24(木) 15:04:34.65ID:jsSCcTfs
5億年ボタンってあるじゃん
0716132人目の素数さん
垢版 |
2018/05/24(木) 15:05:02.54ID:Is4CTbmF
死後の世界とか生まれ変わりとかって本当にあるんでしょうか・・・?
0717132人目の素数さん
垢版 |
2018/05/24(木) 15:13:52.35ID:81MNfsiO
7年前にヒマラヤのスレが男女板に残っている。ヒマラヤの考えは興味深い(笑)

1 名前:名無しさん 〜君の性差〜[] 投稿日:2012/02/27(月) 18:51:07.40 ID:YQFwRny2 [1/2]

1.脳の左右子脳分業化の遅れによる論理的思考力、イメージ力の未発達。

2.化粧・香水・ハイヒール等外見をごまかす習慣によって作られたごまかす脳回路
  による根本的問題解決能力の欠如。
  
3.自分で自分に嘘をつく思考回路による真実を追究する脳回路の発達障害。

4.1〜3の総合的効果による客観的思考力の未発達。
  その結果、自分の願望と現実の区別が曖昧になる。

その他の多くの意見をどうぞ。
0718132人目の素数さん
垢版 |
2018/05/24(木) 15:15:57.14ID:Is4CTbmF
>>717
それ俺の書き込みじゃねーぞ。
0719132人目の素数さん
垢版 |
2018/05/24(木) 16:15:47.75ID:ToF5KKWY
正三角形△ABCの辺AC上に、AD:DC=1:3となる点Dをとります。
またBDをDの方向に延長し、BE=BAとなる点Eをとります。
△ABCと合同な△EFGを、FG⊥BE、BF<BEとなるようにつくるとき、△ABCの内部で△EFGの内部でもある部分の面積を求めなさい。
0721132人目の素数さん
垢版 |
2018/05/24(木) 17:13:52.03ID:c5tI+u1e
即身成仏させられてる最中の徳の高いお坊さんがお経を唱えてる限りはなんかまだ息がある証拠だと思って皆スルー推奨ってことだな。
まあこの荒らしは何の徳も感じさせないけど
0722132人目の素数さん
垢版 |
2018/05/24(木) 22:28:48.48ID:iTcg9zRk
>>702
何でそういう設定なん?
0723132人目の素数さん
垢版 |
2018/05/24(木) 22:40:08.77ID:XPKkhwEN
ヒットマークを探してるんでしょ
実際ニートでおっさんなのは確定みたいだし
0724132人目の素数さん
垢版 |
2018/05/24(木) 23:11:36.41ID:wK4NR8t+
120人を40人ずつ3学級に振り分けた中学校があるとします。
一年に一回クラス替えをするとして、以下の確率を教えてください。
@ある人Aが1〜3年の間に一回も同じ人と同じクラスにならない確率
Aある人Aと3年間同じクラスの人がいない確率
0725132人目の素数さん
垢版 |
2018/05/24(木) 23:54:54.74ID:XPKkhwEN
もっとはっきり書け

i

ある人Aが、1〜3年の間に一回もある人Bと同じクラスにならない確率?
(そこそこ高い)

ある人Aが、1〜3年の間に二回以上同じクラスになった人がいない確率?
(めっちゃ低い)

ii

ある人Aと3年間を通して一回以上同じクラスになった人がいない確率?
(0)

ある人Aと3年間三回同じクラスになった人がいない確率?
(かなり高い)
0726132人目の素数さん
垢版 |
2018/05/25(金) 01:03:57.09ID:b68wSvtw
俺から見て問題の趣旨は明らかだがな
はっきりも何も、問われてることは明記されてる
0727132人目の素数さん
垢版 |
2018/05/25(金) 03:31:04.32ID:FzI0O2aB
次の積分を求めよ
∫∫e^(x^3)dxdy
D={(x,y) : 0≦y≦1,√y≦x≦1}
お願いします
0728132人目の素数さん
垢版 |
2018/05/25(金) 03:38:51.14ID:/yyhiNqF
1辺の長さが1の立方体ABCD-EFGHがある。
線分GCの中点をI、線分BIを3:1に内分する点をJとする。線分AJと三角形BDEの交点をPとするとき、APベクトルをABベクトル、ADベクトル、AEベクトルの1次結合で表せ
https://i.imgur.com/duJBhkr.jpg
0729132人目の素数さん
垢版 |
2018/05/25(金) 06:51:18.84ID:ohjGIEVt
>>727
先にxで積分するのは大変なので、まずyで積分しよう。
 D = { (x,y):0≦x≦1,0≦y≦x^2 }
と表わして、
 ∫[0,x^2] e^(x^3) dy = e^(x^3)・x^2,
 (与式) = ∫[0,1] e^(x^3)・x^2 dx = [ (1/3)e^(x^3) ](x=0,1) = (e-1)/3,
0730132人目の素数さん
垢版 |
2018/05/25(金) 09:32:14.69ID:ohjGIEVt
>>719

直線BDE をx軸とする。

A (14L,(2√3)L)
B (0,0)
C (10L,-(6√3)L)
D (13L,0)
E (1,0)
F (1-(√3)/2,-1/2)
G (1-(√3)/2, 1/2)
L = 1/(4√13).
0732132人目の素数さん
垢版 |
2018/05/25(金) 09:58:35.44ID:ohjGIEVt
>>728

ABをx軸,ADをy軸,AEをz軸にとる。(デカルト座標)
A (0,0,0)
B (1,0,0)
C (1,1,0)
D (0,1,0)
E (0,0,1)
F (1,0,1)
G (1,1,1)
H (0,1,1)
I (1,1,1/2)
J (1,3/4,3/8)
P (x,y,z)
とする。
線分AJ  x:y:z = 8:6:3
△BDE  x+y+z = 1
より
P (8/17,6/17,3/17)
0733132人目の素数さん
垢版 |
2018/05/25(金) 12:30:31.14ID:Apkabz57
>>728
Vectors AB↑, AD↑, and AE↑ are linearly independent.

AJ↑=(3/4)AI↑+(1/4)AB↑=(3/4)((1/2)AC↑+(1/2)AG↑)+(1/4)AB↑=(3/4)((1/2)(AB↑+AD↑)+(1/2)(AB↑+AD↑+AE↑))+(1/4)AB↑=AB↑+(3/4)AD↑+(3/8)AE↑.

sAB↑+tAD↑+(1-s-t)AE↑=uAB↑+(3/4)uAD↑+(3/8)uAE↑
⇔s=u, t=(3/4)u, (1-s-t)=(3/8)u
⇔s=u=8/17, t=6/17.

AP↑=(8/17)AB↑+(6/17)AD↑+(3/17)AE↑.
0734132人目の素数さん
垢版 |
2018/05/25(金) 12:52:11.00ID:eFov7qL5
n個の物を一列に並べるパターンはn!通りというのは直感的には明らか(n個のものから1つ選んで、その後n-1個のものから1つ選んで.....を繰り返す)ですが、これはどのように数学的に正当化されているのですか?
そもそもn元集合からn元集合への全単射の個数をn!と定義しているのか、有限回の操作というのは何か公理的に特徴づけられているのか...
数学を真面目に取り組んだことが無いので変なことを言っているとは思いますが、回答よろしくおねがいします
0739132人目の素数さん
垢版 |
2018/05/25(金) 21:31:20.31ID:bFr/Rjwl
4つのドアがあります

それぞれのドアを開けると1または9の表記のあるプレートが1枚置かれているものとします
1のプレートは1枚
9のプレートは3枚 
あなたは4つのドアから一つを選択します
さてあなたがドアを選択した後に選択外のドアを開いたところ9のプレートがありました
あなたの選択したドアの向こうに1のプレートがある確率は変動していますか?

この問題なんですが、
4分の1のままですよね?
0740132人目の素数さん
垢版 |
2018/05/25(金) 21:34:14.77ID:ljSfkNMq
>>739
1/3です

1と9がそれぞれ1枚ずつの場合を考えてみましょう
もう一方が9だとわかった時点で自分のが1だということが確定しますね
0741132人目の素数さん
垢版 |
2018/05/25(金) 21:46:52.82ID:bFr/Rjwl
>>740
そうなりますか
ありがとうございます
0742132人目の素数さん
垢版 |
2018/05/25(金) 21:55:56.04ID:lc5apeh1
>>739
1/4のまま変わらない
仮に自分の選んだドアをAとし、それ以外のドアをBCDとする

選択外のDのドアを開けるという行為は

1)実際にDに1がある
2)実際にはDには1はない

この2つの分岐の判明過程にしかすぎんからな
確率は1/4

もしAのドア開けたあとBCDのドアをシャッフルするなら1/3
0743132人目の素数さん
垢版 |
2018/05/25(金) 22:04:50.58ID:fpgsNXPt
↑これが数学板の実力です
専門板なのに異常にレベルが低い
せいぜい数学の少しできる高校生レベル


ここの回答者って、レベル低いんですね
0744132人目の素数さん
垢版 |
2018/05/25(金) 22:05:00.39ID:lc5apeh1
ほんまかどうかはしらんのやが
うわさでは、数学板では、早稲田の問題
間違えてる答えのほうが「正しい」とする意見が主流になったらしい

さすが5ちゃん、アホばっかりwwwwwwwwwwwwwwwwwwwwww

ID:ljSfkNMqも自分が再抽選してることに気づいてないアホ
0745132人目の素数さん
垢版 |
2018/05/25(金) 22:07:32.53ID:fpgsNXPt
>>744
4つのドアがあります

それぞれのドアを開けると1または9の表記のあるプレートが1枚置かれているものとします
1のプレートは1枚
9のプレートは1枚 
あなたは2つのドアから一つを選択します
さてあなたがドアを選択した後に選択外のドアを開いたところ9のプレートがありました
あなたの選択したドアの向こうに1のプレートがある確率は変動していますか?
0746132人目の素数さん
垢版 |
2018/05/25(金) 22:07:53.92ID:fpgsNXPt
>>744
2つのドアがあります

それぞれのドアを開けると1または9の表記のあるプレートが1枚置かれているものとします
1のプレートは1枚
9のプレートは1枚 
あなたは2つのドアから一つを選択します
さてあなたがドアを選択した後に選択外のドアを開いたところ9のプレートがありました
あなたの選択したドアの向こうに1のプレートがある確率は変動していますか?
0747132人目の素数さん
垢版 |
2018/05/25(金) 22:09:05.68ID:lc5apeh1
52枚の正しいトランプを俺が一枚引く時
「俺が赤を引ける確率」は頻度主義によって26/52という数式によって1/2なんやが

俺が一枚引いたあと
「俺が赤を引けた確率」は1/2なんかに絶対ならんwwwwwwwwwwwwwww
100%か0%かやwwwwwwwwwwwwww

無限に繰り返す前提あるなら、両者はかぎりなく近づきはするんやが、そもそも
「俺が赤を引ける確率」と「俺が赤を引けた確率」は別種のもんやねん

多分数学板にそれを理解できてる奴はほとんどおらんと思うwwwwww
ほとんどが理学部数学科未満の
数1数2レベルの理解で確率を語ってる
0748132人目の素数さん
垢版 |
2018/05/25(金) 22:10:25.50ID:fpgsNXPt
>>747
2つのドアがあります

それぞれのドアを開けると1または9の表記のあるプレートが1枚置かれているものとします
1のプレートは1枚
9のプレートは1枚 
あなたは2つのドアから一つを選択します
さてあなたがドアを選択した後に選択外のドアを開いたところ9のプレートがありました
あなたの選択したドアの向こうに1のプレートがある確率は変動していますか?
0749132人目の素数さん
垢版 |
2018/05/25(金) 22:11:56.42ID:lc5apeh1
早稲田問題の解が10/49だと信じている知的障害猿が観察できるスレ

馴れ合い [無断転載禁止]c2ch.net
ttp://potato.2ch.net/test/read.cgi/mj/1483111206/
馴れ合い2 [無断転載禁止]c2ch.net
ttp://egg.2ch.net/test/read.cgi/mj/1485659365/
【麻雀に】何切る?【正解はない】 [転載禁止]c2ch.net
ttp://potato.2ch.net/test/read.cgi/mj/1438922977/
中森明菜11・30ロックアルバムDSも追加 [無断転載禁止]c2ch.net
ttps://potato.5ch.net/test/read.cgi/mj/1476413552/
0750132人目の素数さん
垢版 |
2018/05/25(金) 22:13:02.19ID:fpgsNXPt
>>749
一方が1で他方が9

一方が9だとわかったとき他はどっちになっている?

これをわからない、と答えるということはどういうことですか?

あなたがバカだということですよね
0751132人目の素数さん
垢版 |
2018/05/25(金) 22:13:47.68ID:lc5apeh1
>>751
続きは?
0753132人目の素数さん
垢版 |
2018/05/25(金) 22:18:22.10ID:6I7obK3m
常に9のプレートがあるドアを開けるので変わらない。
0754132人目の素数さん
垢版 |
2018/05/25(金) 22:18:48.32ID:fpgsNXPt
>>753
一方が1で他方が9

一方が9だとわかったとき他はどっちになっている?

これをわからない、と答えるということはどういうことですか?

あなたがバカだということですよね
0755132人目の素数さん
垢版 |
2018/05/25(金) 22:21:01.63ID:6I7obK3m
9のプレートは3枚
0756132人目の素数さん
垢版 |
2018/05/25(金) 22:22:15.45ID:fpgsNXPt
具体例を無視するんですね

9のドアを必ず開ける
しかし、その場合でも、その開けたドアに1がある場合が全体から除かれてるんですよ
だから母数が減るから確率が増えるんです
0757132人目の素数さん
垢版 |
2018/05/25(金) 22:27:56.44ID:6I7obK3m
常に1を開けずに9を開ければいい。
0758132人目の素数さん
垢版 |
2018/05/25(金) 22:31:07.42ID:lc5apeh1
1/4で当たるカードが外れた時
「目の前に一枚ある、ハズレと大きく書かれたカードは
ハズレなのかアタリなのか最抽選してみよう!!!」

ここに答えあるが
これだけコピペしても
猿には理解できんやろ???

俺は猿までいちいち相手する気ないで
0759132人目の素数さん
垢版 |
2018/05/25(金) 22:31:20.57ID:fpgsNXPt
開ける前
1999
9199
9919
9991

開けたあと
199(9)
919(9)
991(9)
999(1)

19(9)9
91(9)9
99(1)9
99(9)1

1(9)99
9(1)99
9(9)19
9(9)91

(1)999
(9)199
(9)919
(9)991

()のところのドアを開けるとしましょう

頭が悪いなら、具体的に書き出せばいいんです
0760132人目の素数さん
垢版 |
2018/05/25(金) 22:31:38.43ID:lc5apeh1
アホ猿は自分から「母数が減るから」とか言うてるなw
とど松再抽選
0761132人目の素数さん
垢版 |
2018/05/25(金) 22:32:16.78ID:fpgsNXPt
>>758
できませんね
ハズレはハズレでしょうからね
トンデモ猿の考えは全くわかりませんね
0763132人目の素数さん
垢版 |
2018/05/25(金) 22:34:40.64ID:lc5apeh1
>>759
それ懐かしいなwふいた
次は漸化式も出してくるやろうな
0764132人目の素数さん
垢版 |
2018/05/25(金) 22:35:44.35ID:fpgsNXPt
>>763
どこがおかしいのかいってみてください?

あと

一方が1で他方が9

一方が9だとわかったとき他はどっちになっている?

これをわからない、と答えるということはどういうことですか?

あなたがバカだということですよね

↑これにも一言お願いします
逃げないでくださいね
0765132人目の素数さん
垢版 |
2018/05/25(金) 22:36:16.69ID:6I7obK3m
1があるドアを開けないようにすればいい。
0766132人目の素数さん
垢版 |
2018/05/25(金) 22:38:03.88ID:lc5apeh1
>>764
逃げとんのどっちやねんw
これ以上ここでやると迷惑やろしこっち来いや

***何切る?統一スレッド 6***
http://egg.5ch.net/test/read.cgi/mj/1527211154/

俺は逃げも隠れもせんがお前が逃げるのは自由やw
0767132人目の素数さん
垢版 |
2018/05/25(金) 22:38:40.11ID:fpgsNXPt
>>765
開ける前
1999
9199
9919
9991

開けたあと
199(9)
919(9)
991(9)

19(9)9
91(9)9
99(9)1

1(9)99
9(9)19
9(9)91

(9)199
(9)919
(9)991

()のところのドアを開けるとしましょう
(1)は1があったので開けませんでしたから、今回の状況と合致しませんから、この表から覗きました

はい、めでたく確率が1/3になりましたね
0768132人目の素数さん
垢版 |
2018/05/25(金) 22:38:44.53ID:lc5apeh1
ノナメってコテ付けとるからすぐわかるやろ
まぁ逃げるんやろけどw
0770132人目の素数さん
垢版 |
2018/05/25(金) 22:40:28.89ID:R0Bevpsx
両方NGにしました
0771132人目の素数さん
垢版 |
2018/05/25(金) 22:41:18.19ID:R0Bevpsx
両方出てってくれ
0772132人目の素数さん
垢版 |
2018/05/25(金) 22:41:43.44ID:6I7obK3m
>>767
それ等確率じゃない。
0774132人目の素数さん
垢版 |
2018/05/25(金) 22:47:03.97ID:fpgsNXPt
まーじゃんの話しかしなくなったので帰ってきました
つまらないですね
0775132人目の素数さん
垢版 |
2018/05/25(金) 22:48:26.97ID:CY9jQQ5L
やっぱり逃げたなw
もし気がむいたら、時間かかると思うけど
上にリンク張ってる4スレ全部読んでみて
自分がいかに痴呆かわかると思うから
0776132人目の素数さん
垢版 |
2018/05/25(金) 22:49:18.10ID:fpgsNXPt
>>775
どうしてID変えたんですか?

あなたが逃げたんですよね?
私はあなたの問いに答えましたよ?
あなたの俺が赤を引けた確率云々の話は正しいです

なにが言いたいのかはっきりしてくださいね
0779132人目の素数さん
垢版 |
2018/05/25(金) 22:53:39.82ID:CY9jQQ5L
>>776
上の4スレ読んだ?
0781132人目の素数さん
垢版 |
2018/05/25(金) 22:56:23.49ID:b68wSvtw
数学で議論するだけなら勝手だが、罵り合うんだったら目障りだから消えろ
0782132人目の素数さん
垢版 |
2018/05/25(金) 23:18:27.40ID:6I7obK3m
最初に選んだドア,選んでないドア,選んでないドア,選んでないドア
1999 a
9199 b
9919 c
9991 d

1(9)99 e
19(9)9 f
199(9) g
91(9)9 h
919(9) i
9(9)19 j
991(9) k
9(9)91 l
99(9)1 m

e+f+g=a
h+i+j+k+l+m=b+c+d
だから変わらない。
0784132人目の素数さん
垢版 |
2018/05/25(金) 23:29:35.51ID:fpgsNXPt
一方が1で他方が9

一方が9だとわかったとき他はどっちになっている?

やっぱ、これでわからなきゃ、もうなのやったってダメですね
0785132人目の素数さん
垢版 |
2018/05/25(金) 23:43:09.29ID:YH4wabrP
>>735
読みましたが疑問は晴れませんでした
0786132人目の素数さん
垢版 |
2018/05/25(金) 23:48:15.72ID:6I7obK3m
a=e+f+g
b=h+i
だから
a=b>0
e=f=g=h=i
にはならない。
0788132人目の素数さん
垢版 |
2018/05/25(金) 23:55:47.67ID:ow7D2QsB
>>785
は?2個の置換はいくつ?3個は?
0789132人目の素数さん
垢版 |
2018/05/25(金) 23:57:48.94ID:YH4wabrP
>>788
それは分かりますがと書こうと思ったら数学的帰納法でn!になることが示せますね
ありがとうございます
0790132人目の素数さん
垢版 |
2018/05/26(土) 00:09:06.43ID:f9RTKmih
「群Gのある部分群Hが全てのGの部分群を含むとき、Gは巡回群で位数は素数のべき乗になることを示せ」
という問題で、Hに含まれないaでGは生成されるので巡回群になることは分かったのですがそこから進めません
1.aの位数が無限だと矛盾
2.aの位数をnとしたときnを割る素数が2つ存在したら矛盾
の2つを示せれば良いとは思うんですがヒントを下さい
0791132人目の素数さん
垢版 |
2018/05/26(土) 00:10:11.27ID:f9RTKmih
>>790
Hは真の部分群でGの全ての真の部分群を含む
です
0792132人目の素数さん
垢版 |
2018/05/26(土) 00:14:13.81ID:efhvyUI2
>>782
今思いましたけどこれ問題すり替わってますよね

モンティホールじゃないですよ
たまたま1がなかったんです
0795132人目の素数さん
垢版 |
2018/05/26(土) 00:35:28.15ID:d0tKLffX
>>790
>1.aの位数が無限だと矛盾
無限だとZと同型だから極大部分群がいくつもある
0796132人目の素数さん
垢版 |
2018/05/26(土) 00:39:20.19ID:d0tKLffX
>>790
>2.aの位数をnとしたときnを割る素数が2つ存在したら矛盾
Znm
(n,m)=1
とするとZnとZmと同型な部分群がありそれらを両方含む部分群の位数はnmの倍数だからZnm全体つまり真部分群ではない
0797132人目の素数さん
垢版 |
2018/05/26(土) 01:33:49.94ID:vnDRDQ0M
>>790
Hを極大部分群としてx∈G\HをとればG=<x>になる。
こいつの位数が有限で素数べきを言えば良い。
結局位数nが∞でもnを割り切る素数が2つある場合でも極大部分群が2つあることを示せば良い。
0798132人目の素数さん
垢版 |
2018/05/26(土) 01:49:06.66ID:Tm+bfCXy
>>790

2.
#G は有限として、素因数に分解する。
 #G = Σ[i=1,k] (p_i)^(e_i),  e_i≧1
Sylowの定理より、位数 (p^i)^(e^i) の部分群 h_i が存在。
k≧2 と仮定すると h_i ⊂ G    (真部分群)
>>791 により h_i ⊆ H
Lagrangeの定理より、 (p_i)^(e_i) = #(h_i)| #H
これがすべての i について成り立つから
 #G = Σ[i=1,k] (p_i)^(e_i) | #H,
一方、 >>791 より
 G ⊃ H    (真部分群)
 #G > #H
(矛盾)
0799132人目の素数さん
垢版 |
2018/05/26(土) 03:00:37.14ID:kUd1LxPB
一辺の長さが1の立方体ABCD-EFGHの面CDHGの内接円をKとする。
またC上に点Pをとり、直線APを軸にこの立方体を一回転させてできる立体をVpとする。
PがC上を一周するとき、空間内でVpに含まれうるを領域Vpcとする。点AとVpc上の点Qの距離をLaqとするとき、Laqの最大値を求めよ。
0800132人目の素数さん
垢版 |
2018/05/26(土) 03:34:11.62ID:rN3zALnO
問題よんで損した。
0801132人目の素数さん
垢版 |
2018/05/26(土) 04:12:30.94ID:P1CW6u7K
>>790
H は G の真の部分群だから、a∈G−H が取れる。<a> は G の部分群だから、
もしこれが真の部分群なら、<a>⊂H となって a∈G−H に矛盾する。
よって、G=<a> である。次に、異なる素数 p,q を任意に取る。<a^p>,<a^q> は
ともに G の部分群である。もし両方とも真の部分群なら、<a^p>,<a^q>⊂H となるので、
特に a^p,a^q∈H である。Hは群であるから、k,l∈Z に対して a^{pk+ql}∈H である。
gcd(p,q)=1 に注意して、ある k.l に対して pk+ql=1 なので、a^1∈H となり、
<a>⊂H となり、よって G⊂H となって矛盾する。よって、<a^p>=G または <a^q>=G が成り立つ … (1)
もし a^n=e なる n≧1 が存在しないなら、<a^p>≠G, <a^q>≠G となることが言えるので(1)に矛盾する。
よって、a^n=e なる n≧1 が存在する。そのような n のうち最小のものを再び n と置く。
このとき、G=<a> は位数nの巡回群である。n=p_1^{e_1}…p_m^{e_m} と素因数分解する。
もし m≧2 ならば、<a^{p_1}>≠G, <a^{p_2}>≠G となることが言えるので(1)に矛盾する。
よって m=1 であり、n=p_1^{e_1} となる。よって、G は位数が素数ベキの巡回群である。
0802132人目の素数さん
垢版 |
2018/05/26(土) 18:44:24.54ID:DXXWtdJl
260 ノナメ ◆fR1KiTvorM [] 2018/05/26(土) 18:37:08.45 ID:e5pwOtO8

野球板もプロ選手が書いてるんちゃうからな
バット20年以上握ったこともないトラキチとかカープファンや
数学板もそんなもんやでアホばっかり
0803132人目の素数さん
垢版 |
2018/05/26(土) 19:06:07.80ID:HUORxEdm
X={a,b,c}の時にXを覆う集合族はいくつあるか?
さらに、その中で分割はいくつあるか?

考え方がわかりません。
覆っているというものはつまり
2^Xからいくつかの要素を取り出した集合族Uがあり、
要素をu_iとしたとき、u_iの和集合がXになるもの全てということでしょうか?

また、分割というものは
{{a},{b},{c}}、{{a,b},{c}}(似たようなもの他2つ)
{{a,b,c}}の5つということになりますか?
回答お願いします
0805132人目の素数さん
垢版 |
2018/05/26(土) 19:30:17.72ID:f9RTKmih
790です
皆さんのおかげで解けました
ありがとうございました

もう一つ、aの位数が無限のとき、整数全体のなす加法群Zとの同型を考えればa^2が生成元にならないことは分かるのですが、同型を取らずに直接示すにはどのようにすればいいでしょうか
0806132人目の素数さん
垢版 |
2018/05/26(土) 22:10:52.49ID:p7ZlenKz
>>805
a^2が生成源ならa=(a^2)^nを満たす整数が取れるけど、それは位数有限に矛盾してるでヨサゲ
0807132人目の素数さん
垢版 |
2018/05/26(土) 22:15:02.65ID:p7ZlenKz
>>803
集合族全体は256個。
被覆がaを含まないのは16個
被覆がa,bを含まないのが4個
被覆が何にも含まないのが2個
0808132人目の素数さん
垢版 |
2018/05/26(土) 22:29:17.22ID:FBhGdsdh
>>806
位数無限に矛盾しているですね
ありがとうございます
0809132人目の素数さん
垢版 |
2018/05/27(日) 13:10:32.22ID:poggp8He
199 焼き鳥名無しさん sage 2018/05/14(月) 21:11:39.39 ID:x3xQuMQL
えっ、要するにこういうことか?

1p,9p,9p,9pの4枚をよく混ぜ伏せて並べる

A B C D

この初期状態の時、左端、Aの牌が1pである確率は1/4。
ここで、Aの牌をめくったら9pでした。

ノナメ理論だと、この時『Aが1pである確率』って1/4のままなの?


これも、これだけは、yesかnoかだけたのむわ


200 ノナメ ◆fR1KiTvorM 2018/05/14(月) 21:12:36.66 ID:Z8v6AYYw
いえす
0810132人目の素数さん
垢版 |
2018/05/27(日) 15:26:31.37ID:WlCVafzc
一辺の長さが2である立方体ABCD-EFGHのABの中点をMとする。
対角線BHを軸とする半径1の円柱をC_1、直線MGを軸とする半径1の円柱をC_2とするとき、以下の問いに答えよ。

(1)xyz空間の円柱x^2+y^2=1を、x軸を含みx軸と角θで交わる平面αθで切る。その切り口の面積をθで表せ。
ただしθはxy平面からz軸の正の方向に回転した角度とし、0≦θ<π/2とする。

(2)C_1とC_2の共通部分の体積を求めよ。
0811132人目の素数さん
垢版 |
2018/05/27(日) 16:06:06.42ID:s9yJF/4c
(1)は(2)のヒントになってるかなぁ?2軸を含む平面で切った菱形の面積積分する方が楽な希ガス。やる気ないのでどうでもいいけど。
0812132人目の素数さん
垢版 |
2018/05/27(日) 16:23:25.96ID:CGYiTgTM
>>807
>集合族全体は256個。
2^2^3
>被覆がaを含まないのは16個
2^2^2
>被覆がa,bを含まないのが4個
2^2^1
>被覆が何にも含まないのが2個
2^2^0
空集合と空集合を含む集合族は除かないの?
0813132人目の素数さん
垢版 |
2018/05/27(日) 16:24:59.33ID:CGYiTgTM
>>803
それでいいよ
ただ
>>807
のように空集合を含む集合族を許すのなら倍
0814132人目の素数さん
垢版 |
2018/05/27(日) 16:56:02.76ID:MiYdiExp
追加ですいません、
(P→Q)→(R→notS)
を連言標準形にせよ。という問題ですが、
これ、Fになるパターンが1つしかないので選言標準形の方が1項のみで、逆に連言標準形が15項の論理積になるかと思ったのですがあってますか?
0815132人目の素数さん
垢版 |
2018/05/27(日) 19:12:43.34ID:yiDHP8Qn
>>814
門外漢なのでよくわかんないけどwikiに書いてある事を信じると

not (P→Q)→(R→notS)
= ((not p) and r and s) or (q and r and s)

の否定だから2項の積になるのでわ?
0816132人目の素数さん
垢版 |
2018/05/27(日) 19:28:14.78ID:DWIbh3ID
>>814
一般に4変数の標準形で15個も節は要らない
(P→Q)→(R→¬S)
=(P∧¬Q)∨(¬R)∨(¬S)
=(P∨¬R∨¬S)∧(¬Q∨¬R∨¬S)
0817132人目の素数さん
垢版 |
2018/05/27(日) 19:30:31.02ID:MiYdiExp
>>815
連言なので∧を使って繋げるんですよね
いろいろ考えたんですけどやっぱり選言標準形だとすっきり表せて、連言なら15通り出ると思いました…どうなんでしょう
0818132人目の素数さん
垢版 |
2018/05/27(日) 19:32:14.36ID:MiYdiExp
>>816
あ、そうかそういうことですか…
ちょっと勘違いしてたみたいですすいません
どうもありがとうございます
0819132人目の素数さん
垢版 |
2018/05/27(日) 19:35:02.48ID:WlCVafzc
(1)自然数nに対してn^2+1が10の倍数になるとき、nはどのような数かを述べよ。

(2)kを2でない自然数とする。n^2+1とn^k+1をともに10の倍数とするようなnが存在するとき、kはどのような数か。
0820132人目の素数さん
垢版 |
2018/05/27(日) 19:39:12.47ID:DWIbh3ID
>>817
Fが1通りの場合はむしろ簡単で、
例えばP∨Q∨R∨Sは選言標準形であり連言標準形でもある
0821132人目の素数さん
垢版 |
2018/05/27(日) 22:07:49.48ID:EdUtsj53
(1) mod 10でn^2≡9⇔n≡3,7(⇔n≡±3)
(2) kは非負整数
mod 10で
n≡3,7のときn^4≡1より
n^(4k+0)=((n^4)^k)*(n^0)≡1≡1
n^(4k+1)=((n^4)^k)*(n^1)≡n≡3,7
n^(4k+2)=((n^4)^k)*(n^2)≡n^2≡9
n^(4k+3)=((n^4)^k)*(n^3)≡n^3≡7,3

1の位に相当
0822132人目の素数さん
垢版 |
2018/05/27(日) 22:09:57.61ID:EdUtsj53
同じkを使ってしまったが察して
0823132人目の素数さん
垢版 |
2018/05/28(月) 00:30:02.72ID:35mGdcfM
ひてい
0824132人目の素数さん
垢版 |
2018/05/28(月) 00:46:55.38ID:35mGdcfM
ミスしました

述語論理についてなんですけど、
∀x Ey P(x,y)
とすると、すべてのxについてyが存在するかどうかについて考えるのが良いのですか?

例えば、P(x,y,z)がx+y=zだとして

∀x∈N , Ey∈N P(x,y,0)
の場合は、すべての自然数xに自然数yを足して0になるyが存在するかどうかを考えれば良いのでしょうか?
この場合だと、不可能?偽?どのように答えるのが良いのでしょうか?
0825132人目の素数さん
垢版 |
2018/05/28(月) 00:48:57.32ID:W2N/CyZK
どんなxを選んでも、y=-xと選べばx+y=0となるので、その命題は正しい命題ですね
0826132人目の素数さん
垢版 |
2018/05/28(月) 01:01:37.62ID:AujL21eY
一般に-xは自然数ではないので偽
0828132人目の素数さん
垢版 |
2018/05/28(月) 01:34:23.45ID:BgTET7DV
a,b,cを正の実数とするとき、以下のA,Bの最小値を求めよ。
A={a/(b+c)}+{b/(c+a)}+{c/(a+b)}
A=ln{a/(b+c)}+ln{b/(c+a)}+ln{c/(a+b)}
0829132人目の素数さん
垢版 |
2018/05/28(月) 01:54:37.16ID:1VyhgqB1
Bはないな。あってもないけどww
0830132人目の素数さん
垢版 |
2018/05/28(月) 06:32:59.95ID:TTo2rnUU
>>728

A = (1/2){(2a+b+c)/(b+c) + (a+2b+c)/(c+a) + (a+b+2c)/(a+b) -3}
 = {(a+b)/(b+c)+(b+c)/(a+b)}/2 + {(b+c)/(c+a)+(c+a)/(b+c)}/2 + {(c+a)/(a+b)+(a+b)/(c+a)}/2 - 3/2
 ≧ 1 + 1 + 1 - 3/2
 = 3/2.
(*) x>0 ⇒ x + 1/x ≧ 2 を使った。

A = (a+b+c){1/(b+c) + 1/(c+a) + 1/(a+b)} - 3
 = {(b+c) + (c+a) + (a+b)}{1/(b+c) + 1/(c+a) + 1/(a+b)}/2 -3
 ≧ (3^2)/2 - 3   (←チェビシェフ or コーシー)
 = 3/2.

e^B = abc/{(b+c)(c+a)(a+b)}
 ≦ abc/{(2√bc)(2√ca)(2√ab)}
 = abc/(8abc)
 = 1/8,

∵ (b+c)(c+a)(a+b) - 8abc = a(b-c)^2 + b(c-a)^2 + c(a-b)^2 ≧ 0.
0831132人目の素数さん
垢版 |
2018/05/28(月) 15:58:14.44ID:7L/Au6Pq
たとえばフェルマー予想とかの
具体的な数の問題が数学を抽象化することで解けるようになるのってなんでなんですか?
具体的な方法でも解けるけど記述が長くなりすぎるから抽象的な記述をしているだけなのか
それとも抽象化以外の解法がないのか、どっちなんでしょうか?
0832132人目の素数さん
垢版 |
2018/05/28(月) 18:53:36.94ID:1GO2+eBu
>>831
>具体的な数の問題が数学を抽象化することで解けるようになるのってなんでなんですか?
0833132人目の素数さん
垢版 |
2018/05/28(月) 19:12:36.08ID:35mGdcfM
有向グラフで二項関係Rを考え、反射性、対称性、反対称性、推移性を答える問題

反射性とは、いわば自分への辺なのはわかります。
推移性に関しては、1→2, 2→3 があったら1→3もある、というようになっているかどうかもわかります。
対称性、および反対称性についてが不明です。

対称性とは、有向グラフでいうとどの部分を表しているのでしょうか?
1→2,2→1のようになっていることでしょうか?
そもそも、対称性と反対称性の有向グラフでの違いはどう考えるのでしょうか
0834132人目の素数さん
垢版 |
2018/05/28(月) 19:15:23.87ID:GtBRvjFW
問題による
0835132人目の素数さん
垢版 |
2018/05/28(月) 21:42:31.68ID:1GO2+eBu
>>833
反対称性って何だっけ?
a≦b∧a≧b→a=b
のこと?
0836132人目の素数さん
垢版 |
2018/05/28(月) 23:04:25.69ID:3gPI6er1
鎌倉の大仏の知能は、圧倒的世界一の超絶天才数学者をも凌駕しているのでしょうか?
0837132人目の素数さん
垢版 |
2018/05/28(月) 23:29:01.34ID:3gPI6er1
秘密曼陀羅十住心論を書いた空海は、東大理V首席よりも賢いですか?
0838132人目の素数さん
垢版 |
2018/05/29(火) 01:29:02.66ID:vRlAKW/L
>>835
それです、
対称性については
例えばノードaからbへの辺があるならb→aの辺があること。
つまるところ、関係行列としては対角成分を軸に対称的な位置の要素が1になってることなのはわかったんですけど、
反対称性についてが未だに理解出来てません…
0839132人目の素数さん
垢版 |
2018/05/29(火) 01:30:12.90ID:ZAB9lIJR
>>831
抽象化することによって見えていなかった性質が見えてきたり、既に研究が進んでいる他の分野を応用できたりするため
抽象化というのはある意味で本質や実体を捉えるためのステップ
0840132人目の素数さん
垢版 |
2018/05/29(火) 01:52:54.81ID:qEPn7ntH
>>838
ノードaからbへ、行って来いができたと思ったが、そんなことはなかったぜ
な、何のことを言っているのかわからないと思うが、俺も最初分からなかった
実際には俺は一歩も動いていなかったんだ

ってことだろ?
0841132人目の素数さん
垢版 |
2018/05/29(火) 02:32:19.23ID:cuIjcuNH
【ホリエモン】なんでみんな就職するの?やる気がない人ほど起業して利益率の高い仕事を選択し、有望な者に投資しろ
https://www.youtube.com/watch?v=y3WFObrOIoQ
ホリエモンのQ&A vol.155起業のすすめ
https://www.youtube.com/watch?v=2n1O4oUeIXg
堀江貴文「大企業に就職なんて、とっくにオワコン」「今の時代、金ですらオワコン」
https://www.youtube.com/watch?v=gSvIk_Bnwlo
堀江貴文の名言がすごい!「つまらない仕事なんか今すぐ辞めろ!楽しいことだけやれ!」
https://www.youtube.com/watch?v=4w3XOl5CoU8
堀江貴文 決められたレールの上を歩く⇒人生終了で、自殺者増える
https://www.youtube.com/watch?v=CYRo8o2Y_D8
【堀江貴文】※サラリーマン必見!君らいい加減仕事辞めたら?wはっきり言って全部無駄だ!!
https://www.youtube.com/watch?v=IgyRIVdvxhk
これからは個人の時代!ヒカルは話が上手いしヒカキンは編集が上手い。
これからの通貨の未来はどうなるのかも話そう
https://www.youtube.com/watch?v=4hQngvBCugA
個人が大金を稼ぐ!ライブ配信時代が本格的にやって来てその領域は
さらに拡大していき無名から著名になる人も増加する
https://www.youtube.com/watch?v=1H0R-kBtUOo
0842132人目の素数さん
垢版 |
2018/05/29(火) 06:18:06.18ID:U0bqQoKK
>>838
じゃ
異なるabの間にa→bがあるならb→aはないってことだよ
0843132人目の素数さん
垢版 |
2018/05/29(火) 07:20:01.38ID:vRlAKW/L
対称性の反対というか否定を考えるのか
0844132人目の素数さん
垢版 |
2018/05/29(火) 13:54:48.07ID:tflU+QS4
fをXからYの写像、ψをXの冪集合からYの冪集合への写像としたとき、
fが単射であることと、ψが単射であることが同値であることを示せ
という問題です。

fが単射→ψが単射
とその逆のψが単射→fが単射
の両方が成り立つことを示すのがわかりますが、
仮にfが単射→ψが単射とはどうやって示すことが出来るのでしょうか?

そもそも、単射を示すということがわかっていませんが、ここではfが単射であることは前提として、そこからψが単射であることを導くのでしょうか?
(もっと言うと、P→Qを示すと言うのは、
P⊆Q とその逆を示すことなので、4パターン示さなければならないという考えであっていますか?)

何が分からないのか分かってないのでどうか解説か、回答だけでもいいのでよろしくお願い致します。
0845132人目の素数さん
垢版 |
2018/05/29(火) 14:02:09.98ID:J8fJm/T7
示せるわけない。
0848132人目の素数さん
垢版 |
2018/05/29(火) 14:51:59.78ID:+BWlXE/G
fが単射なら f^(-1)(f(A)) = A
0853132人目の素数さん
垢版 |
2018/05/29(火) 16:40:19.13ID:LrJ8VHO5
すいませんそれすら分かってないです。
ちょっと写像の根本から勉強しないとですね。。。

写像に触れないまま授業進んでしまってるので(泣)
0854132人目の素数さん
垢版 |
2018/05/29(火) 16:49:08.48ID:h5GyrdT7
写像なんて、関数じゃん。対象が数以外のものでも構わないだけ
0856132人目の素数さん
垢版 |
2018/05/29(火) 17:32:16.81ID:6ltIIHde
>>844
写像を学習したら、
"ψ" をどのように定義すれば >>851 記述の問題の趣旨に合致するかを考えてみよう。
0857132人目の素数さん
垢版 |
2018/05/29(火) 17:58:05.46ID:TSoO8D7M
以下の条件(1)(2)を満たすxの関数f(x)の例を1つ挙げ、それが条件を満たしていることを説明せよ。
(1)-∞<x<∞で何回でも微分可能である
(2)xy平面の曲線y=f(x)はちょうど3つの異なる変曲点を持ち、それらは同一直線上にある
0859132人目の素数さん
垢版 |
2018/05/29(火) 20:55:51.74ID:8U3hRUxB
線形の本質ってなんですか?
グラフが直線になることですか?
交換法則や結合法則が成り立つことですか?
0861132人目の素数さん
垢版 |
2018/05/30(水) 00:12:38.30ID:nZstFZFd
直線代数
0862132人目の素数さん
垢版 |
2018/05/30(水) 02:41:43.36ID:LUI2iFYL
なぜか観てしまう!!サバイバル系youtuberまとめ
http://tokyohitori.hatenablog.com/entry/2016/10/01/102830
あのPewDiePieがついに、初心YouTuber向けに「視聴回数」「チャンネル登録者数」を増やすコツを公開!
http://naototube.com/2017/08/14/for-new-youtubers/
27歳で年収8億円 女性ユーチューバー「リリー・シン」の生き方
https://headlines.yahoo.co.jp/article?a=20170802-00017174-forbes-bus_all
1年で何十億円も稼ぐ高収入ユーチューバー世界ランキングトップ10
https://gigazine.net/news/20151016-highest-paid-youtuber-2015/
おもちゃのレビューで年間12億円! 今、話題のYouTuberは6歳の男の子
https://www.businessinsider.jp/post-108355
彼女はいかにして750万人のファンがいるYouTubeスターとなったのか?
https://www.businessinsider.jp/post-242
1億円稼ぐ9歳のYouTuberがすごすぎる……アメリカで話題のEvanTubeHD
https://weekly.ascii.jp/elem/000/000/305/305548/
専業YouTuberがYouTubeでの稼ぎ方具体的に教えます。ネタ切れしない方法は〇〇するだけ。
https://www.youtube.com/watch?v=Co9a9fHfReo
YouTubeで稼げるジャンルは〇〇動画です。YouTube講座
https://www.youtube.com/watch?v=_Nps8xb5czQ
富を築くYouTuber年収7億円〜15億円稼ぐツワモノも!
https://www.kaigainet.com/youtube10.html
月に推定26億円稼ぐ人も 低予算の動画で驚くほどの金額を手にする「ユーチューバー」トップ10
https://www.huffingtonpost.jp/2015/02/09/youtube-stars-huge-earnings_n_6642684.html
0863132人目の素数さん
垢版 |
2018/05/30(水) 13:15:58.94ID:My3C3KVt
If a baseball and a bat cost $1.10 together,
and the bat costs $1.00 more than the ball,
how much does the ball cost?

WRONG ANSWXER = 10&cent;
CORRECT ANSWER = 5&cent;

英語力の無さもあってどうして5セントになるか理由がわかりません。
よろしくお願いします。
0864132人目の素数さん
垢版 |
2018/05/30(水) 13:18:20.51ID:BuOX6i6+
野球ボールとバット合わせて1.10ドル、バットはボールより1ドル高い
ボールはいくら?

間違い 1.10-1=0.10
正しい0.05+1.05=1.1
0865132人目の素数さん
垢版 |
2018/05/30(水) 15:36:48.22ID:/iCOYSYN
バットにはイチローのサイン、ボールにはダルのサインがありました
0866132人目の素数さん
垢版 |
2018/05/30(水) 16:24:40.16ID:Zov7LODs
殺す殺す殺す殺す殺す殺す殺す殺す殺す殺す殺す殺す殺す殺す
殺す殺す殺す殺す殺す殺す殺す殺す殺す殺す殺す殺す殺す殺す
殺す殺す殺す殺す殺す殺す殺す殺す殺す殺す殺す殺す殺す殺す
殺す殺す殺す殺す殺す殺す殺す殺す殺す殺す殺す殺す殺す殺す 👀
Rock54: Caution(BBR-MD5:1341adc37120578f18dba9451e6c8c3b)
0867132人目の素数さん
垢版 |
2018/05/30(水) 23:46:20.49ID:tB7oIF94
∫∫∫ (x^2) / (x^2+y^2+z^2+1)^3
という積分が分かりません。教えてください。
0868132人目の素数さん
垢版 |
2018/05/30(水) 23:53:34.89ID:DRfOW4UB
>>867
>∫∫∫ (x^2) / (x^2+y^2+z^2+1)^3
∫∫∫ (x^2+y^2+z^2) / (x^2+y^2+z^2+1)^3 /3
0869132人目の素数さん
垢版 |
2018/05/31(木) 05:07:35.91ID:WOMjP57J
y=e^xの0≦x≦log2の部分の長さを求めよという問題が分かりません。
∫√(1+e^2x)dxなんて計算できるんですか?
0870132人目の素数さん
垢版 |
2018/05/31(木) 06:04:27.65ID:r643jL3Z
逆関数
0871132人目の素数さん
垢版 |
2018/05/31(木) 06:56:43.58ID:YExPTj9n
>>869
t=√(1+e^(2x))と置換
x=(1/2)log(t^2-1)
dx=t/(t^2-1)dt
0872132人目の素数さん
垢版 |
2018/05/31(木) 10:25:15.29ID:1i3xzGBS
>>867

D(R) = { (x,y,z) | xx+yy+zz ≦ RR } とする。
>>868 に従い
∫∫∫_D xx/(xx+yy+zz+1)^3 dxdydz
= (1/3)∫∫∫_D (xx+yy+zz)/(xx+yy+zz+1)^3 dxdydz
= (1/3) ∫[0,R] rr /(rr+1)^3 (4πrr)dr   (← 極座標)
= (π/6) ∫[0,R] 8(r^4)/(rr+1)^3 dr
= (π/6) [ 3arctan(r) - r(5rr+3)/(rr+1)^2 ](r=0,R)
= (π/6) { 3arctan(R) - R(5RR+3)/(RR+1)^2 }
→ (π/2)^2   (R→∞)
0873132人目の素数さん
垢版 |
2018/05/31(木) 11:41:02.86ID:1i3xzGBS
>>869

√{1+e^(2x)} = e^(2x)/√{1+e^(2x)} + 1/√{1+e^(2x)},

∴∫√{1+e^(2x)} dx = √{1+e^(2x)} + ∫1/√{1+e^(2x)} dx

>>871 を使って

(右辺第2項) = ∫1/√{1+e^(2x)} dx
 = ∫ 1/(tt-1)dt
 = (1/2)∫{1/(t-1) - 1/(t+1)} dt
 = (1/2)log{(t-1)/(t+1)}
 = …
0874132人目の素数さん
垢版 |
2018/05/31(木) 12:40:06.52ID:1i3xzGBS
>>869

>>871 を使って
= ∫[√2,√5] tt/(tt-1) dt
= ∫[√2,√5] { 1 + (1/2)[1/(1-t) - 1/(1+t)] } dt
= [ t + (1/2)log((t-1)/(t+1)) ](t=√2,√5)
= {√5 - log((√5 +1)/2)} - {√2 - log(√2 +1)}
= 1.222016177
0875132人目の素数さん
垢版 |
2018/05/31(木) 13:15:23.81ID:1i3xzGBS
>>857
 5次関数で可能…
 f(x) = x^5 -(10/3)aax^3 +bx +c,
 f "(x) = 20x(x+a)(x-a)
 変曲点
 (-a,(7/3)a^5 -ab+c)
 (0,c)
 (a,-(7/3)a^5 +ab+c)
0878132人目の素数さん
垢版 |
2018/05/31(木) 21:58:32.01ID:lSPNDXw4
無限級数Σ(((n!)^2)a^n)/(2n)!) (0<a)についての質問です
収束判定法でa<4と4<aの時で収束発散が変わることは分かったのですが
a=4の時はどう判定すればいいのでしょうか?
0879132人目の素数さん
垢版 |
2018/05/31(木) 22:08:06.84ID:r643jL3Z
収束半径ゼロだろ
0880132人目の素数さん
垢版 |
2018/05/31(木) 22:11:52.49ID:Mw5UFFh6
>>878
スターリングの公式を使うと、0<a<4のとき収束し、a≧4 のとき発散することが分かる。
0881132人目の素数さん
垢版 |
2018/05/31(木) 22:36:42.69ID:aZkDH4FO
まあa=4のときは
第n+1項=第n項×(2n+2)/(2n+1)
だから明らかに発散するけどね。
でもスターリングの公式使うのが本格的。
0885132人目の素数さん
垢版 |
2018/06/01(金) 16:56:00.64ID:Msi5d0uy
中心極限定理では、元の分布によって収束する速さが異なると思いますが、キュミュラントの視点から何が言及できますか?
0886132人目の素数さん
垢版 |
2018/06/01(金) 20:17:42.63ID:gsMzIl4U
1と書かれたカードがn枚、2と書かれたカードが2n枚、3と書かれたカードが3n枚、4と書かれたカードが4n枚、合計10n枚のカードがある。
この中から無作為に3n枚のカードを選び、それらを並べて3n桁の整数Nを作る。
Nが3の倍数となる確率と1/3の大小を比較せよ。
0887132人目の素数さん
垢版 |
2018/06/02(土) 06:55:32.44ID:qc99k5Fr
>>886

N ≡ (各桁の数字の和) (mod 9)

1または4のカード j枚
2 のカード k枚
3 のカード (3n-j-k)枚
のとき
 N ≡ j + 2k (mod 3)
0889132人目の素数さん
垢版 |
2018/06/02(土) 11:26:48.91ID:SKZf7qZs
朗報、誤答爺さんが2チャンやめるって

75 名前:132人目の素数さん[sage] 投稿日:2018/06/01(金) 10:46:59.65 ID:wHdQHx/t [3/3]
(>>74の続き)
以後、私は2チャンには書かないことにする。今回はしっかりと明記する。
2チャンでゴタゴタさせられ濡れ衣を着せられたりして、巻き込まれるのが嫌になった。
2チャンでは、背理法の原理を教わったことが唯一の救いだ。
0891132人目の素数さん
垢版 |
2018/06/02(土) 14:18:00.23ID:fg2B06o8
これこういうことですか?
https://imgur.com/a/QPqpfS7
θが一致するからこんな式になるだけで
点oに関係ないθってありえないのかな
左の三角形のcos、y/rかと思った
0893132人目の素数さん
垢版 |
2018/06/02(土) 14:52:52.79ID:fg2B06o8
>>892
えっち
0896132人目の素数さん
垢版 |
2018/06/02(土) 17:03:04.97ID:fg2B06o8
回答しねえなら意味不明な動画載せんなよ!!!!!
0898132人目の素数さん
垢版 |
2018/06/02(土) 21:07:36.50ID:mWY0yoZc
9で割り切れる数は各桁の和が9の倍数
3で割り切れる数は各桁の和が3の倍数
とわかりました。

証明は出来ないのですが、11で割り切れるかどうかの判定は、
例えば、整数nがK桁だとして、順に桁を並べ2桁ずつで11の倍数を引いていき、最終的に0になるってことで合ってますか?

つまり、
121なら12-11=1.11-11=0よって割り切れる

123456784なら12-11=1、13-11=2、24-22=2、25-22=3、36-33=3、37-33=4、48-44=4、44-44=0
よって、割り切れる。
(しかも、11×整数を並べると1122,33,44で商が作れると思います)
これを一般化したものをご存知でしたら名前を教えていただけたら助かります。
0903132人目の素数さん
垢版 |
2018/06/02(土) 22:18:05.59ID:iniu0Mjz
k上のベクトル空間Vでn次元のものというのは、ただk上のベクトル空間k^nの標準基底でない基底を採用したものという認識でオッケーですか?
0904132人目の素数さん
垢版 |
2018/06/02(土) 22:19:52.79ID:Iwqbrmxy
k^nでもいいしそうでなくてもいい
標準基底でもいいし別の基底でもいい

何故変な仮定をつけようとするのか
0905132人目の素数さん
垢版 |
2018/06/02(土) 22:28:24.23ID:hqzOfJsV
k^nには標準基底を入れて考えていると思っていたのですがそうではないんですか?
だとしたらわざわざVをk^nと書いているときはなぜ...(もちろん集合としての違うはあるのは知ってますが)
0906132人目の素数さん
垢版 |
2018/06/02(土) 22:33:16.22ID:Qe1TBQQS
体k上のベクトル空間Vには底が存在し、その底が有限個のとき、
どの底も同じ個数からなり、一つ底をさだめるごとに、その底を用いて
Vとk^nの同型な線形写像を構成することができる。
0907132人目の素数さん
垢版 |
2018/06/02(土) 22:51:00.62ID:hqzOfJsV
有限次元k-ベクトル空間の同型類は次元によって完全に決定される
しかしその同型の決め方はcanonicalじゃなく基底を定めないといけないからそれぞれk^nとかVとか(基底を込めて)違う記号で書いてると思ったのですが違う??
0908132人目の素数さん
垢版 |
2018/06/03(日) 00:49:28.79ID:LciQfjpc
>>899
計算機で計算したら1/3よりでかいときと小さい時が4周期前後で交代にでてくるけど、ぴったり4周期でもなく結構入り乱れてる。持ってる答えほんとに合ってる?
0910132人目の素数さん
垢版 |
2018/06/03(日) 20:37:41.75ID:FMlwT4S5
>>907
「Vは(有限次元)ベクトル空間」と言えばそれ以上の意味はない
Vが集合として何であるかは決めてないし、もちろん基底も固定していない

もう一度言うが、何で変な仮定をつけようとするの?
V≠(k^n,標準基底)とするなら具体例としてV=(k^n,標準基底)を考えることができなくなるけど、それでもいいの?
0911132人目の素数さん
垢版 |
2018/06/03(日) 21:06:12.81ID:GUNpMLhn
>>659の人かわかる人いますかー?
この方法でなんで求まるかが自分の中で理解出来ていません。
教えるの上手い人教えてください
0913132人目の素数さん
垢版 |
2018/06/03(日) 22:42:51.58ID:uThlUoHH
>>910
ここでのVは一般のVというわけではなく、自分が書いた意味での(k^n,標準基底)とは別の任意のn次元ベクトル空間(たとえば(k^n,標準基底じゃないやつ)とか)というつもりで書いてました

ええと、では係数体が固定されていて次元も確定(有限次元)している場合、そのベクトル空間をわざわざVと書く理由がわかりません(k^nしかないのに)
0914132人目の素数さん
垢版 |
2018/06/03(日) 22:46:08.33ID:8QbmVSQ9
>>912
数学用語での「または」には排他的な意味はありません。
A「または」B は  「A「かつ」B」も含みます。
つまり、A「または」B は AとBの少なくとも一方は、と同じ意味です。
0915132人目の素数さん
垢版 |
2018/06/03(日) 22:59:12.52ID:8QbmVSQ9
>>913
k^n は特に数ベクトル空間などとよばれる体k上のn次元ベクトル空間の具体例の一つです。
数ベクトル空間ではないn次元ベクトル空間としてよく現れるものに、
不定元xに関する体k上のn-1次以下の多項式全体のなす集合に通常の多項式の和の構造をい入れたものがあります。
当然ながらこれはk^nに線形同型であり、記号的には k[x](<n)などと書かれることもありますが、k^nではありません。
0916132人目の素数さん
垢版 |
2018/06/03(日) 22:59:34.70ID:SbqQM1O0
>>914
ああ、両方なんですか

できれば>>909もお願いします
0917132人目の素数さん
垢版 |
2018/06/04(月) 01:22:27.31ID:eLfureAF
>>866 >>897
j:1〜100での答え。出題者の持ってる答えと合ってる?なんか規則ありそうでなさそうで。
ホントに出題ミスじゃないの?そうじゃないなら考えてみるけど。

(1,LT),(2,LT),(3,GT),(4,GT),(5,GT),(6,GT),(7,LT),(8,LT),(9,LT),(10,LT),
(11,GT),(12,GT),(13,GT),(14,GT),(15,LT),(16,LT),(17,LT),(18,LT),(19,GT),(20,GT),
(21,GT),(22,GT),(23,LT),(24,LT),(25,LT),(26,LT),(27,GT),(28,GT),(29,GT),(30,GT),
(31,GT),(32,LT),(33,LT),(34,LT),(35,LT),(36,GT),(37,GT),(38,GT),(39,GT),(40,LT),
(41,LT),(42,LT),(43,LT),(44,GT),(45,GT),(46,GT),(47,GT),(48,LT),(49,LT),(50,LT),
(51,LT),(52,GT),(53,GT),(54,GT),(55,GT),(56,LT),(57,LT),(58,LT),(59,LT),(60,LT),
(61,GT),(62,GT),(63,GT),(64,GT),(65,LT),(66,LT),(67,LT),(68,LT),(69,GT),(70,GT),
(71,GT),(72,GT),(73,LT),(74,LT),(75,LT),(76,LT),(77,GT),(78,GT),(79,GT),(80,GT),
(81,LT),(82,LT),(83,LT),(84,LT),(85,GT),(86,GT),(87,GT),(88,GT),(89,GT),(90,LT),
(91,LT),(92,LT),(93,LT),(94,GT),(95,GT),(96,GT),(97,GT),(98,LT),(99,LT),(100,LT)
0918132人目の素数さん
垢版 |
2018/06/04(月) 02:04:24.66ID:t2ICP58s
正規分布の平均値周りの偶数次モーメントを求めてくださいませんか。
途中式を書いてくださると助かります。

σ^2nが出ることはわかるのですが、積分ができません。
0919132人目の素数さん
垢版 |
2018/06/04(月) 02:24:08.29ID:eLfureAF
>>918
σ=1のときは
E((x-0)^(2n))
=(1/√(2π))∫[-∞, ∞]x^(2n)exp(-x^2/2)dx
=(2/√(2π))∫[0, ∞]x^(2n)exp(-x^2/2)dx
=(2/√(2π))∫[0, ∞](2t)^(n)exp(-t)dt/(√(2t))
=(2^n/√(π))∫[0, ∞]t^(n-1/2)exp(-t)dt
=(2^n/√(π))Γ(n+1/2)
=(n-1)!!
一般はこれのσ^n倍。
0920132人目の素数さん
垢版 |
2018/06/04(月) 03:49:09.19ID:Ew3FIvyX
>>917
適当に作った問題だと思うよ
別にそれが悪いことじゃないけど
解いてもあまり面白くもないよね
0921132人目の素数さん
垢版 |
2018/06/04(月) 04:10:29.17ID:Utc1nkXv
>>920

でも本人の口ぶりだとキチンと解けて感動するとかいってたけどねぇ?
検算くらいしてから出せば余計な恥ずかしい思いしなくて済むのに。
0922132人目の素数さん
垢版 |
2018/06/04(月) 06:19:15.13ID:26vcFj3P
「いま〜と置く」の「いま」はなぜつけるんですか?
付けないと駄目なんですか?
0923132人目の素数さん
垢版 |
2018/06/04(月) 09:02:58.52ID:SYEVbRdt
A を平面の点の空でない集合とし、 f(x, y) を A で定義された関数とする。
平面の点の集合 S に対し、最大値の定理の証明の中だけで使う記号
A ≦ S と A > S を定義する。 A の任意の点 (x, y) に対し、 S に含まれる
A の点 (s, t) で f(x, y) ≦ f(s, t) をみたすものが存在するとき、 A ≦ S と
書く。 A の点 (x, y) で、 S に含まれる A の任意の点 (s, t) に対し
f(x, y) > f(s, t) となるものが存在するとき、 A > S と書く。記号 A ≦ S と
A > S の意味は関数 f(x, y) によって決まるものだが、記号からは省略した。

A が S の部分集合ならば A ≦ S である。 A は空集合ではないから、
A と S が交わらないならば A > S である。 A ≦ S ならば f(x, y) の最大値を
とる A の点で S に含まれるものがあるはずであり、 A > S ならば f(x, y) の
A での最大値をとる点は S には含まれない。

(1)と(2)のどちらが A ≦ S の定義でしょうか?

(1)
A ≦ S



∀(x, y) ∈ A, ∃(s, t) ∈ A ∩ S such that f(x, y) ≦ f(s, t)

(2)
A ≦ S



∃(s, t) ∈ A ∩ S, ∀(x, y) ∈ A such that f(x, y) ≦ f(s, t)
0924132人目の素数さん
垢版 |
2018/06/04(月) 09:03:21.86ID:SYEVbRdt
>>923

(1)だと解釈すると、

「A が S の部分集合ならば A ≦ S である。」

は成り立ちますが、

「A ≦ S ならば f(x, y) の最大値をとる A の点で S に含まれるものがある。」

は成り立ちません。

(2)だと解釈すると

「A が S の部分集合ならば A ≦ S である。」

は成り立ちませんが、

「A ≦ S ならば f(x, y) の最大値をとる A の点で S に含まれるものがある。」

は成り立ちます。
0925132人目の素数さん
垢版 |
2018/06/04(月) 09:39:15.00ID:DrdlgPon
本ぐらい自分で訂正して読め。
0926132人目の素数さん
垢版 |
2018/06/04(月) 09:48:05.29ID:NoT+jWYT
>>909
Y2=G1・X2+G2・X3+G3・X4
Y1=G1・X1+G2・X2+G3・X3
Y0=G1・X0+G2・X1+G3・X2
・はAND、+はORまたはXOR
0927132人目の素数さん
垢版 |
2018/06/04(月) 15:54:42.50ID:CW5QHAZO
sを与えられた無理数とする。
(1)任意の正の数εに対して、不等式0<|s-p|≦ε…(A)を満たす有理数pが存在することを示せ
(2)(1)においてεをある無理数に固定する。ただしεは有理数qを用いてε=qsとは表されないものとする。このとき、(A)を満たすpの最大値および最小値が存在するかを述べよ。
0928132人目の素数さん
垢版 |
2018/06/04(月) 16:08:15.24ID:CW5QHAZO
BC=1,AB=ACの二等辺三角形△ABCがある。
kを正の実数とし、ABをk:1に内分する点をK、内角∠BKCの2等分線をl、lと直線BCとの交点をLとする。

(1)Lは線分BC上にあることを示せ。

(2)l//ACのとき、KLの長さをkで表せ。
0929132人目の素数さん
垢版 |
2018/06/04(月) 16:08:31.27ID:KN0vzly3
何これ?最大値存在するか否かなんてs+εが有理数かって聞いてるだけちゃうの?
0930132人目の素数さん
垢版 |
2018/06/04(月) 16:29:54.66ID:SYEVbRdt
四角形 ABCD が、半径 65/8 の円に内接している。この四角形の周の長さが 44
で、辺 BC と辺 CD の長さがいずれも 13 であるとき、残りの2辺 AB と DA の長さ
を求めよ。

このような問題を解くとき、解があるとすれば、こうなるはずだというところまでで
解答としてはパーフェクトでしょうか?
0931132人目の素数さん
垢版 |
2018/06/04(月) 17:29:06.71ID:+L5w0iOl
アルティン環Aから環Bへの全射準同型があればBはアルティン環になります

証明はBの任意のイデアルIに対しf^-1(I)がAのイデアルになることから分かりますが、なぜ全射性が必要なのでしょうか
0932132人目の素数さん
垢版 |
2018/06/04(月) 18:04:04.38ID:CW5QHAZO
空間のx軸を中心軸とする半径1の円柱を、z軸の周りにθ回転させた円柱をC(θ)とする。
どのC(θ)にも含まれるような空間の点全体からなる領域をDとするとき、Dは球であるか。
0933132人目の素数さん
垢版 |
2018/06/04(月) 18:13:39.06ID:vjcUHvHA
宇宙飛行士とリーマン予想を証明した人はどっちの方が頭が良いですか?
0934132人目の素数さん
垢版 |
2018/06/04(月) 19:08:08.45ID:KN0vzly3
>>931

AからBへ準同型があるだけではだめに決まってるやん。
kが体、A=k、B=k[x]でfを自然なk射にするときBのどの真のイデアルIをとってもfによる引き戻しは0イデアルになる。
0936132人目の素数さん
垢版 |
2018/06/04(月) 19:12:11.41ID:nC9cJMea
ロピタルの定理って高校範囲で示せるのでは?

ロルの定理よりg'(x)≠0からg(b)≠g(a)である
そこで(f(b)-f(a))/(g(b)-g(a))=C(定数)と置く

関数F(x)=f(x)-f(a)-c{g(x)-g(a)}を考えてF(a)=F(b)である

ロルの定理からF'(c)=f'(c)-Cg'(c)=0かつa<c<bを満たすcの存在が認められる

又f(x)とg(x)の定義よりf(a)=g(a)=0である
∴f(b)/g(b)=f'(c)/g'(c)かつa<c<bを満たすcが存在する

はさみうちの原理からb→a+0のときc→a+0である
∴lim[b→a+0]f(b)/g(b)=lim[c→a+0]f'(c)/g'(c)

x→a-0は同様に, 両者を使えばx→aも示せる

∴主張(ロピタルの定理)が示された
0937132人目の素数さん
垢版 |
2018/06/04(月) 19:53:24.25ID:cy8DIAit
どうみても場違いなわからない問題があるのですが(中学レベル)、適応するスレをどなたか教えていただけませんか
0938132人目の素数さん
垢版 |
2018/06/04(月) 20:16:35.13ID:tzM+Pvvj
>>930
いいえ。

外接円の半径を R とすると
 AB = 2R sin(∠AOB/2),
 BC = 2R sin(∠BOC/2),
 CD = 2R sin(∠COD/2),
 DA = 2R sin(∠DOA/2),
また題意より
 R = 65/8,
 BC = CD = 13,
 AB+BC+CD+DA = 44,
したがって
 AB + DA = 44 -13 -13 = 18,
 ∠AOB/2 = 2arctan(4/7) = arcsin(56/65) = 59.4897626゚
 ∠BOC/2 = 2arctan(1/2) = arcsin(4/5) = 53.130102゚
 ∠COD/2 = 2arctan(1/2) = arcsin(4/5) = 53.130102゚
 ∠DOA/2 = 2arctan(1/8) = arcsin(16/65) = 14.25003゚
よって
 AB = 14
 DA = 4
0939132人目の素数さん
垢版 |
2018/06/04(月) 20:30:38.81ID:SYEVbRdt
>>938

でも問題集の解答には、解があるとすれば、こうなるはずだというところまでしか
書いてありません。
0940132人目の素数さん
垢版 |
2018/06/04(月) 20:32:21.86ID:SYEVbRdt
四角形 ABCD が、半径 65/8 の円に内接している。この四角形の周の長さが 44
で、辺 BC と辺 CD の長さがいずれも 13 である

と書いてあるので、存在のほうは出題者が保証している形の問題ではないでしょうか?
0941132人目の素数さん
垢版 |
2018/06/04(月) 21:00:27.26ID:1w5RTCiM
>>940
受験数学ならそうだな。その手のやつは存在証明しなくても点はもらえる。
しかし満点もらえるんだからこれでOKとか思ってる奴はソコソコ止まり。
0942132人目の素数さん
垢版 |
2018/06/04(月) 21:34:33.17ID:N4QA1GQW
>>939
その問題集の解答のスクショを見せろ
ちなみに東大の問題なので探せば幾らでも載ってる本が見つかる
疑問があるならそれらも参照すればいいんじゃね
0943132人目の素数さん
垢版 |
2018/06/04(月) 22:09:21.37ID:SYEVbRdt
>>923-924

「A の任意の点 (x, y) に対し、 S に含まれる A の点 (s, t) で
f(x, y) ≦ f(s, t) をみたすものが存在する」の意味ですが、これ
を素直に解釈した(1)の意味らしいです。

「A ≦ S ならば f(x, y) の最大値をとる A の点で S に含まれるものがある。」

は本当に成り立ちますか?

(1)
A ≦ S



∀(x, y) ∈ A, ∃(s, t) ∈ A ∩ S such that f(x, y) ≦ f(s, t)

(2)
A ≦ S



∃(s, t) ∈ A ∩ S such that ∀(x, y) ∈ A, f(x, y) ≦ f(s, t)
0945132人目の素数さん
垢版 |
2018/06/04(月) 22:12:35.63ID:SYEVbRdt
>>942

どんな問題集でも、

>>930

の問題に限らず、他の問題でも存在のほうは出題者が
保証しているから示さないという立場の解答ばかりである
といってもいいような状況ではないでしょうか?

ですので、わざわざ画像をアップロードするまでもないように
思いますが、そうではない問題集も存在するのでしょうか?
0946132人目の素数さん
垢版 |
2018/06/04(月) 22:14:25.88ID:SYEVbRdt
>>923-934
>>943

正誤表を見てみてもこの件については書いてありませんでした。
0947132人目の素数さん
垢版 |
2018/06/05(火) 00:41:33.11ID:+e1CS0n2
まぁ残念ながら>>945の言ってることは正しいな。
存在性の確認なんか全く無意味のような解答のほうが正しいような風潮が受験数学の世界にははびこってる。
しかし、だから存在証明の抜けてる解答が数学的に正しいわけではないし、その解答で納得してるような生徒は結局大学では行き詰まる。
ホントに理系で一歩抜け出ることができるのはそんな解答よんで少なくとも ”なんかおかしい” と思える人間に限られる。
0948132人目の素数さん
垢版 |
2018/06/05(火) 00:44:07.19ID:SY5SVVbZ
これこれこういうときどうなりますか?って聞かれてるのに、本当にそういう状況が成しえるかどうかを気にする人は、バカな人です
少なくとも国語はできないんでしょうね
0949132人目の素数さん
垢版 |
2018/06/05(火) 01:01:08.77ID:SY5SVVbZ
Q:ドラえもんとコロ助が戦ったらどっちが勝つんですか?

A:どっちも架空の存在なので、両者が戦うという事象は起こるはずがないため質問自体が無意味

はぁ?て感じになりますよね
>>930のような問題で存在性にぐちぐち言うということは、このようなことなわけですね
0950132人目の素数さん
垢版 |
2018/06/05(火) 01:01:23.61ID:+e1CS0n2
行き詰まったのがなんか言ってるなwww
0951132人目の素数さん
垢版 |
2018/06/05(火) 01:03:32.24ID:k73mhkxG
「 P という前提条件のもとで、Q という結論が成り立つことを示せ 」

という問題 ―― すなわち、「 P⇒Q 」が真であることを示せという問題に対して、

「 P を用いて Q を導いただけではダメ。P が真であることまでチェックしなければ不完全。大学では行き詰まる」

などと考えるのはただのバカ。なんで「 P⇒Q 」を示すときに「 P の 」真偽を気にするんだよw

・ P が偽のときは「 P⇒Q 」は真。
・ P が真のときは、Q が真であるときに限り「 P⇒Q 」は真。

従って、我々が気にすべきは「 Q の 」真偽だけであり、
Qを導いだたけで完全な解答になるのであり、

「 P が真であることまでチェックしなければ不完全。大学では行き詰まる 」

なんてことにはならないんだよ。むしろ P の真偽を気にしてるバカの方が行き詰まるわ。
「 P⇒Q 」が何を表しているのか理解してない証拠だからな。
0952132人目の素数さん
垢版 |
2018/06/05(火) 01:08:20.35ID:+e1CS0n2
>>951に書いてありような反論は聞き飽きてるんだよ。それ自分が発見した新解釈だとでも思ってる時点でおめでたいんだよ。
まぁ、数学っぽいことやって自己満足しとけ。
0953132人目の素数さん
垢版 |
2018/06/05(火) 01:13:39.35ID:k73mhkxG
>>952
意味が分からない。自分が発見した新解釈ってどういうこと?
>>951なんて常識だろ?「 P⇒Q 」について書いてるだけだよ?

そんな常識が「聞き飽きてる」ってのも意味不明。
聞き飽きてるくらいに「 P⇒Q 」のことを把握してるなら、
そもそも>>947みたいなバカな間違いはしないよね?
0955132人目の素数さん
垢版 |
2018/06/05(火) 01:15:27.89ID:+e1CS0n2
>>954
はいはい。わかりませんでした。君の勝ち〜。おめでと〜〜
0956132人目の素数さん
垢版 |
2018/06/05(火) 01:20:46.27ID:SY5SVVbZ
今日も「解いた側」の圧勝かぁ・・・。
毎日毎日、ラクラク解ける問題ばかりだから常勝なんだよね・・・。
たまには、解けない解けないっと悩んで負けてみたい、それが今の切実な悩み。
0958132人目の素数さん
垢版 |
2018/06/05(火) 01:52:03.86ID:K/6mvYAb
y=(exp(-a*x))*(b+(c+a*b)*x)

いきないりですけど、これの逆関数が計算できません。
つまりxに付いて解くにはどうやったらいいでしょうか。
0959132人目の素数さん
垢版 |
2018/06/05(火) 01:58:03.26ID:+e1CS0n2
解ける問題ばっかりに囲まれて羨ましいかぎりだなぁ
0961132人目の素数さん
垢版 |
2018/06/05(火) 07:02:08.26ID:RI7aB28L
>>886 >>897 >>899

>>887 より
1または4のカード j枚
2 のカード k枚
3 のカード (3n-j-k)枚
のとき
 N ≡ j+2k ≡ j-k (mod 3)
∴ j-k ≡ 0 (mod 3) となる確率を考える。

P(n) = {1/C[10n,3n]} Σ[0≦k≦2n,0≦j+k≦3n] C[5n,j] C[2n,k] C[3n,3n-j-k] {ω^(j-k) + ω^(k-j) + 1}/3,
ω = (-1+i√3)/2  … 1の3乗根。

P(1) = 1/3 + 1/C[10,3]
P(2) = 1/3 + 1/C[20,6]
0962132人目の素数さん
垢版 |
2018/06/05(火) 07:10:52.13ID:1s9rbgtS
四角形 ABCD が、半径 65/8 の円に内接している。この四角形の周の長さが 44
で、辺 BC と辺 CD の長さがいずれも 13 であるとき、残りの2辺 AB と DA の長さ
を求めよ。

「四角形 ABCD が、半径 65/8 の円に内接している。この四角形の周の長さが 44
で、辺 BC と辺 CD の長さがいずれも 13 である」

と言っているので、やはりそのような四角形が存在するが、その2辺 AB と DA の
長さを求めよ、と言っていますよね。

もし、この問題でそのような四角形がそもそも存在しないとすると、出題ミスという
ことになるかと思います。 👀
Rock54: Caution(BBR-MD5:1341adc37120578f18dba9451e6c8c3b)
0963132人目の素数さん
垢版 |
2018/06/05(火) 07:16:15.14ID:lLmH4xhF
>>958
>これの逆関数が計算できません。
W関数
0964132人目の素数さん
垢版 |
2018/06/05(火) 07:55:14.05ID:RI7aB28L
>>958

いきなりですけど
X = -a {b/(c+ab) + x},
とおくと
 exp(-ax) = exp(X) exp{ab/(c+ab)},
 b + (c+ab)x = -{(c+ab)/a}X,
辺々かけて
 y = -{(c+ab)/a} exp{ab/(c+ab)} X・exp(X),
 X・exp(X) = -{a/(c+ab)} exp{-ab/(c+ab)} y,
∴ X = W(-{a/(c+ab)} exp{-ab/(c+ab)} y) … Lambert のW函数
 x = -X/a - b/(c+ab) に入れる。

a=0 のときは y = b+cx
c+ab=0 のときは y = b・exp(-ax)
0965132人目の素数さん
垢版 |
2018/06/05(火) 08:04:00.48ID:SY5SVVbZ
>>962
Q:ドラえもんとコロ助が戦ったらどっちが勝つんですか?

A:どっちも架空の存在なので、両者が戦うという事象は起こるはずがないため質問自体が無意味

はぁ?て感じになりますよね
>>930のような問題で存在性にぐちぐち言うということは、このようなことなわけですね
0966132人目の素数さん
垢版 |
2018/06/05(火) 08:30:31.10ID:1s9rbgtS
実数 x は2次方程式 x^2 = -1 を満たしている。

x^4 の値を求めよ。
0968132人目の素数さん
垢版 |
2018/06/05(火) 08:43:21.78ID:SY5SVVbZ
1+1の答えを求めるのに、自然数の和の定義の無矛盾性から考えますか、って話ですね
0970132人目の素数さん
垢版 |
2018/06/05(火) 10:37:37.51ID:7Wnt1KgV
添付の画像の問題を教えていただきたいです
(書き込みが多くあり見辛い画像となってしまい申し訳ありません)
2枚目はzに対する微分方程式を立ててみたのですが、自信がありません…
何卒宜しくお願い致しますm(_ _)m

https://i.imgur.com/Eex87JJ.jpg
https://i.imgur.com/Eex87JJ.jpg
0972スイカ割り
垢版 |
2018/06/05(火) 11:05:10.39ID:y5eMxvts
2÷3は割りきれないけど、実際に長さ2mの(1次元的な)棒を3等分しようとしようとすれば出来るの?
それから3等分してできた棒の1つの長さを計ると何mになるの?
教えて!goo
0973132人目の素数さん
垢版 |
2018/06/05(火) 11:07:44.66ID:CPUOXfyJ
不確定性原理により正しい真の値というのは測定不可能です
残念でしたね
0975132人目の素数さん
垢版 |
2018/06/05(火) 18:44:00.76ID:r5HYSakC
「0でない2つの関数f(x)とg(x)を用いてf(x)g(x)=0となるようなものを一組挙げよ」

お願いします。
0978132人目の素数さん
垢版 |
2018/06/05(火) 19:45:43.93ID:JrF6lphB
リーマン予想とP≠NP予想を証明したいのですが、とりあえず数学と物理学と計算機科学の全分野を網羅した方が良いのでしょうか?
0979132人目の素数さん
垢版 |
2018/06/05(火) 21:22:09.01ID:MR5aOihI
AをR^nの凸集合、BをAの閉包の内点とするとき、B⊂Aを示せ
どのようにやればいいのでしょうか
0981132人目の素数さん
垢版 |
2018/06/05(火) 22:22:19.86ID:y93ap0Jy
位相知らないなら黙ってればいいのに、と一瞬思ってしまった

>>979
問題文は正確に書いてね
0982132人目の素数さん
垢版 |
2018/06/05(火) 22:27:27.97ID:ekSWY8PA
「次の命題が成り立つことを対偶を用いて証明せよ
x,yがともに正の数のとき、x^2+y^2≧6 ならば x≧√3 または y≧√3である」
これって元の命題成り立ちますか?
0985132人目の素数さん
垢版 |
2018/06/05(火) 22:32:40.58ID:ekSWY8PA
ありがとうございます
0988132人目の素数さん
垢版 |
2018/06/05(火) 23:34:16.84ID:PbqFpKWz
内点は点
0990132人目の素数さん
垢版 |
2018/06/06(水) 00:29:01.43ID:/3pfmLjy
>>980
>>981
ああ、すいません
内点全体の集合(内部)です

正しい問題文は以下のとおりです

AをR^nの凸集合
Bを、Aの閉包の内点全体の集合とするとき、B⊂Aを示せ
0991132人目の素数さん
垢版 |
2018/06/06(水) 01:44:15.29ID:I5g3t//e
>>990
B-A∋xとするとxの近傍で全部Aの触点なのがある
0992132人目の素数さん
垢版 |
2018/06/06(水) 06:23:47.51ID:xxwxn7ab
>>975

xが自然数または0のとき  f(x) = 1,g(x) = 0,
xが負の整数のとき     f(x) = 0,g(x) = 3,
xが有理数(≠整数)のとき f(x) = 5,g(x) = 0,
xが代数的無理数のとき   f(x) = 0,g(x) = 6,
xが超越的実数のとき    f(x) = 7,g(x) = 0,
xが複素数(≠実数)のとき f(x) = 0,g(x) = 2,
xが4元数のとき      f(x) = 0,g(x) = 4,
xが8元数のとき      f(x) = 0,g(x) = 8,

>>982

2・Max{xx-3,yy-3} ≧ (xx-3) + (yy-3) = xx+yy - 6,
0994132人目の素数さん
垢版 |
2018/06/06(水) 18:34:09.94ID:Q1+o1co8
1個のさいころを3n回続けて振り,
出た目の数の和のS, 二乗の総和をTとする.
条件3|Tのもとで, 3|Sである条件付き確率を求めよ.

この問題を教えてください
0996132人目の素数さん
垢版 |
2018/06/06(水) 19:38:50.86ID:i4OBaZ0i
>>994
何これ解けない
0997132人目の素数さん
垢版 |
2018/06/06(水) 22:57:18.67ID:eT3pen36
>>994
P(3|T)
=(1/3+2/3)^3n + (1/3 + 2ω/3)^3n + (1/3 + 2ω^2/3)^3n
=1 + (√3/9i)^n + (-√3/9i)^n

P(3|T & 3|S)
=(1/3+1/3+1/3)^3n + (1/3+ω/3+1/3)^3n + (1/3+ω^2/3+1/3)^3n
+(1/3+1/3+ω/3)^3n + (1/3+ω/3+ω/3)^3n + (1/3+ω^2/3+ω/3)^3n
+(1/3+1/3+ω^2/3)^3n + (1/3+ω/3+ω^2/3)^3n + (1/3+ω^2/3+ω^2/3)^3n
=1+3(√3/9i)^n + 3(-√3/9i)^n
0998132人目の素数さん
垢版 |
2018/06/07(木) 02:05:34.17ID:4qUpYzhu
うんこぶりぶり。
0999132人目の素数さん
垢版 |
2018/06/07(木) 06:43:56.49ID:LV2DjXT6
>>994

>>961 より
1または4の目 j回
2または5の目 k回
3または6の目 (3n-j-k)回
出たとき
 S ≡ j-k (mod 3)
 T ≡ j+k (mod 3)

P(3|T) = P(j+k≡0)
 = Σ[L=0,3n] C[3n,L] (2/3)^L (1/3)^(3n-L) {1 +ω^L +ω^(-L)}/3
 = {(1/3 + 2/3)^(3n) + (1/3 + 2ω/3)^(3n) + (1/3 + 2/(3ω))^(3n)}/3
 = {1 + (i/√3)^(3n) + (-i/√3)^(3n)}/3
 = {1 + 2(1/3)^(3n/2)cos(nπ/2)}/3,

P(3|T ∧ 3|S) = P(j≡0 ∧ k≡0)
 = Σ[0≦j+k≦3n] (3n)!/{j! k! (3n-j-k)!} (1/3)^(3n) {1+ω^j +ω^(-j)}/3・{1+ω^k +ω^(-k)}/3
 = {(1/3+1/3+1/3)^(3n) + 2(1/3+1/3+ω/3)^(3n) + 2(1/3+1/3+1/3ω)^(3n) + (1/3+ω/3+ω/3)^(3n) + (1/3+1/3ω+1/3ω)^(3n) + 2(1/3+ω/3+1/3ω)^(3n)}/9
 = {1 + 2((2+ω)/3)^(3n) + 2((2+1/ω)/3)^(3n) + ((1+2ω)/3)^(3n) + ((1+2/ω)/3)^(3n) + 0^(3n)}/9
 = …
1000132人目の素数さん
垢版 |
2018/06/07(木) 13:54:52.19ID:DXwRCU0Z
q
10011001
垢版 |
Over 1000Thread
このスレッドは1000を超えました。
新しいスレッドを立ててください。
life time: 33日 14時間 39分 36秒
10021002
垢版 |
Over 1000Thread
5ちゃんねるの運営はプレミアム会員の皆さまに支えられています。
運営にご協力お願いいたします。


───────────────────
《プレミアム会員の主な特典》
★ 5ちゃんねる専用ブラウザからの広告除去
★ 5ちゃんねるの過去ログを取得
★ 書き込み規制の緩和
───────────────────

会員登録には個人情報は一切必要ありません。
月300円から匿名でご購入いただけます。

▼ プレミアム会員登録はこちら ▼
https://premium.5ch.net/

▼ 浪人ログインはこちら ▼
https://login.5ch.net/login.php
レス数が1000を超えています。これ以上書き込みはできません。

ニューススポーツなんでも実況